0% found this document useful (0 votes)
6K views

SAT Workbook (Answer Key)

Uploaded by

Huseyn Huseynov
Copyright
© © All Rights Reserved
We take content rights seriously. If you suspect this is your content, claim it here.
Available Formats
Download as PDF, TXT or read online on Scribd
0% found this document useful (0 votes)
6K views

SAT Workbook (Answer Key)

Uploaded by

Huseyn Huseynov
Copyright
© © All Rights Reserved
We take content rights seriously. If you suspect this is your content, claim it here.
Available Formats
Download as PDF, TXT or read online on Scribd
You are on page 1/ 510

Contents

Introduction 6
How to Use This Book 6
Learn to Practice Effectively 3

Reading 5
Reading Unit #1 7
Reading Introduction 8
Active Reading 13
General Strategy 16
Applying Active Reading - Sulfur Mining & Chain Letters 18
Homework Drill #1 - The Thirty-Nine Steps 23
Homework Drill #2 - Anti-Suffragists and War 25
Homework Drill #3 - The Sea Sapphire 27

Reading Unit #2 31
Mark Up the Passage 33
Question Types - Part 1 - FAQ 37
Recommended Reading List 47
Homework Drill #1 - Old Creole Days 51
Homework Drill #2 - Cut Out the Middle Man 53
Homework Drill #3 - Haiti Earthquake 56

Reading Unit #3 58
Infographics 60
Paired Passages 71
Homework Drill #1 - Pragmatism 82
Homework Drill #2 - Space Junk 84
Homework Drill #3 - Borders of Comedy 88

Reading Unit #4 91

SAT v17.10 Unauthorized copying or reuse of any part of this page is illegal.
(SAT Teacher Edition)
Strategy Review 93
More Question Types 94
Reading Vocabulary: Tone / Attitude 105
Reading Vocabulary: Other Essential Words 109
Reading Vocabulary: Word Roots 113

Writing 117
Writing Unit #1 119
Introduction 120
General Strategy 123
Homework Drill #1 - In Search of Baby Dinosaurs 129
Homework Drill #2 - Note-Taking: A Surprising Discovery 132
Homework Drill #3 - The Benefits of Bike Lanes 135

Writing Unit #2 139


Parts of Speech 140
Adjectives vs. Adverbs 149
Verb Tense 153
Subject Verb Agreement 163
Pronoun Agreement and Clarity 170
Homework Drill #1 - Meal Planning 181
Homework Drill #2 - The Value of a Used Car 184
Homework Drill #3 - Antibodies: The Power Within 187

Writing Unit #3 191


Sentences & Fragments 193
Run-On Sentences 201
Colons, Dashes, and Semicolons 207
Commas and Non-Essential Clauses 211
Possessives, Plurals, and Contractions 217
Homework Drill #1 - Hand-Washing Hullabaloo 222
Homework Drill #2 - Coaching Youth Sports: Fun and Fulfilling 225
Homework Drill #3 - No Kidding: A Fresh Approach to Landscaping 228
Writing Unit #4 231
Informational Graphics 232
Logical Transitions 241
Relevance: Adding & Removing Info 247
Reordering a Sentence or Paragraph 253
Homework Drill #1 - When Time Is Muscle 260
Homework Drill #2 - A Day on the Water 263
Homework Drill #3 - Nursing: An Evolving Profession 266

Writing Unit #5 269


Diction and Idioms 270
Parallelism and Word Pairs 279
Misplaced Modifier 282
Redundancy & Wordiness 285
Homework Drill #1 - Stand and Deliver 290
Homework Drill #2 - Happiness Matters 293
Homework Drill #3 - A New Approach to Conservation 296

Math 299
Math Unit #1 - Math Strategy Guide 301
General Math Strategy 302
Solving Backwards 311
Picking Your Own Numbers 315
Basic Math - Part 1 319
Basic Math - Part 2 323
Homework Drill #1 327
Homework Drill #2 328
Homework Drill #3 329

SAT v17.10 Unauthorized copying or reuse of any part of this page is illegal.
(SAT Teacher Edition)
Math Unit #2 - Heart of Algebra 331
Translating Word Problems - Part 1 333
Translating Word Problems - Part 2 339
Solving Linear Equations - Part 1 343
Solving Linear Equations - Part 2 349
Interpreting Linear Equations 353
Linear Inequalities & Absolute Values 361
Solving Linear Systems 367
Graphs of Lines 373
Homework Drill #1 377
Homework Drill #2 378
Homework Drill #3 379

Math Unit #3 - Problem Solving & Data Analysis 381


Rates, Ratios, and Proportions - Part 1 383
Rates, Ratios, and Proportions - Part 2 389
Percentages 393
Probability 399
Data Tables 403
Charts and Graphs 407
Mean, Median, Mode, and Range 413
Homework Drill #1 418
Homework Drill #2 419
Homework Drill #3 420

Math Unit #4 - Passport to Advanced Math 421


Roots & Exponents 423
Operations with Polynomials - Part 1 429
Operations with Polynomials - Part 2 435
Solving Quadratics 441
Graphing Quadratics (Parabolas) 447
Function Notation 457
Real-Life Nonlinear Models 465
Homework Drill #1 468
Homework Drill #2 469
Homework Drill #3 470
Math Unit #5 - Additional Topics 471
Angles 473
Triangles 479
Circles 485
Area & Volume 491
Complex Numbers 495
Basic Trigonometry 499
Homework Drill #1 503
Homework Drill #2 504
Homework Drill #3 505

SAT v17.10 Unauthorized copying or reuse of any part of this page is illegal.
(SAT Teacher Edition)
How to Use This Book
1. Learn about the SAT. Make sure you're familiar with the instructions for each
section, so you'll know exactly what to expect on test day.

2. Complete an authentic SAT test to establish a baseline score.

3. Meet with your instructor to analyze your scores and identify the areas of greatest
potential improvement.

4. Work through the book, focusing on those areas of greatest potential


improvement. Take lots of notes in the margins because this course workbook will
become your personalized study guide for review sessions.

5. Reinforce the lessons from your workbook by logging into your online account,
submitting your answers, and watching the video solutions for each problem you
answered incorrectly.

6. Complete the custom worksheets that your instructor creates for you through the
online system.

7. Take additional practice tests throughout the course to document your progress,
quantify your improvement, and update your areas of greatest potential
improvement.

Note: This book is a comprehensive SAT workbook. But, because you had already
mastered some SAT concepts before you showed up (and because time is limited)
your tutor will choose among the lessons in this workbook to customize a course
based on your needs. Don't be surprised if you and your instructor are skipping
around a bit. That's the most efficient way to raise your score!
Learn to Practice Effectively
1. Eliminate distractions! Start with the electronics. Turn off the TV, forget about
your Facebook, silence your phone, and put it in another room. Oh, and lose the
headphones. You won't be allowed to use any of these things on test day.

2. Most of the time snacks aren't a problem, but save them for a break in the
action. Take a break. Eat. Get back to studying. Don't try to do both at once.

3. Learn to spot the common SAT Reading problem types and memorize the
special strategies for each of these problems.

4. Read the questions and answers aloud, scrutinizing every word. Often, a
single word can make the difference between a correct answer and an incorrect
answer.

5. As you use process of elimination, consciously ask yourself, "Why is this answer
choice correct/incorrect?" Don't rush this moment. This is where the learning
happens.

6. Review every single problem you miss. Never dismiss a wrong answer as a
"silly mistake." There is something to be learned from every single wrong
answer!

7. Pay attention to trends. Which passage type(s) do you enjoy the most/least?
Which type(s) do you do best/worst on? Use this information to decide which
passages to attack first on test day. Pay attention to the fine line between
working quickly and rushing. Practice working quickly without rushing. Think
of yourself as a race car with a number of gears. Find the gear that will move
you as fast as possible - without causing you to lose control.

The advice on this page is so important


that we've included it in this book a
couple of times. Get used to seeing
these recommendations. More
importantly, get used to following
them!

SAT v17.10 Unauthorized copying or reuse of any part of this page is illegal. 3
(SAT Teacher Edition)
4
Reading

SAT v17.10 Unauthorized copying or reuse of any part of this page is illegal. 5
(SAT Teacher Edition)
6
1 Reading Introduction 8

Active Reading 13
Reading Unit #1

General Strategy 16

Applying Active Reading 18

Homework Drills:

1 2 3
23 25 27

SAT v17.10 Unauthorized copying or reuse of any part of this page is illegal. 7
(SAT Teacher Edition)
Reading Introduction

Learn to Study Effectively


1. Eliminate distractions! Start with the electronics. Turn off the TV, forget about
social media, silence your phone and put it in another room. Oh, and lose the
headphones.

2. Most of the time snacks aren't a problem, but save them for a break in the
action. Take a break. Eat. Get back to studying. Don't try to do both at once.

3. Learn to spot the common SAT Reading problem types and memorize the
special strategies for each these problems.

4. Read the questions and answers aloud, scrutinizing every word. Often, a
single word can make the difference between a correct answer and an incorrect
answer.

5. As you use process of elimination, consciously ask yourself, "Why is this answer
choice correct/incorrect?" Don't rush this process. This is where the learning
happens.

6. Review every single problem you miss. Never dismiss a wrong answer as a
"silly mistake." There is something to be learned from every single wrong
answer!

7. Pay attention to trends. Which passage type(s) do you enjoy the most/least?
Which type(s) do you do best/worst on? Use this information to decide which
passages to attack first on test day. Pay attention to the fine line between
working quickly and rushing. Practice working quickly without rushing. Think
of yourself as a race car with a number of gears. Find the gear that will move
you along as fast as possible - without causing you to lose control.

8
Find Strategies that Work For You.
For as long as there have been standardized tests, there have been strategies,
tips, and "tricks" that claim to outsmart the test. Some test prep companies
claim that reading the questions first is the secret to a perfect score. Others
swear by underlining key phrases, jotting notes in the margins, or reading
only the first and last sentences of each paragraph. Some of these strategies
are widely-accepted. Some are disputed, so talk to your tutor before you
drastically alter your approach to the SAT Reading section.

The problem with these strategies is that they're "one-size-fits-all." Worse,


these strategies encourage students to think in terms of quick fixes when in
reality the test does a pretty good job of evaluating how well students read,
understand what they've read, and retain information.

And, while it's true that every student has the same goal on the SAT Reading
test, it's not true that every student faces the same set of challenges in dealing
with this section of the test. So before we get bogged down with a bunch
of strategies that may or may not work for you, let's consider what all these
strategies have in common. They're all intended to make you a better reader
by encouraging you to read actively.

With that in mind, work with your tutor to find an approach that helps you
read the passage actively, understand it thoroughly, and retain the important
information effectively.

Your tutor may recommend one or more of the strategies mentioned in the
first paragraph, above. Whatever approach you and your tutor decide to
follow, remember that your goal is to read actively -- not to outsmart the SAT!

Going forward, this workbook will mention tips and tricks when appropriate,
but most of the instruction will focus on reading actively and using process of
elimination effectively.

SAT v17.10 Unauthorized copying or reuse of any part of this page is illegal. 9
(SAT Teacher Edition)
Format and Difficulty of the SAT Reading Test:
• The SAT Reading Section consists of 5 passages, followed by 10 or 11 questions each.
• The questions in this section are NOT organized according to difficulty, so if you're stuck on a
difficult problem, don't hesitate to skip it and come back later.

• You'll have 65 minutes to read passages and answer questions.


G E S : im a t e ly 500-750
PASS A pro x
READING ll y c o n s ist of ap ld Literatur
e,
n e r a W o r
Passages

ge
c lu d in g U.S. and e . T y p ic ally, you
in ienc
n topics s, and Sc hat includ
e
words o o c ia l S c ie n c e
r t e s t t
nd S es pe
History a s e e t w o passag
ct to
can expe
hics.
infograp

Q: Q:
n fo- ic
I aph
gr
ing
Q: Q:
a d e
Re ssag
Pa
Q: Q: Q:
Q: Q: Q:
S: ns
D IN G Q UESTION y 10 o r 11 questio
RE A ed b
s a g e s a re follow / t h eme, aut
hor's
Pas id e a ,
in g o n the main u s e o f evidence
focu s ation ,
n d s t y le , organiz o t her thing
s.
t o n e a d a f e w
e o f la n guage, an
us

10
The Purpose is More Important than the Topic.
EXPOSITORY NARRATIVE RHETORICAL
The purpose of an expository A narrative passage tells a story. A rhetorical passage is basically a
passage is to objectively present It typically deals with narration persuasive essay. In other words,
information. Usually these read of events and the revelation of a this is an argument in support of
like a research report. They begin character. an idea or course of action.
by introducing a problem or
mystery and then proceed to In other words, these passages These arguments tend to be well
explain the response or research require you to ask yourself, structured and supported by a
that's been done to solve the "What happened, to whom, number of pieces of evidence.
problem or understand the and how does everyone involved
mystery. feel about the events in the As you read, watch for the
passage?" author's thesis, supporting
These passages sometimes share evidence, and any points that
more than one theory or point These passages present a tiny the author refutes or concedes.
of view on the issue without piece of a larger story, but they
endorsing one side or the other. have a beginning, a middle, and Paired passages often feature
a revelation at the end. rhetorical passages, presenting
As you read, pay attention arguments on each side of
to the sequences of events, As you read, focus on the plot, an issue. In that case, pay
cause and effect relationships, characters, and mood of the attention to the specific issues
comparisons, and possible passage. Pay attention to the that the authors agree and
explanations for unexplained characters' feelings about disagree on.
phenomena. events and one another.
Remember that feelings are
usually implied, not stated.

AK OUT. . . in Australia,
DON’T FRE r b e h av io r
vote le
y re a d p a s s ages about es, or one man's strugg
You ma a rc h , s u p e rvolcano e e d to b e a n expert
e
stem cell res for his son. You don’t n erything you need to
sh v
to catch a fi se topics. On this test, e .
e e
on any of th ined within the passag
ta
know is con
!
e k n o w le d ge required
No outsid
, p ay a tte ntion to the
age e
d in g a R h e torical Pass some questions about th t.
When rea n
expect to se pment of the argume
e
t h e s is a n d lo
author's eve
r t in g e v id ence and d
suppo
on to
e Pa s s a g e , pay attenti You should
g a Narrativ s of events.
When readin elings and description t the author's beliefs,
u
everyone's fe answer questions abo
to
be prepared le.
y
tone, and st
SAT v17.10 Unauthorized copying or reuse of any part of this page is illegal. 11
(SAT Teacher Edition)
Time is a factor
There are five passages and 52 questions to be completed in 65 minutes. That's just
13 minutes to read each passage and answer all of its questions.

Let's say you read the passage in four minutes. That leaves 540 seconds to answer
10 or 11 questions. That means you'll have around 50 to 55 seconds per question.
Clearly, time is a factor!

Make an effort to boost your reading speed on all passage types. If you can work
quickly through the passage types you enjoy without making mistakes, then you can
bank some time for the sections you find more challenging.

For example, if you're good at US & World Literature passages and can complete that
section in just 10 minutes, you can reallocate the time you saved to another section.
That extra minute on a challenging passage can make a big difference.

As you continue to practice, pay attention to your strengths and weaknesses. Know
which sections tend to be quicker or slower for you, and use that information to make
good decisions on test day.

Raise Your Reading Score Between Sessions


1. Read every single day. Don't go to bed at night without reading at least one
op-ed article from a newspaper or magazine, such as TIME, The Scientific American,
The New Yorker, or Smithsonian.
2. Read at least two full articles per week. Start with articles on topics you enjoy
like sports, fashion, video games, travel, etc. It doesn't matter what it's about
as long as it's well written and you're into the subject matter.
3. Read at least two full articles per week that examine subjects that you have
absolutely no interest in reading about. This is your opportunity to practice
remaining focused even when you're bored out of your mind.
4. Always be reading a book for pleasure. Biographies are great. The Classics are
very good. Nonfiction is good. Paranormal teen romance novels are... technically
allowed.
5. Whatever you're reading, practice like it's game day. In other words, make sure
that you're reading actively and asking yourself questions about the main idea,
tone, and relationships between characters/events/ideas.
6. As you read, practice visualizing the characters and events in the story.

12
Active Reading

Five Tips for Active Reading


Have a Conversation with the Author
If someone were trying to tell you something that would help you do better on the
SAT, you'd pay close attention and ask a lot of questions, right? Well that's exactly
what each reading passage is trying to do!

Treat each passage like an important conversation. Pay close attention and
ask the writer lots of questions. Make sure you're following what he's saying. If you
get confused, ask him to repeat that last bit. Pay attention to your reaction to new
information in the passage. Is the information expected or unexpected?

Make sure you understand why the writer bothered to speak to you in the
first place. Does he or she want to tell you a story, teach you something new, or
change your opinion on a topic?

Someone Would Love Reading This. Be That Someone!


Occasionally, students report that "Nothing really happened in the passage." Well, put
that thought out of your head right now because you will never read an SAT passage in
which nothing really happens.

Suppose you've just read a prose fiction passage about a mother in 1890 attempting to
teach her daughter the proper etiquette for an upcoming social event. Pretty grim, right?

Not so fast! Someone out there would love this story and it's your job to become that
person for the next few minutes - just long enough to learn the most important facts
and interesting details.

Naturally, you won't be able to remember all the details, but make sure you're able to
convey the main idea/event of the story, mood, relationships among characters,
their motivations, what happened to these characters, and how they respond to
these events.

SAT v17.10 Unauthorized copying or reuse of any part of this page is illegal. 13
(SAT Teacher Edition)
Confused or Distracted? Refocus Your Attention.
You're not expected to understand everything the first time you read the passage.
These passages are selected for the SAT because they often require multiple readings.
With that in mind, don't beat yourself up when you get confused or distracted while
reading a dense passage. Staying calm and collected is part of the test after all.
Simply take a deep breath and reread the part you didn't understand the first
time.

Remember, you are not expected to understand everything the first time. Seriously,
you're not.

You are, however, expected to reread the passage (a few times if necessary) to make
sure you understand it well enough to answer the questions. So be prepared to
reread the passages a few times. Don't let your brain give up on a sentence or the
passage as a whole just because its meaning is not immediately clear to you. Stick
with it because the ideas expressed in the passages are usually pretty simple
once you get past the complex vocabulary and phrasing.

Make a Note of Redirection Words in the Passage


Even though passages can examine virtually any topic (within the three categories),
the structure of each passage is pretty similar. Passages often begin by making a
main point, then offer specific examples to support that point, and then examine the
significance of that point.

The following words and phrases indicate that the passage continues in the
same direction:

furthermore, therefore, as a result, so we see, because of this, also, additionally,


consequently, and thus.

Sometimes, however, a passage changes direction, and instead of emphasizing a key


point, it goes on to refute that point or introduce a competing theory or idea. Marking
up the passage when you see a redirection word will help you to find specific pieces
of information later.

The following words and phrases indicate a change in direction:

however, but, by contrast, on the other hand, and surprisingly.

14
Mark up the Passage
Different tutors have different methods for "marking-up" the passage and/or leaving
notes in the margins. Talk to your tutor for specific tips, but the general idea is to
summarize the passage as you go. There are two main benefits to doing this.

First, it will help you to incorporate the information from each new paragraph into your
general understanding of the passage. If a new paragraph doesn't seem to fit with
your general understanding, then you'll need to reconsider the main idea of the
passage.

Second, it forces you to admit when you've gotten confused or lost your focus. If you
can't summarize a paragraph into a few words, then you probably need to reread
that paragraph!

Now, as we mentioned earlier, this approach is not for everyone. Talk to your tutor, and
use whatever approach you both agree works best for you. If you do decide to use this
approach, make sure you've practiced it a lot before you take the real test.

One Last Thing Before We Talk Strategy...


On the next page, we've outlined the General Strategy for SAT Reading.
There are a variety of reasons why we recommend you at least try this strategy.

First, it's not complicated, so it's easy to learn.

Second, unlike more complicated strategies, it becomes second nature, so it's easy
to follow - even with the stress of test day.

Third, by focusing on process of elimination, it encourages you to read critically.

And fourth, it works.

Obviously, there is no perfect, silver bullet, one-size-fits-all, best strategy for everyone,
but this one is a pretty good start. Before you commit to a more complicated strategy
(i.e. reading all the questions first), give the general strategy a try. You'll probably find
that it works for you.

Skeptical? That's good. That means you're reading critically. Keep it up.

SAT v17.10 Unauthorized copying or reuse of any part of this page is illegal. 15
(SAT Teacher Edition)
General Strategy

The General Strategy for SAT Reading


1. Actively read the passage first.

2. Read the question slowly, carefully, and twice if necessary!


Nearly half of all mistakes in the Reading section are the result of students misreading the
questions. Don't become a statistic! Make sure that you understand the question.

3. Review the relevant portion of the passage.


If the question directs you to a line number or a specific paragraph, then go back and read it
a second time. Reference the notes you may have taken in the margins. If the question asks
about the passage as a whole, then you probably won't need to reread any particular part of the
passage. Instead, take a moment to look over your notes in the margins. Make sure you've got a
good idea what the main idea is.

4. * * * ANSWER THE QUESTION IN YOUR HEAD! * * *


Don't even think about looking at the answer choices until you've attempted to answer the
question in your own words. On difficult questions, multiple answer choices may sound
pretty good. If you don't already know what you're looking for, then you're more likely to be
tempted by a trap answer. On the other hand, if you know what you're looking for, then you can
confidently eliminate answer choices that don't match the answer you came up with.

5. Grade each answer choice: Pass or Fail.


It's process of elimination time. Eliminate any answer that disagrees with the answer you came
up with in your head in step 4. It's okay if you've got more than one "passing" answer.

6. Compare the remaining answer choices that "passed" in step 5.


Suppose you had two passing answer choices, X and Y. Ask yourself, how is answer choice
X different from answer choice Y? Once you've spotted a difference, you should be able to
eliminate one of the two answer choices. The last one standing is your answer.

7. Make sure you haven't chosen a Trap Answer.


Take two seconds to make sure your answer isn't one of the common trap answers on the next
page.

8. Bubble in your final answer - or guess - and move on.


If you're confident in your answer, then bubble it in on your answer sheet and move on. If you're
confused or had difficulty eliminating answer choices, then skip the problem and
come back to it later.

16
Five Wrong Answer Types
1. THE “95% RIGHT” ANSWER - THAT IS 100% WRONG:
This type of answer sounds good because it's almost entirely correct - except that a tiny
piece of the statement is definitely wrong. Often the problem occurs when the answer choice
expands the scope of the author's statements. If, for example, a passage deals exclusively with
polar bears, then a 95% Right Answer Choice might take a statement about polar bears and
apply it to woolly mammoths or large land mammals in general.

2. THE "SEEMS FAMILIAR" ANSWER CHOICE:


These answers tempt you with words or phrases taken verbatim from the passage, but the
answer itself is definitely incorrect. These answers trap lazy students who play the match
game by simply picking an answer choice that directly quotes a memorable statement in the
passage. Don’t be lazy! Never choose an answer choice just because it sounds familiar.

3. THE OBVIOUSLY TRUE STATEMENT THAT DOESN'T ANSWER THE QUESTION:


Often you’ll come across an answer choice like “Education is important to a free society.” This
statement is almost certainly true, but we're not being asked to find the true statement. We've
been asked to find the answer that directly answers the question.
A statement, true or otherwise, is only the correct answer if it answers the question.

4. THE "NOT MENTIONED IN THE PASSAGE" ANSWER CHOICE:


Don’t be tempted by an answer choice that deals with a subject that’s not mentioned in the
passage - even if that subject is related to the answer choice. If the passage is all about the
intelligence of dolphins, don’t choose an answer choice about the intelligence of sharks just
because it’s closely related. An exception to this occurs on Indirect Questions that include the
phrase, “Which of the following, if true...” That phrase tells you to take for granted that all of
the answer choices are true, and consider the implications of facts that were not part of the
original passage.

5. THE "TOO BOLD" ANSWER CHOICE:


A statement like “California always needs more rain” is usually a bad answer choice because
just one counter-example can disprove it. It’s easy to imagine a scenario in which an already
flooded California doesn't need more rain. The word “always” makes this one wrong. Watch for
words like: always, never, everyone, everybody, no one, nobody, none, impossible, cannot, , must
not, must only, all, etc. In contrast, better answer choices use words like: many, most, nearly,
some, could, can, might, often, rarely, may, few, frequently, infrequently, sometimes, someone,
somebody, possibly, etc.

SAT v17.10 Unauthorized copying or reuse of any part of this page is illegal. 17
(SAT Teacher Edition)
Applying Active Reading

It's your turn.


Use ACTIVE READING SKILLS as you read
this sample passage. Pay attention to the
PURPOSE of passage. Is it an EXPOSITORY,
NARRATIVE, or RHETORICAL passage?

Make a note of the MAIN IDEA, STRUCTURE,


and TONE of the passage as you read
through and MARK UP the passage.

18
Keep going!
Time to use what you've learned to
answer a few questions. As you're
working, remember to:

READ each question carefully.

Come up with an answer IN YOUR HEAD


before checking the answer choices.

Use PROCESS OF ELIMINATION.

Look for EVIDENCE in the passage.

ER:
REMEMB s
t e r y o u r answer ring
En
o r in s t a nt sco
online f !
d v id e o solutions
an
Unauthorized copying or reuse of any part of this page is illegal. 19
SAT v17.10
(SAT Teacher Edition)
Applying Active Reading

20
SAT v17.10 Unauthorized copying or reuse of any part of this page is illegal. 21
(SAT Teacher Edition)
ER:
REMEMB rs
ur answe
Enter yo ring
f o r in s tant sco
online !
solutions
22 and video
Reading Unit #1
Homework Drill #1

SAT v17.10 Unauthorized copying or reuse of any part of this page is illegal. 23
(SAT Teacher Edition)
24
Reading Unit #1
Homework Drill #2

SAT v17.10 Unauthorized copying or reuse of any part of this page is illegal. 25
(SAT Teacher Edition)
26
Reading Unit #1
Homework Drill #3

SAT v17.10 Unauthorized copying or reuse of any part of this page is illegal. 27
(SAT Teacher Edition)
28
SAT v17.10 Unauthorized copying or reuse of any part of this page is illegal. 29
(SAT Teacher Edition)
ER:
REMEMB s
t e r y o u r answer
En
r in s t a n t scoring
online fo !
solutions
30 and video
2 Mark Up The Passage 33

Question Types - Part 1 - FAQ 37


Reading Unit #2

Recommended Reading List 47

Homework Drills:

1 2 3
51 53 56

SAT v17.10 Unauthorized copying or reuse of any part of this page is illegal. 31
(SAT Teacher Edition)
MARK UP THE PASSAGE, BUT DON'T MAKE A MESS OF IT!

When marking up the passage, be careful not to overdo it and make a mess. Remember that underlining
everything is worse than underling nothing at all. Also, remember that an underline has the same effect
as a circle but is faster and won't get in the way of the text.

Read a full paragraph at a time before underlining, circling, or making notes in the margins. This
approach helps you to avoid overdoing it.

Step 1. Read a paragraph.


Step 2. Pause a moment to consider the paragraph's:

Direction (same direction? new direction?)


Role in the passage (background info, explain a problem, etc.)
Use of vocabulary (key terms? multiple meanings?)
Use of evidence (analogous situation? statistic?, etc. )

Step 3. Mark up the paragraph with notes that you'll find useful later.
Step 4. Jot down a few words in the margin to summarize the paragraph.
Step 5. Move on to the next paragraph and repeat the process.

This process also forces you to consider the paragraph you've just read instead of proceeding to the next
paragraph before filling your head with more information.

Until you get the hang of this method, be sure to exaggerate the pause when you consider the paragraph
in Step 2.

Your instructor may have some special techniques for marking up the passage. Use the space below to
jot down some notes.

32
Mark Up The Passage

SAT v17.10 Unauthorized copying or reuse of any part of this page is illegal. 33
(SAT Teacher Edition)
vs.
Samurai
h -->
new tec
of
adoption
h.
new tec

Learn
business from
has Samurai's
changed (+)
example

Duelist?

old
ways
of biz

Don't lose
s!
customer

You can't
afford
NOT to
New Citie
s advertise
cause
in
changes
&
business
need for
ng
advertisi
Comp:
ng
Advertisi
to biz =
ng fertilizer
Advertisi .
Proven to farmer
ul
Successf

Comp:
w/o
business
ng =
advertisi
and
hand to-h a
in
combat
range-war

tising
No adver
--> R.I.P.

34
SAT v17.10 Unauthorized copying or reuse of any part of this page is illegal. 35
(SAT Teacher Edition)
ER:
REMEMB s
t e r y o u r answer
En
r in s t a n t scoring
online fo !
solutions
36
and video
Question Types - Part 1 - FAQ

Frequently Asked Questions


On the next few pages, we'll take a look at the question types that show up most frequently
on the SAT. Getting to know these question types now will help you work more efficiently
on test day.

In this section we'll dissect each of the most frequently occurring question types, identify
the best strategies to use on each one, and share a few tips and tricks.

It's important that you keep in mind that there are NO tips or tricks that can help you
answer the questions if you haven't read and understood the passage. With that in mind,
don't think for a second that any of these specific strategies are intended to take the place
of active reading. Mark up the passage, and think critically about the author's purpose,
tone, evidence, and vocabulary.

Each page in this section covers a different question type, but the first one (page 38) is
just designed to help you understand the format of this section. Check out that example
before you begin studying the special strategies for the Frequently Asked Questions.

The Frequently Asked Questions Include:

1. Main Idea Questions

2. Word / Phrase in Context Questions

3. Detail Questions

4. Inference Questions

5. Cite the Evidence Questions

SAT v17.10 Unauthorized copying or reuse of any part of this page is illegal. 37
(SAT Teacher Edition)
A SPECIFIC QUESTION TYPE

KEY CHARACTERISTIC: Check out this box for an example of what this
type of question looks like on the test

HOW DO YOU KNOW YOU'RE A) Answer Choice A


DEALING WITH THIS TYPE OF B) Answer Choice B
C) Answer Choice C
QUESTION? D) Answer Choice D

SOME TYPES OF QUESTIONS ARE ALWAYS PHRASED THE SAME WAY.


OTHERS VARY A BIT. HERE, YOU'LL PRACTICE UNDERLINING THE
IMPORTANT PIECES OF THE TEXT.
1. This sample question is a lot like all the others, 3. There are lots of ways to ask the same question. Make
but its phrasing may be slightly different. sure you're familiar with all of them.

2. This question asks you pretty much the same 4. If you know all the ways you can be asked to find the
as the last one, but the phrasing is slightly main idea, for example, then you'll be able to work
different. more quickly on test day.
the passage is that
2.4. The central
main idea
problem
of the that
passage
Friedman
is describes in

passage?
3.1. Which
The central
of theclaim
following
of the best
passage
summarizes
is that the

EACH OF THESE QUESTION TYPES HAS A WEAKNESS. THIS IS


WHERE WE TEACH YOU HOW TO EXPLOIT THAT WEAKNESS!

1. Once you've identified a specific problem type, use these steps to attack that problem.

2. This step might say something about referring to the notes you wrote in the margins.

3. Whenever possible, use process of elimination.

4. Choose the best answer.

TIPS:
Most specific problem types follow patterns. If you're aware of those patterns, then you'll
be able to identify the trap answers that the test makers throw at you.

Other tips involve where to look for evidence in the passage, or how to eliminate wrong
answers.

38
MAIN IDEA QUESTIONS

The main purpose of the passage is to


KEY CHARACTERISTIC:
ASKS YOU TO IDENTIFY THE A) tell the story of a forgotten people
B) speculate as to the use of an ancient
AUTHOR’S THESIS OR PRIMARY structure
MOTIVATION FOR WRITING THE C) invite the reader to imagine what life may
PASSAGE have been like long ago
D) show how false beliefs can cripple
societies

UNDERLINE THE WORD OR PHRASE THAT INDICATES


YOU’RE DEALING WITH A MAIN IDEA QUESTION.

1. The central claim of the passage is that 3. Which of the following best summarizes the passage?

2. The main idea of the passage is 4. The central problem that Friedman describes in the
passage is that

SPECIAL APPROACH FOR MAIN IDEA QUESTIONS:


The Main Idea Question is one of the most common question types, so make sure you're
looking for the Main Idea and taking notes as you read through every passage. If you
can't identify the main idea off the top of your head, quickly review your notes. Your notes
should give you a good framework to start eliminating wrong answers.

The Main Idea is generally introduced early and restated at the end of the passage.

TIPS:
The Main Idea of an Expository Passage is usually a Question, such as "What are the forces that
affect voter behavior?"

The Main Idea of a Rhetorical Passage is a Thesis Statement, such as "Dogs are better than cats."

The Main idea of a Narrative Passage is the Protagonist's Struggle, such as "A young attorney
battles nerves as she prepares for her first case."

The correct answer includes all the important information without adding incorrect or
unimportant information. The correct answer will relate to the entire passage.

Watch for trap answers that state the main idea of a single paragraph or supporting argument.

SAT v17.10 Unauthorized copying or reuse of any part of this page is illegal. 39
(SAT Teacher Edition)
WORD/PHRASE IN CONTEXT QUESTIONS

KEY CHARACTERISTIC: As used in line 51, "transcend" most


nearly means:
TESTS YOUR ABILITY TO A) escape
DETERMINE THE MEANING OF B) surmount
A WORD OR PHRASE BY HOW C) extend
D) overcome
IT IS USED IN CONTEXT.

UNDERLINE THE WORD OR PHRASE THAT INDICATES


YOU’RE DEALING WITH A MEANING OF WORDS QUESTION.
1. As it is used in line 66, "sharp" most nearly 3. As used in line 101, "prolific" most nearly
means means

2. As used in line 16, "blind-sided" most nearly 4. As used in lines 19-20, "bowled over" most
means nearly means

SPECIAL APPROACH FOR MEANING OF WORDS QUESTIONS:


1. Read the line in the passage that contains the word or phrase you’ve been asked about
in the question. You may need to read a few lines before and after the line for context.

2. Underline (or cross out) the word or phrase you've been asked about in the question.

3. Look for clues in the sentence that tell you the missing word’s attributes.

4. Use those attributes to come up with an educated guess as to what the word might be.

5. Eliminate all answer choices that are inconsistent with your educated guess.

6. Choose the best remaining answer.

TIPS:
The correct answer to a Meaning of Words Question is almost never the most common
definition of the word. You’re not being asked how the word is generally used.

You’re being asked how the word is used in a specific sentence of the passage, so strongly
consider eliminating the most common literal definition. For example, if a shirt is described
as “loud” in the passage, the correct answer is almost certainly not “noisy.” It’s much more
likely that the answer is “tacky.”

40
DETAIL QUESTIONS

According to the passage, Chandler was most


KEY CHARACTERISTIC: dominant in which type of event?
ASKS YOU TO PARAPHRASE A
A) throwing events
STATEMENT, AN ARGUMENT,
B) endurance events
OR OTHER INFORMATION THAT C) relay events
IS DIRECTLY STATED BY THE D) javelin
AUTHOR WITHIN THE PASSAGE

UNDERLINE THE WORD OR PHRASE THAT INDICATES


YOU’RE DEALING WITH A DETAIL QUESTION.
1. Jefferson claims that which of the following is a 3. According to Rodgers, the presence of the clouds
widely misunderstood phenomenon? near the summit is significant because

2. The narrator indicates that Rusty, Ted, and Larry 4. According to the author, town hall meetings are
are best described as

SPECIAL APPROACH FOR DETAIL QUESTIONS:


1. Remember that Detail Questions ask you to choose the answer that most accurately
restates information from the passage. There is no need to make an inference or read
between the lines on a Paraphrase Question. Research your answer by rereading the part
of the passage that the question directs you to consider.

2. Eliminate incorrect answer choices using Process of Elimination.

3. Choose the best remaining answer choice.

TIPS:
Beware of answer choices that directly quote a word or phrase from the passage - they’re
usually trap answers!

Also watch out for answer choices that rely on statements or opinions made by people other
than the author - like a critic or an expert.

Keep in mind that the correct answers to Detail Questions often use abstract phrasing to
confuse you. For example, the detail in the passage may deal with football helmets, but the
correct answer may reference "safety equipment." Another question may deal with a prohibition
on jet skis, but the correct answer may reference "pending legislation." Be careful!

SAT v17.10 Unauthorized copying or reuse of any part of this page is illegal. 41
(SAT Teacher Edition)
INFERENCE QUESTIONS

What can be reasonably inferred about Eddie's


KEY CHARACTERISTIC: plans for next summer?
ASKS YOU TO DRAW A
CONCLUSION THAT IS NOT A) He intends to return to Camp Anawanna
as a counselor.
STATED IN THE PASSAGE BUT B) He hopes to attend a different camp
IS LOGICALLY SUPPORTED BY as a counselor.
INFORMATION IN THE PASSAGE C) He will be too old to attend Camp
Anawanna as a camper.
D) He is pleased that he will no longer
have to attend summer camp.

UNDERLINE THE WORD OR PHRASE THAT INDICATES


YOU’RE DEALING WITH AN INFERENCE QUESTION.
1. It can be inferred that the author of Passage 1 3. How would the author of Passage 2 most likely
believes that politicians are respond to the claim made in the final paragraph
of Passage 1?
2. The passage most strongly suggests that the
demonstration is likely to encounter which 4. The authors of both passages would most likely agree
response? with which statement about cheese products?
response?
demonstration
agree with which
is likely
statement
to encounter
about cheese
whichproducts?
2. The authors
4. passage of
most
bothstrongly
passages
suggests
would that
mostthe
likely

believes
to the claim
thatmade
politicians
in the are
final paragraph of Passage 1?
1. It
3. Howcanwould
be inferred
the author
that the
of Passage
author of2Passage
most likely
1 respond

SPECIAL APPROACH FOR INFERENCE QUESTIONS:


The defining characteristic of an Inference Question is that its answer is NOT directly
stated in the passage. Instead, the answer to an Inference Question is something that can
reasonably be concluded based on the evidence in the passage. The following words and
phrases tell you that you’ll need to make an inference:

suggests inferred most likely


probably would likely would respond to

TIPS:
Before you attempt to answer an inference, ask yourself, "Is it necessary to make an
inference on this question?” The goal is to avoid making inferences when they're not
necessary (i.e. choosing a bunch of incorrect answers).

Once you're certain that you're dealing with an Inference Question, remember that the
correct answer answer won’t make a huge logical leap, but it will build significantly on one
or more facts that are directly stated in the passage. The correct answer will never get off
track or introduce information that is inconsistent with the author’s main idea.

One more thing: Your feelings don't matter! Don’t ever base your answer to an Inference
Question on your own opinion or outside information.

42
CITE THE TEXT QUESTIONS

KEY CHARACTERISTIC: Which choice provides the best evidence for the
ASKS YOU TO FIND A SPECIFIC answer to the previous question?
LINE OF TEXT IN THE PASSAGE A) Lines 14-18 ("The debate . . . credibility")
THAT DIRECTLY SUPPORTS B) Lines 39-41 ("Allegedly . . . innocent")
THE ANSWER TO THE PREVIOUS C) Lines 57-58 ("In 2013 . . . subpoena")
D) Lines 14-18 ("What . . . make")
QUESTION -- OR A CLAIM
STATED IN THE QUESTION.

UNDERLINE THE WORD OR PHRASE THAT INDICATES


YOU’RE DEALING WITH A CITE THE TEXT QUESTION.
1. Which choice provides the best evidence for the 3. Which choice provides the best evidence that the
answer to the previous question? narrator did not understand the gravity of his error?

2. Which choice best supports the conclusion that 4. Which choice provides the best evidence that the
the Asian Carp is an invasive species? author of Passage 1 would agree to some extent with
the quote in the final paragraph of Passage 2?

SPECIAL APPROACH FOR CITE THE TEXT QUESTIONS:


1. Reread the previous question and your answer before you look at the answer choices for
the new question, focusing on the relationship between the text references in the Cite the
Text Question and the answer you chose on the previous question.
2. Make sure you know what you're looking for. For example, if you're asked to cite evidence
in support of the answer to a Detail or Inference Question, don't choose the evidence that
broadly supports the Main Idea. The evidence you cite must be clear and direct.
3. Eliminate any answer choice that does not provide direct support to the answer you chose
for the previous problem.
4. Choose the best answer and move on.

TIPS:
It's a good idea to keep an eye out for upcoming Cite the Text Questions, so you can attack
them at the same time that you attack the questions that precede them. That way you can
use the line references in the Cite the Text Question to direct you to the evidence in support
of the correct answer. Pretty clever, huh?

If you want to get really clever, you can answer the Cite the Text Question first. Try drawing
lines to connect each of its answer choices to the corresponding statements in the previous
question that you're looking for evidence to support. This can help you keep things clear.

Then, as you begin using process of elimination, you'll have TWO chances to eliminate each
answer choice: first, the answer fails to address the question, and second, if it lacks quality
support in the Cite the Text Question.

SAT v17.10 Unauthorized copying or reuse of any part of this page is illegal. 43
(SAT Teacher Edition)
Question Types - Part 1 - FAQ

44
SAT v17.10 Unauthorized copying or reuse of any part of this page is illegal. 45
(SAT Teacher Edition)
46
Recommended Reading List

Recommended Reading List


First of all, we recommend reading. Period. Find something you enjoy reading.
Read it. Find something else you enjoy reading. Read it. Then repeat the process.

If you want to be more specific, we recommend reading texts that are similar to
those on the SAT. Makes sense, right?

With that in mind, you'll want to read texts that fall into one of these three
categories:

Challenging Magazine and Newspaper Articles

100 Books You Should Have Read in School (or a few of them at least)

United States' Founding/Formative Documents

Challenging Magazines and Newspapers


Magazines
The New York Times Magazine Discover
The New Yorker Mental Floss
Newspapers
The Atlantic Popular Science
The Wall Street Journal
The Economist Science News
The New York Times
Time Magazine Wired
USA Today
Scientific American Skeptic
The Washington Post
New Scientist Smithsonian
ID: Ideas & Discoveries Popular Mechanics
National Geographic

SAT v17.10 Unauthorized copying or reuse of any part of this page is illegal. 47
(SAT Teacher Edition)
100 Books You Should Have Read in School (or a few of them at least)
To Kill a Mockingbird - Harper Lee Their Eyes Were Watching God - Zora Neale Hurston
Romeo and Juliet - William Shakespeare Things Fall Apart - Chinua Achebe
The Great Gatsby - F. Scott Fitzgerald Twelve Angry Men - Reginald Rose
Lord of the Flies - William Golding Ender's Game - Orson Scott Card
Macbeth - William Shakespeare Inferno - Dante Alighieri
Catcher in the Rye - J.D. Salinger A Raisin in the Sun - Lorraine Hansberry
Animal Farm - George Orwell Beloved - Toni Morrison
Atlas Shrugged - Ayn Rand The Things They Carried - Tim O'Brien
Of Mice and Men - John Steinbeck Much Ado About Nothing - William Shakespeare
The Scarlet Letter - Nathaniel Hawthorne Oedipus Rex - Sophocles
Hamlet - William Shakespeare Heart of Darkness - Joseph Conrad
The Adventures of Huckleberry Finn - Mark Twain The Fountainhead - Ayn Rand
The Odyssey - Homer The Adventures of Tom Sawyer - Mark Twain
1984 - George Orwell All Quiet on the Western Front - Erich Maria Remarque
The Diary of a Young Girl - Anne Frank The Chosen - Chaim Potok
Pride and Prejudice - Jane Austen The Hound of the Baskervilles - Arthur Conan Doyle
The Giver - Lois Lowry One Flew over the Cuckoo's Nest - Ken Kesey
Julius Caesar - William Shakespeare Oliver Twist - Charles Dickens
Night - Elie Wiesel Siddhartha - Hermann Hesse
Jane Eyre - Charlotte Brontë Grendel - John Gardner
Brave New World - Aldous Huxley The Glass Menagerie - Tennessee Williams
The Crucible - Arthur Miller As I Lay Dying - William Faulkner
Fahrenheit 451 - Ray Bradbury The Tempest - William Shakespeare
Great Expectations - Charles Dickens 20,000 Leagues Under the Sea - Jules Verne
The Grapes of Wrath - John Steinbeck A Modest Proposal - Jonathan Swift
The Outsiders - S.E. Hinton Gulliver's Travels - Jonathan Swift
Frankenstein - Mary Shelley Les Misérables - Victor Hugo
A Midsummer Night's Dream - William Shakespeare The Lord of the Rings - J.R.R. Tolkien
Wuthering Heights - Emily Brontë Our Town - Thornton Wilder
A Tale of Two Cities - Charles Dickens The Metamorphosis - Franz Kafka
Beowulf - Unknown A Doll's House - Henrik Ibsen
Othello - William Shakespeare Steppenwolf - Hermann Hesse
The Canterbury Tales - Geoffrey Chaucer The Ecological Rift - John Bellamy Foster
Flowers for Algernon - Daniel Keyes Robinson Crusoe - Daniel Defoe
The Little Prince - Antoine de Saint-Exupéry Pygmalion - George Bernard Shaw
The Hobbit - J.R.R. Tolkien Ethan Frome - Edith Wharton
The Old Man and the Sea - Ernest Hemingway The Jungle - Upton Sinclair
Crime and Punishment - Fyodor Dostoyevsky The Stranger - Albert Camus
Taming of the Shrew - William Shakespeare The Handmaid's Tale - Margaret Atwood
A Separate Peace - John Knowles A Clockwork Orange - Anthony Burgess
Death of a Salesman - Arthur Miller Rebecca - Daphne du Maurier
Slaughterhouse-Five - Kurt Vonnegut A Tree Grows in Brooklyn - Betty Smith
The Red Badge of Courage - Stephen Crane The Lottery and Other Stories - Shirley Jackson
I Know Why the Caged Bird Sings - Maya Angelou Invisible Man - Ralph Ellison
The Awakening - Kate Chopin Catch-22 - Joseph Heller
The Good Earth - Pearl S. Buck The Pigman - Paul Zindel
The Pearl - John Steinbeck Madame Bovary - Gustave Flaubert
Moby Dick - Herman Melville Little Women - Louisa May Alcott
The Importance of Being Earnest - Oscar Wilde Walden - Henry David Thoreau
King Lear - William Shakespeare The Sound and the Fury - William Faulkner

48
Founding Documents & The Global Conversation

Founding Documents
The Anti-Federalist Papers
The Articles of Confederation
The Bill of Rights
The Constitution of the United States of America
Debates in the Several Conventions on the Adoption of the Federal Constitution - Jonathan Elliot
Debates in the Federal Convention of 1787 - James Madison
Farewell Address - George Washington
The Federalist Papers - Alexander Hamilton, James Madison, and John Jay
The United States Declaration of Independence

Global Conversation
The Declaration of the Rights of Man and the Citizen - National Constituent Assembly of France
Society and Solitude - Ralph Waldo Emerson
Hind Swaraj or Indian Home Rule - Mohandas Gandhi
The Gettysburg Address - Abraham Lincoln
Letter from Birmingham Jail - Martin Luther King Jr.
On Liberty - John Stewart Mill
Walden - Henry David Thoreau
Democracy in America - Alexis de Tocqueville
A Quilt of a Country - Anna Quindlen
Four Essays on Liberty - Isaiah Berlin
Reflections on the Revolution in France - Edmund Burke
Leviathan - Thomas Hobbes
Second Treatise of Civil Government - John Locke
The Spirit of the Laws - Montesquieu
Common Sense - Thomas Paine
A Vindication of the Rights of Woman - Mary Wollstonecraft
The "Spirit of Liberty" Speech - Judge Learned Hand

Recommended Compilations
Great Speeches of the 20th Century - Bob Blaisdell (editor)
Lend Me Your Ears - Great Speeches in History - William Safire (editor)
The American Intellectual Tradition: Volume I: 1630-1865 - David Hollinger & Charles Capper
The American Intellectual Tradition, Vol. II: 1865 to the Present - David Hollinger & Charles Capper

SAT v17.10 Unauthorized copying or reuse of any part of this page is illegal. 49
(SAT Teacher Edition)
50
Reading Unit #2
Homework Drill #1

SAT v17.10 Unauthorized copying or reuse of any part of this page is illegal. 51
(SAT Teacher Edition)
52
Reading Unit #2
Homework Drill #2

SAT v17.10 Unauthorized copying or reuse of any part of this page is illegal. 53
(SAT Teacher Edition)
54
SAT v17.10 Unauthorized copying or reuse of any part of this page is illegal. 55
(SAT Teacher Edition)
Reading Unit #2
Homework Drill #3

56
SAT v17.10 Unauthorized copying or reuse of any part of this page is illegal. 57
(SAT Teacher Edition)
ER:
REMEMB nswers o
nline
y o u r a
Enter
s c o r in g and video
nt
for insta
!
solutions
58
3 Infographics 60

Paired Passages 71
Reading Unit #3

Homework Drills:

1 2 3
82 84 88

SAT v17.10 Unauthorized copying or reuse of any part of this page is illegal. 59
(SAT Teacher Edition)
Infographics

60
SAT v17.10 Unauthorized copying or reuse of any part of this page is illegal. 61
(SAT Teacher Edition)
ER:
REMEMB rs
n t e r y o ur answe g
E
o r in s t a nt scorin
online f !
d v id e o solutions
62 a n
the
e among
Looks lik er
the answ
groups in h e s t
the hig
choices, ped
ge of tip
percenta rs
is worke
workers h
than a hig
with less
loma.
school dip

g
er ba
$1.00 p tops
zil
in Bra s
s t of tip
the li 10.
q u e stion #
in

ently,
Appar
isn't
tipping
in
a thing
ore.
Singap

Infographics
There was a time when the SAT focused on a bunch of isolated skill sets. Students sat down
and answered a bunch of bite-sized questions that had nothing to do with one another. If
you missed one question, then it wasn't that big a deal because the next one was a whole
new ballgame. Well, those days are gone.

The new SAT is big on combining skill sets. Put simply, that means you should expect to see
a couple informational graphics (infographics) on the verbal sections of the SAT. Typically,
you'll encounter two infographics on the Reading Section and two more on the Writing
Section.

These infographics show up in the form tables, charts, and graphs that are referenced
by the passage. Make sure you understand the information that's represented in the
infographic and how it relates to the information in the Reading Passage. Then, you'll need
to answer a few questions about the graphic. Often you'll be asked which statement in the
answer choices is supported by the data in the infographic(s).

SAT v17.10 Unauthorized copying or reuse of any part of this page is illegal. 63
(SAT Teacher Edition)
Graphs show relationships
• Each type of graph is used to highlight relationships
within the data being presented.
• Become familiar with the different graph types, and their
variations, so that you are not surprised on the test.

D1D2

Line Graphs
D1
D2
Data Points
150
Calcium Content (mg)

that the
Line Graphips bimeptlyween variables
relationsh ONTINUOUSLY, even 100
changes C ata points.
between d
50

0
7 14 21 28
y-axis Day of cycle x-axis nt
Dependent Independe
Variable Variable

INDEPENDENT VARIABLE (N):


A variable whose value does NOT depend on
the value of other variables in the graph. Usually
plotted on the x-axis.

DEPENDENT VARIABLE (N):


A variable whose value DOES depend on the value
of other variables in the graph. Usually plotted on
the y-axis.

64
Scale and Axes

104 104
log
103 103
log
Salm
102 102 Salmon

meq/L
meq/L

101 log 101


linear
100 100
10-1 10-1
0.1 1 10 100 0 50 100 150 200
Salinity (ppt TDS) Salinity (ppt TDS)

This is the
SAME DATA
represented onT
TWO DIFFEREN
SCALES!

Use keys actively!


• Read the keys and the axis labels.
• Transfer the information in the key to the graph itself.

Look for non-standard scales


• Is either axis split? On which side do your data fall?
• Recognize log-based (a.k.a. exponential) scales.
• Look at the spacing of tick marks to tell if the scale is
log or linear.

SAT v17.10 Unauthorized copying or reuse of any part of this page is illegal. 65
(SAT Teacher Edition)
Scatter
E Plots E

QUANTITATIVE (ADJ):
A correlation that can be described numerically. A
4 4 104
show
teaspoon and a swimming pool hold quantitatively
1010 used to
t s a r e different volumes of water.
Scatter
P lo int is
o r r e la
3 t e
3 . Each po 3
c 1010
how data
e m ent. QUALITATIVE10
(ADJ):
s u r
one mea 2 2 A correlation that is 2best described non-numerically.
1010 ates 10
oints indic
meq/L

e r n o f p A pool with the water slide is qualitatively different


The pat t e
1qu1alitativ
than the lap pool.
a t iv e an10d 10 101
quan t it
ions
correlat 0 0
1010 100
F -1Magnesium F -1Magnesium
1010-1 10
10 1000100 0.10.1 11 1010 100
1000 100 0.1 1 10

800 800
600 600 Best Fit/Trendline

400 400
Chloride

Chloride

200 200
0 0
0 50 100 150 0 50 100 150
lation is lation
T he corre N # 2 : T he corre .
CONCLU
S IO N # 1:
at higher CONCLU
SIO e and linear
m o r e variable to be p o s it iv
quantitat
ively appears
X values.
Magnesium
Magnesium Magnesium
1000 1000
Magnesium
800 800
600 600
400 400
200 200
0 0
0 50 100 150 0 50 100 150
relation relation
L U S IO N : The cor L U S IO N : The cor
CONC s to be CONC s to be
w n h e r e appear w n h e r e appear
sh o ntial. sh o .
ly expone and linear
66 qualitative negative
Graph Variations
Prepare to see some weird graphs.
• Many graphs present multiple datasets in one plot.
• Only rarely on the SAT will you find graphs with dual
y-axes. Learn how to catch them, and how to read them!

help y be
Grid lines Keys ma
ata
you estim
ate absent; d
beled
values. may be la
lot.
within a p
100 100
n a
Chlorophyll B g m u lt ip le data o
80 80 Plottin ou to
g le g r a ph help y
sin
% Absorption

s.
Chlorophyll A mparison
60 60 make co
can
mparison
40 40 What co o m this grap
h?
k e f r
you ma
20 20

0 0
400 500 600 700
Wavelength (nm)

100 100

80 80
Carotene
% Absorption

150 600
Y-AXES!
60 60
D U A L
OUT FOR is C
WATCH ight y-axis
mg/ml D1 and D2

e , t h e r 40 D1
abov 40 help .
In graph ju s t t here to Xanthophyll D2
; it 's 100 400
harmless 20
mg/ml C

right 20
t o t h e right, the
ap h the
In the gr lu e s o f C, while
ts va0 0
y-axis plo 400 t h e er values600
oth500
50
700
200
a x is p lo t s
left Wavelength (nm)
0 0
Figure 1
7 14 21 28
Day of Cycle

SAT v17.10 Unauthorized copying or reuse of any part of this page is illegal. 67
(SAT Teacher Edition)
Infographics

68
SAT v17.10 Unauthorized copying or reuse of any part of this page is illegal. 69
(SAT Teacher Edition)
ER:
REMEMB s
t e r y o u r answer
En
r in s t a n t scoring
online fo !
solutions
70 and video
Paired Passages

Passage 2

SAT v17.10 Unauthorized copying or reuse of any part of this page is illegal. 71
(SAT Teacher Edition)
72
ER:
REMEMB rs
ur answe
Enter yo coring
e f o r instant s
onlin !
solutions
and video
SAT v17.10 Unauthorized copying or reuse of any part of this page is illegal. 73
(SAT Teacher Edition)
It's your turn.
On the next page you'll find a pair of short
practice passages. You shouldn't expect
to see any passages this short on test day, Passage 1
but this is a good opportunity to practice
working with paired passages.

As you read the passages, be sure to use


the ACTIVE READING SKILLS you've learned
in previous units.

Pay attention to the PURPOSE of each


passage. Is it an EXPOSITORY, NARRATIVE,
or RHETORICAL passage?

Make a note of the MAIN IDEA, STRUCTURE,


and TONE of the passage as you read Passage 2
through and MARK UP the passage.

Finally, be sure to notice the relationship


between the passages.

What is the SHARED TOPIC of the two


passages?

How do the authors' opinions on that topic


compare?

On which points do they AGREE. How do


they DISAGREE?

How does each author use EVIDENCE to


support his or her position?

74
Passage 1 ated
Refriger ns -->
rai
Cargo T
roduce
better p
S
across U

s a
CA grow
t o f p r oduce
lo
> G et s first
--
the
pick of
o d st uf f.
go

Passage 2
Quotes an
expert.

NOT get
CA does
b es t produce.
the

duce
Best pro
ped so
gets ship
in good
it arrives
cond it io n.

SAT v17.10 Unauthorized copying or reuse of any part of this page is illegal. 75
(SAT Teacher Edition)
76
Let's try one more.
Again, this is just a practice passage with a
few questions designed to get you thinking
about the relationship between the
passages.
Passage 1
Remember to be careful when you're
dealing with MORE THAN ONE point of
view.

Passage 2

SAT v17.10 Unauthorized copying or reuse of any part of this page is illegal. 77
(SAT Teacher Edition)
78
Paired Passages

SAT v17.10 Unauthorized copying or reuse of any part of this page is illegal. 79
(SAT Teacher Edition)
80
SAT v17.10 Unauthorized copying or reuse of any part of this page is illegal. 81
(SAT Teacher Edition)
Reading Unit #3
Homework Drill #1

82
SAT v17.10 Unauthorized copying or reuse of any part of this page is illegal. 83
(SAT Teacher Edition)
Reading Unit #3
Homework Drill #2

84
SAT v17.10 Unauthorized copying or reuse of any part of this page is illegal. 85
(SAT Teacher Edition)
86
SAT v17.10 Unauthorized copying or reuse of any part of this page is illegal. 87
(SAT Teacher Edition)
Reading Unit #3
Homework Drill #3

88
SAT v17.10 Unauthorized copying or reuse of any part of this page is illegal. 89
(SAT Teacher Edition)
90
4 Strategy Review 93

More Question Types 94


Reading Unit #4

Vocabulary: Tone / Attitude 105

Vocabulary: Other Essential Words 109

Vocabulary: Word Roots 113

SAT v17.10 Unauthorized copying or reuse of any part of this page is illegal. 91
(SAT Teacher Edition)
92
Strategy Review

The General Strategy Review


1. Actively read the passage first.
2. Read the question slowly, carefully, and twice if necessary!
3. Review the relevant portion of the passage.
4. * * * ANSWER THE QUESTION IN YOUR HEAD! * * *
5. Grade each answer choice: Pass or Fail.
6. Compare the remaining answer choices that "passed" in step 5.
7. Make sure you haven't chosen a Trap Answer.
8. Bubble in your final answer - or take a guess - and move on.

Make Sure You've Prepared for:


Process of Elimination (Common Wrong Answer Types)
The Frequently Asked Question Types
Infographics
Paired Passages
The Less-Frequently Asked Question Types

Spend some time with the Vocabulary Resources


Reading Vocabulary: Tone / Attitude Words (105)
Reading Vocabulary: Other Essential Words (109)
Reading Vocabulary: Word Roots (113)
Vocabulary Builder - Synonym Clusters (507)

Read as Much as Possible From the Recommended Reading Lists


Challenging Magazines and Newspapers (47)
100 Books You Should Have Read in School (48)
Founding Documents & The Global Conversation (49)

SAT v17.10 Unauthorized copying or reuse of any part of this page is illegal. 93
(SAT Teacher Edition)
More Question Types

94
SAT v17.10 Unauthorized copying or reuse of any part of this page is illegal. 95
(SAT Teacher Edition)
96
More Question Types
On the next few pages, we'll take a look at a few of the question types that show
up less frequently on the SAT. Even though they don't show up as often as the
Frequently Asked Questions that we covered earlier, they're still worth reviewing
before test day. Just as with the FAQs, getting to know these question types now
will help you work more efficiently on test day.

This section follows the same format as last time: dissect each question type,
identify the best strategies to use, and share a few tips and tricks.

It's worth repeating that tips and tricks can only help you answer the questions if
you have already read and understood the passage. Don't think for a second that
any of these specific strategies are intended to take the place of actively reading,
marking up the passage, and thinking critically about the author's purpose, tone,
use of evidence, and use of vocabulary.

The Less-Frequently Asked Questions Include:

1. Author's Voice or Method Questions

2. Tone / Attitude Questions

3. Summary Questions

4. Assumption Questions

SAT v17.10 Unauthorized copying or reuse of any part of this page is illegal. 97
(SAT Teacher Edition)
AUTHOR’S VOICE OR METHOD QUESTIONS

The author mentions Ambrose Bierce in line 63


KEY CHARACTERISTIC: primarily in order to
ASKS YOU TO DETERMINE THE
A) give an example of a superb humorist.
ROLE OF A WORD, PHRASE, B) give the impression that he is well-read.
SENTENCE, PARAGRAPH, C) caution the reader against imitating great
OR LITERARY DEVICE AS IT authors.
D) recommend that young authors read his
RELATES TO THE REST OF work.
THE PASSAGE.

UNDERLINE THE WORD OR PHRASE THAT INDICATES


YOU’RE DEALING WITH AN AUTHOR’S VOICE OR METHOD QUESTION.
1. In lines 61-64, the author of Passage 2 refers to a 3. The author refers to work by Dr. Fraiser and others
statement made in Passage 1 in order to (line 77) in order to

2. The author most likely uses the examples in lines


51-59 ("Alarmingly . . . noisemaker") to highlight 4. The author's main purpose of including the
the information about the espionage is to

SPECIAL APPROACH FOR AUTHOR’S VOICE OR METHOD QUESTIONS:


1. Read the text surrounding the lines you’re asked about in the question.

2. Typically, these questions focus on how a statement or a piece of evidence relates to the
author's thesis. Most commonly, the answer is one of the following:

provide an example quote an expert acknowledge a counterpoint


draw a distinction cite a statistic answer a critic/criticism
pose a question ask a rhetorical question introduce a key term
appeal to logic draw a comparison expand the scope of the argument

3. Eliminate any answer choice that contradicts or undermines the author’s main argument.

4. Choose the best remaining answer.

TIPS:
Carefully consider the scope of the question. It is usually necessary to consider the
statement in the question as it relates to the Main Idea of the passage. With that in mind,
you can see that the line reference does not necessarily guide you to the answer. You may
need to consider other parts of the passage — or all of it — in order to find the answer.

98
SUMMARY QUESTIONS
KEY CHARACTERISTIC: Which choice best summarizes the events at the
ASKS YOU TO SUMMARIZE THE picnic?
A) Many of the cars in the parking lot had
EVENTS OF A PARAGRAPH been robbed.
OR THE ENTIRE NARRATIVE B) A canoe capsized and everyone rushed
PASSAGE. MAY ALSO ASK YOU to lend assistance.
C) The picnic was ended abruptly due to
ABOUT THE ORDER OF EVENTS inclement weather.
IN THE PASSAGE. D) Nobody was able to start the barbecue.

UNDERLINE THE WORD OR PHRASE THAT INDICATES


YOU’RE DEALING WITH A SUMMARY QUESTION.

1. Which choice best summarizes the passage? 3. The author of the passage would probably respond
to the quote in the fifth paragraph (line 66-72) by
pointing

2. Which choice best reflects the overall sequence 4. Which of the following statements about "celebrity
of events in the passage? culture" (line 29) would the author most likely
support?

SPECIAL APPROACH FOR SUMMARY QUESTIONS:


This type of question typically occurs on Narrative Passages.

If you remember to use your Active Reading Skills and take good notes as you Mark Up the
passage, then you shouldn't have any trouble with Summary Questions.

If you can't eliminate answer choices based on your notes, then begin by rereading the
paragraph in question.

After reading, use process of elimination and choose the best remaining answer.

TIPS:
Remember that Summary Questions ask you to list the events that took place in the passage.

Many of the wrong answers tend to be tricky trap answers that bundle unnecessary or
incorrect information into the answer choice.

As a result, it's very important to use Process of Elimination in order to get rid of the bad
answers before you attempt to choose the best answer.

SAT v17.10 Unauthorized copying or reuse of any part of this page is illegal. 99
(SAT Teacher Edition)
ASSUMPTION QUESTIONS

The author makes which of the following


assumptions about “self-interested individuals”
KEY CHARACTERISTIC: (line 14)?
ASKS YOU TO IDENTIFY AN
ASSUMPTION THAT THE A) Acting in one’s self-interest is always bad.
B) A person always acts in his or her self
AUTHOR OF THE PASSAGE interest.
MAKES C) A person can be compelled to act outside his
or her self-interest.
D) People are generally unaware of their true
self-interest.

UNDERLINE THE WORD OR PHRASE THAT INDICATES


YOU’RE DEALING WITH AN ASSUMPTION QUESTION.
1. Which of the following is an unstated assumption 3. An unstated assumption made by the authors is that
made by the author of the passage? the writers of 1950s television were

2. The passage indicates that the assumption made by 4. The author’s claim that Devon is a “fraud” indicates
the selection committee in lines 78-84 may be that he has taken which of the following for granted?

SPECIAL APPROACH FOR ASSUMPTION QUESTIONS:


Assumption Questions are tricky. They ask you to figure out which of the statements
(answer choices) the author takes for granted. Usually, you need to consider the answer
choices in terms of how they relate to the author’s entire argument. So, in order to answer
an Assumption Question correctly, you must be pretty good at identifying the author’s
argument and understanding how all of its pieces fit together.

If you find this difficult, consider saving Assumption Questions for last.

Eliminate all answer choices that are not essential to supporting the author’s argument.

TIPS:
It makes no difference whether the assumption in the answer choice is true or false from
your point of view.

Most students who struggle on Assumption Questions do so because they choose the
answer that sounds most likely to be true. Remember, you’re not looking for the answer
that’s most likely to be true. You’re looking for the author’s unstated assumption.

100
TONE / ATTITUDE QUESTIONS

KEY CHARACTERISTIC: Dr. Cho's attitude toward the proposed


ASKS YOU TO IDENTIFY HOW legislation is best described as
AN AUTHOR, CHARACTER, OR
A) optimistic
GROUP FEELS ABOUT SOMEONE B) disinterested
OR SOMETHING DESCRIBED IN C) disapproving
D) disingenuous
THE PASSAGE.

UNDERLINE THE WORD OR PHRASE THAT INDICATES


YOU’RE DEALING WITH A TONE / ATTITUDE QUESTION.

1. The references to "light" and "warmth" at the end 3. Over the course of the passage, the narrator's
of the final paragraph mainly have which effect? attitude shifts from

2. Carl's mood after learning the diagnosis is best 4. In the passage, Stella addresses Artie with
described as

SPECIAL APPROACH FOR TONE / ATTITUDE QUESTIONS:


Tone / Attitude Questions are fairly common, especially on Narrative Passages, so prepare
yourself by paying attention to the author’s attitude as you read the passage.

The best evidence of the author’s attitude tends to be adjectives and adverbs because
these are the words that the author chose to describe his subject. It’s a good idea to
underline expressive adjectives that tell you how the author feels about his subject.

TIPS:
Remember that an author may have positive feelings toward something in the passage despite
an overall negative tone. Make sure you answer the question that you’re asked.

Tone / Attitude Questions are all about distinguishing opinion from fact.

Try this exercise. Go find a movie review. Ideally, pick one that’s written by a movie critic with
whom you’re pretty sure you agree. Read the review and focus on the facts about the movie -
like how long it is, who’s in it, the basic plot, etc. As you read, pay close attention to the author’s
use of adjectives and adverbs because that’s where his or her opinion will sneak into the movie
review. Perhaps the reviewer dislikes the script, calling the dialogue “unconvincing.” Perhaps
the reviewer felt the action sequences were over the top, so he refers to them as “gratuitous.”
These aren’t facts. These are opinions. Practice spotting the author’s opinion.

Also, take some time to get familiar with the list of Tone / Attitude Words on page 105.

SAT v17.10 Unauthorized copying or reuse of any part of this page is illegal. 101
(SAT Teacher Edition)
More Question Types

102
SAT v17.10 Unauthorized copying or reuse of any part of this page is illegal. 103
(SAT Teacher Edition)
104
Reading Vocabulary: Tone / Attitude

Tone / Attitude Words


accusatory (adj) blaming other cheerful (adj) happy direct (adj) straightforward;
people straight to the point
concerned (adj) worried or
affectionate (adj) warm and interested disappointed (adj) discouraged
caring
condescending (adj) looking disdainful (adj) scornful
ambivalent (adj) having down on other people
conflicting feelings disgusted (adj) revulsion;
confrontational (adj) a hostile, distaste; intense dislike
amused (adj) entertained argumentative meeting
dismissive (adj) feeling or
analytical (adj) logical; well- contemplative (adj) studying; showing that something is
reasoned; carefully examining a thoughtful; reflective unworthy of consideration
subject
contemptuous (adj) dissatisfied (adj) not satisfied
angry (adj) mad disrespectful; scornful
dramatic (adj) theatrical
anxious (adj) worried; excited courteous (adj) nice; respectful
earnest (adj) sincere; honest
apathetic (adj) indifferent; critical (adj) finding fault
doesn’t care; not interested eccentric (adj) odd; strange;
crude (adj) not well defined; different
apologetic (adj) sorry; wishing rude or inappropriate
to make amends egotistical (adj) thinking only
cruel (adj) mean; hurtful about oneself; self-centered
appreciative (adj) grateful;
thankful curious (adj) wanting to find out eloquent (adj) fluent and
more persuasive
arrogant (adj) too proud
cynical (adj) believing that encouraging (adj) optimistic
awed (adj) amazed; astonished people are motivated by self-
interest enthusiastic (adj) excited;
amazement (n) great surprise or energetic
wonder defensive (adj) defending
envious (adj) jealous
beguiling (adj) deceptive defiant (adj) going against
authority equivocal (adj) ambiguous;
bewilderment (adj) confusion open to more than one
demonstrative (adj) tending to interpretation; not taking a
biased (adj) partial; one-sided; show feelings openly strong position
unfair
depressed (adj) generally excited (adj) anxious
bitter (adj) angry or hateful unhappy; despondent
expectant (adj) awaiting
bored (adj) not interested; desperate (adj) frantic; without something
unexcited many options
explicit (adj) vivid and detailed;
callous (adj) insensitive determined (adj) not giving up describing something in graphic
fashion
calm (adj) relaxed; not excited didactic (adj) instructive;
intending to convey a lesson fanciful (adj) imaginative
candid (adj) straightforward;
honest; blunt diplomatic (adj) trying not to fearful (adj) afraid
offend

SAT v17.10 Unauthorized copying or reuse of any part of this page is illegal. 105
(SAT Teacher Edition)
formal (adj) appropriate intimate (adj) very familiar; the patient (adj) willing to wait; not
behavior way one talks to close friends in a hurry
forthright (adj) direct and ironic (adj) different than what is patronizing (adj) looking down
honest without hesitation expected or the opposite of what on others; acting superior
is meant
frank (adj) honest and direct pensive (adj) deep in thought
irritated (adj) annoyed
friendly (adj) courteous; polite; perplexed (adj) confused;
warm jovial (adj) happy; cheerful puzzled
frustrated (adj) annoyed; judgmental (adj) judging others; persuasive (adj) trying to
exasperated; angered critical convince
funny (adj) humorous; amusing lackadaisical (adj) lazy; careless pessimistic (adj) not optimistic;
expecting the worst
gentle (adj) not hurtful; careful lament (n) great regret
poetic (adj) expressing beautiful
gloomy (adj) dark; sad; lighthearted (adj) happy, thoughts or words
depressed carefree
praising(adj) expressing warm
happy (adj) cheerful; in a good lofty (adj) arrogant approval
mood
macabre (adj) about death proud (adj) full of pride in
hateful (adj) intensely disliking something
something or someone malevolent (adj) mean; angry;
purposely hateful quizzical (adj) questioning;
haughty (adj) proud; vain; eccentric; odd; amusing
arrogant malicious (adj) purposely
hurtful; very mean reflective (adj) thoughtful;
honest (adj) truthful meditative; thinking
matter-of-fact (adj) truthful;
hopeful (adj) looking forward to straightforward regretful (adj) feeling bad about
something; optimistic the past
melancholy (adj) deeply sad
humorous (adj) funny relaxed (adj) calm
melancholy (n) a deep,
hysterical (adj) wildly emotional pervasive, lasting sadness relieved (adj) following state of
idealistic (adj) thinking of what lessened hardship, pain, or worry
mischievous (adj) up to no good
is best; optimistic; not realistic remorseful (adj) full of regret
mocking (adj) scornful;
imaginative (adj) creative; using ridiculing; making fun of sorrowful (adj) deeply sad
the imagination
negative (adj) unhappy; reverent (adj) deeply respectful
impersonal (adj) not influenced pessimistic; critical
by personal feelings sarcastic (adj) sneering; saying
nervous (adj) concerned; the opposite of what you mean
indifferent (adj) unconcerned; worried as a way of being unfriendly or
not interested making a point
neutral (adj) disinterested;
indignant (adj) angry because neither for nor against sardonic (adj) scornful; very
something is not fair sarcastic
nostalgic (adj) thinking about or
informal (adj) relaxed; not longing for the past satirical (adj) making fun of
formal something to show its weakness
objective (adj) without or to teach a lesson
innocent (adj) not guilty prejudice; fair; unbiased
inspirational (adj) encouraging; optimistic (adj) hopeful; positive
reassuring

106
sentimental (adj) thinking
about feelings, especially when
remembering the past
serious (adj) not joking around
sincere (adj) honest; truthful
skeptical (adj) not easily
convinced; having doubts
sneering (adj) disparaging;
contemptuous; hurtful
snobbish (adj) acting as though
one is better than everyone else
solemn (adj) sad or quiet
reflection
solitary (adj) lonely
stiff (adj) formal, serious
straightforward (adj) direct;
honest
stubborn (adj) refusing to yield
surprised (adj) startled; amazed
suspenseful (adj) tense
sympathetic (adj)
compassionate
tearful (adj) very sad
tense (adj) nervous
threatening (adj) menacing
thwarted (adj) frustrated at the
inability to succeed
tolerant (adj) accepting of
others
tragic (adj) very sad
unequivocal (adj) clear in its
meaning; unambiguous
urgent (adj) saying something
must be done soon; insistent
vindictive (adj) vengeful;
wanting to get back at someone
whimsical (adj) playful; funny;
odd

SAT v17.10 Unauthorized copying or reuse of any part of this page is illegal. 107
(SAT Teacher Edition)
108
Reading Vocabulary: Other Essential Words

Other Essential Words


abstract (adj) existing as an anecdote (n) a short entertaining challenge (n) the act of
idea but without a physical or story questioning the truth of a thing
concrete existence
appeal (v) to seek the opinion of characteristic (n) typical of a
abstraction (n) dealing with a higher authority person, place, or thing
ideas rather than real events
appeal (n) serious or urgent characterize (v) to describe the
academic (adj) not of practical request distinctive nature of a thing
relevance; relating to education
argument (n) a series of points cite (v) to quote as evidence in
account (n) a description of an that support a central claim; the favor of
event author’s central claim
claim (v) to assert that
account for (v) to explain assert (v) to present a point or something is true, typically
argument without evidence
accurate (adj) correct in all
details assuage (v) to calm or soothe claim (n) a point that is asserted
acknowledge (v) to accept or assumption (n) a thing that is clarify (v) to make more clear
admit the existence of a truth accepted as true without proof
collaboration (n) the act of
adapt (v) to change to become assure (v) to tell someone working together to produce a
more suitable positively or confidently of thing
something; dispel doubts
adapted (adj) made suitable commend (v) to praise
for a new purpose; taken from attempt (v) to make an effort
another source commentary (n) the expression
attempt (n) the act of trying of opinions about an event or
address (v) to speak directly to a situation
person or group of people attitude (n) tone, an author’s
emotional state toward his common (adj) occurring, found,
admire (v) to regard with warm subject or done often; prevalent; shared
approval; to be fond of
attribute (v) to give credit for compare (v) to evaluate the
advent (n) the arrival of a something similarity or difference between
notable thing two things
attribute (n) a quality or feature
aesthetic (adj) dealing with regarded as characteristic of a concentrate (v) to focus one’s
beauty person or thing mental effort on
alliteration (n) use of the same bias (n) prejudice in favor or concession (n) a thing that is
letter or sound at the beginning against a thing granted in response to a demand;
of adjacent or closely connected the admission that an opponent
words. bolster (v) to support or has won
strengthen; to prop up
allude (v) to subtly reference conclusion (n) a judgement or
something causal (adj) acting as a cause decision reached by reasoning;
censor (v) to examine a the end or result of an event
analogy (n) a comparison
between two things, typically on thing and ban or suppress concrete (adj) existing in a real,
the basis of their structure and unacceptable parts of it physical form; not abstract
for the basis of explanation or challenge (v) to object or
clarification concur (v) to agree
question the truth of something
analysis (n) careful examination

SAT v17.10 Unauthorized copying or reuse of any part of this page is illegal. 109
(SAT Teacher Edition)
condemn (v) to express demystify (v) to make clearer or evident (adj) plain or obvious;
complete disapproval of; easier to understand easily perceived
denounce
depict (v) to show or represent evoke (v) to bring or call to the
confession (n) an admission of by drawing or describing in detail conscious mind
guilt
detract (v) to take away from; exaggerate (v) to represent
conjecture (n) speculation; an weaken something as larger, greater,
opinion formed on incomplete better, or worse than it actually is
information develop (v) to put together (an
argument); advance; evolve examine (v) to scrutinize,
consider (v) to think carefully analyze, or carefully examine
about something digression (n) a brief departure
from the main subject excerpt (n) part of a larger body
context (n) the circumstances or of text
setting of an event, statement, dilemma (n) a situation in which
or idea a difficult choice must be made exchange (n) a short
between two or more options conversation or argument
contrast (v) to highlight the
difference between two or more discussion (n) an exchange of exhibit (v) to deliberately display
things ideas; conversation
exonerate (v) to free from blame;
contrast (n) the state of being dismiss (v) to send away; to absolve
different from another thing to regard as unworthy of
consideration expose (v) to reveal; to make
controversy (n) a dispute visible
disparity (n) a great difference
convey (v) to communicate a express (v) to convey a thought
message dispute (v) to argue about or feeling by words or gesture
something
counter (v) to speak in extend (v) to make longer or
opposition to dispute (n) an argument wider; to apply in a new way

credibility (n) the quality of distinction (n) a difference or focus (n) central topic of
being believable and true contrast between similar things discussion
or people
criteria (n) the set of principles or furthermore (adv) in addition
standards by which a thing may diversity (n) having variety
generalization (n) a general
be judged or decided document (v) to record in statement or concept obtained
criticize (v) to indicate the written, photographic or other by inference from specific cases
faults of a person or thing in a form
hence (adv) “as a consequence”;
disapproving way dramatize (v) to present in a “for this reason”; similar to
culminate (v) to result in vivid or striking way; present as a “therefore”
play; exaggerate
debate (v, n) to argue; an highlight (v) to draw attention to
argument about something emphasize (v) to give special
importance or significance to a however (adv) in contrast to the
debate (n) an argument thing; to highlight previous statement

define (v) to state or describe the encourage (v) to give support, hypothesis (n) a proposed
exact meaning, nature, or scope confidence, or hope explanation
of a thing hypothetical (adj) supposed but
endorse (v) to declare one’s
deflect (v) to change the approval publicly; support not actually true or real
direction of a thing; redirect from illustrate (v) to serve as an
a straight path; shift the focus enumerate (v) to mention a
number of things one by one example of
away from a thing
episode (n) an event that is part impact (v) to affect or influence a
demonstrate (v) to clearly show person, place, or thing
the existence or truth of a thing; of a larger group of events
to explain how something works erroneous (adj) incorrect impair (v) to weaken or damage

110
implausible (adj) improbable; lack (n) the state of being oppose (v) to disapprove and
unlikely to happen; not likely true without or not having enough attempt to prevent
implication (adj) a conclusion memoir (n) an historical account outright (adv) altogether,
that can be drawn from written from personal knowledge completely
something (although not or special sources
explicitly stated) particularly (adv) to a higher
metaphor (n) a figure of speech degree than normal or average
imply (adj) strongly suggest in which a word is applied to an
object or action where it is not passage (n) a body of text
impression (n) an idea, feeling, literally applicable; ex: “The cat
or opinion about a person or personification (n) the
crept forward on velvet paws.” attribution of human
thing
misguided (adj) having or characteristics to a nonhuman
in spite of (conj) without showing faulty judgement or entity
being affected by the specific reasoning
consideration mentioned phenomenon (n) a fact or
mock (v) to ridicule by imitation situation that is observed to
inconsistency (n) the status of happen
not fitting in or agreeing with the moreover (adv) as a further
rest of the data matter; additionally plausible (adj) seemingly
reasonable or probable
indicate (v) to point out; to be a mutual (adj) shared; experienced
sign or symptom of or done by each of two or more plea (n) a request made in an
parties urgent or emotional manner
infer (v) to deduce or conclude
from evidence and reasoning narrative (n) a story pose (v) to present or constitute
rather than explicit statements (a danger, challenge, or difficulty)
narrator (n) the person who
inference (n) a deduction or recalls the events of a story postulate (v) to suggest or
conclusion based on evidence assume the existence; fact; or
and reasoning rather than neutralize (v) to cancel out; to truth of a thing
explicit statements offset
prejudice (n) bias; a
influence (n) the capacity to nostalgia (n) a feeling of longing preconceived notion that is not
have an effect on the character or for the past based on experience
development of a person or thing note (v) to notice or pay special present (v) to give something to;
inquiry (n) a question or line of attention to to assert information
questioning notion (n) idea prevalence (n) the degree to
insofar as (adv) to the extent which something is common
notwithstanding (adv) in spite
that of this primarily (adv) mainly; for the
inspire (v) to fill someone most part
novel (adj) new
with the urge or desire to do primary (adj) of chief
something objection (adj) an expression of importance; the main
opposition or disapproval consideration
interpretation (n) the action
of explaining the meaning of objective (n) a goal profound (adj) very great;
something observation (n) the action of intense; important
invoke (v) to cite or appeal to observing a thing in order to gain proof (n) evidence that confirms
something as an authority information from it that a thing is true
irony (n) a state of affairs that offer (v) to suggest or proffer prove (v) to confirm the truth of
seems deliberately contrary to something for a person to accept a thing
what one would customarily or reject as desired
expect provide (v) to make available for
opinion (n) a view or judgement use
justify (v) to show or prove to be formed about a thing
right or reasonable

SAT v17.10 Unauthorized copying or reuse of any part of this page is illegal. 111
(SAT Teacher Edition)
provocative (adj) causing sentiment (n) a feeling undermine (v) to work against;
annoyance, anger, or another to weaken
strong reaction serve to (adj) used for the
purpose of underscore (v) to draw attention
provoke (v) to prompt a to; to highlight
response; to stimulate an simulate (v) to imitate the
unwanted or emotional response appearance or character of universal (adj) well-known; of,
affecting, or done by all people
qualification (n) a statement or sketch (v) to give a basic
assertion that makes another less impression validity (n) truthfulness; actually
absolute supporting an argument or claim
sketch (n) a basic impression of
qualify (v) to lessen the absolute verify (v) to confirm as true
sought (v, adj) past tense of seek
nature of a statement; to be virtually (adv) nearly; almost
eligible for something sought (adj) desired
weigh (v) to carefully consider
question (v) to inquire; to inquire specify (v) identify clearly and
as to the truth of a statement definitively whereas (conj) “in contrast to
the fact”
quotation (n) a group of words speculate (v) to form a theory
taken from a text or speech without firm evidence whereby (conj) by which;
through which
recollection (n) a memory or substantiate (v) to provide
account of an event evidence to support

reference (n) the action of subtle (adj) not obvious; so


mentioning or alluding to delicate or precise as to be
something difficult to analyze or describe

reflect (v) to think deeply or support (v) give assistance to;


carefully about provide evidence in favor of

reform (v) to make changes in supposition (n) an uncertain


order to improve a thing belief

refute (v) to prove to be wrong tenuous (adj) very weak or


or false slight; barely linked

regard (v) to think of or consider theory (n) a system of ideas


a thing in a certain way intended to explain a thing

regard (n) attention or concern therefore (adv) for that reason;


for something consequently

relatively (adv) in relation, thesis (n) main idea that is


comparison, or proportion to put forward to be advanced or
something else supported

respectively (adv) separately or thus (adj) therefore; as a result of


individually in the order already this; consequently
mentioned tone (n) general mood; the
rhetorical (adj) for the purpose author’s attitude toward his
of advancing an argument; subject
intended to persuade trace (v) to give an outline of
righteous (adj) morally right or something; to find or describe
justifiable the origin of something

scope (n) the extent or area of trivialize (v) to make something


a subject that something deals seem less important than it
with or to which it is relevant actually is

112
Reading Vocabulary: Word Roots

Prefixes, Roots, and Suffixes


a/an - not, without ced/cess - yield, go doc/doct - teach
ab/a - from, away, apart chrom - color dol - pain
ac/acr - sharp, sour chron - time duc/duct - lead
ad/a - to, towards cide - murder ego - self
ali/altr - another circum - around en/em - in, into
am/ami - love clin/cliv - slope err - wander
ambi/amphi - both clud/clus/claus/clois - eu - well, good
ambl/ambul - walk shut, close
ex/e - out, out of
anim - mind, spirit, breath co/com/con - with,
fac/fit/fect/fy/fea - make,
together
ann/enn - year do
cogn/gno - know
ante/ant - before fal/fals - deceive
contra - against
anthrop - human ferb - boil
corp - body
anti/ant - against, opposite fid - faith, trust
cosmo/cosm - world
aud - hear flu/flux - flow
crac/crat - rule, power
auto - self fore - before
cred - trust, believe
belli/bell - war frac/fract - break
cresc/cret - grow
bene/ben - good fus - pour
culp - blame, fault
bi - two gen - birth, class, kin
curr/curs - run
biblio - book grad/gress - step
de - down, out, apart
bio - life graph/gram - writing
dec - ten, tenth
burs - money, purse grat - pleasing
demo/dem - people
cad/cas/cid - happen, fall grav/griev - heavy
di/diurn - day
cap/capt/cept/cip - take, greg - crowd, flow
dia - across
hold, seize habit/hibit - have, hold
cap/cip - head dic/dict - speak
hap - by chance
carn - flesh dis/dif/di - not, apart, away
helio/heli - sun

SAT v17.10 Unauthorized copying or reuse of any part of this page is illegal. 113
(SAT Teacher Edition)
hetero - other lud/lus - play nom/nym - name
hol - whole mac/maj/mas/max - great nov - new
homo - same, man macro - great nox/nic/nec/noc - harm
hydr - water mal - bad numer - number
hyper - too much, excess man - hand ob - against
hypo - too little, under mar - sea omni - all
in/ig/il/im/ir - not mater/matr - mother oner - burden
in/il/im/ir - in, on, into medi - middle oper - work
inter - between, among mega - great pac - peace
intra/intr - within mem/men - remember palp - feel
it/iter - between, among meter/metr/mens - pan - all
measure
ject/jet - throw pater/patr - father
micro - small
jour - day path/pass - feel, suffer
mis - wrong, bad, hate
jud - judge pec - money
mit/miss - send
junct/jug - join ped/pod - foot
moll - soft
jur - swear, law pel/puls - drive
mon/monit - warn
lat - side pen - almost
mono - one
lav/lau/lu - wash pend/pens - hang
mor - custom, manner
leg/lec/lex - read, speak per - through, by,
mor/mort - dead for, throughout or
lev - light
against,destruction
morph - shape
liber - free
peri - around
mov/mot/mob/mom -
lig/lect - choose, gather
move pet - seek, go twoards
lig/li/ly - bind
mut - change phil - love
ling/lang - tongue
nat/nasc - born phob - fear
liter - letter
nau/nav - ship, sailor phon - sound
lith - stone
neg - not, deny plac - calm, please
loqu/loc/log - speech,
neo - new pon/pos - put, place
thought
nihil - none, nothing port - carry
luc/lum - light

114
pot - drink, power sequ/secu - follow top - place
pre - before sign - mark, sign torp - stiff, numb
prom/pri - first sim/sem - similar, same tort - twist
pro - ahead, forth sin - curve tox - poison
proto - first sol - sun, alone tract - draw
prox/prop - near somn - sleep trans - across, over,
through, beyond
pseudo - false son - sound
trem/trep/turb - shake
pyr - fire soph - wisdom
umbr - shadow
quad/quar/quat - four spec/spic - see, look
uni/un - one
ques/quer/quis/quir - sper - hope
question urb - city
spers/spar - scatter
quie - quiet vac - empty
spir - breathe
quint/quin - five val/vail - value, strength
strict/string - bind
radi/rami - branch ven/vent - come
stuict/stru - build
rect/reg - straight, rule ver - true
sub - under
reg - king, rule verb - word
summ - highest
retro - backward vert/vers - turn
super/sur - above
rid/ris - laugh vict/vinc - conquer
surge/surrect - rise
rog - ask vid/vis - see
syn/sym - together, same
rud - rough, crude vil - base, mean
tacit/tic - silent
rupt - break viv/vit - life
tact/tag/tang - touch
sacr/sanct - holy voc/vok/vow - call, voice
ten/tin/tain - hold, twist
scrib/script/scriv - write vol - wish
tend/tens/tent - stretch
se - apart, away volv/volut - turn, roll
term - end
sec/sect/seg - cut vor - eat
terr - earth, land
sed/sid - sit test - witness
sem - seed, sow the - god
sen - old therm - heat
sent/sens - feel, think tim - fear, frightened

SAT v17.10 Unauthorized copying or reuse of any part of this page is illegal. 115
(SAT Teacher Edition)
116
Writing

SAT v17.10 Unauthorized copying or reuse of any part of this page is illegal. 117
(SAT Teacher Edition)
118
1 Introduction 120

General Strategy 123

Homework Drills:
Writing Unit #1

1 2 3
129 132 135

SAT v17.10 Unauthorized copying or reuse of any part of this page is illegal. 119
(SAT Teacher Edition)
Format and Difficulty of the SAT Writing Section
• There are 4 passages on the Writing section of the SAT. Each passage is followed by 10 or 11
multiple choice questions, for a total of 44 questions on the section. You'll have 35 minutes to
read the passages and answer the questions. That's 48 seconds per question.

• The passages are sometimes accompanied by informational graphics such as charts, graphs,
and tables. These informational graphics provide additional information that is relevant to
the passage. You may be asked to consider the information in graphic(s) as you choose which
answer choice would improve the passage by clarifying or correcting a particular statement.

• The questions on this section test your ability to repair grammatical errors and eliminate
rhetorical confusion.

• SAT Writing questions are NOT organized according to difficulty, so it's a good idea to skip
difficult or time-consuming problems and come back to them once you've answered all the
quicker, easier ones. There is no guessing penalty on the SAT, so make sure you've bubbled in
an answer for every problem on each section before time is called.

- ic Q: Q:
fo
in aph
gr
g Q: Q:
i n
it age
r
W ss
Pa
Q: Q: Q:

Q: Q: Q:
120
Topics of SAT Writing Passages
HUMANITIES PASSAGES SOCIAL STUDIES PASSAGES SCIENCE PASSAGES

These passages often These passages typically These passages typically


analyze topics and trends present information about present a scientific topic and
within the world of arts people, movements, or then explore its significance.
and letters. These passages events from history as well as The author is typically
examine trends in fine art, topics in the social sciences. concerned with hypotheses,
music, dance, poetry, and experiments, data, and cause
written language -- every These passages often feature and effect relationships.
form of expression, really. topics drawn from cultural These passages may draw
anthropology, sociology, from theories, inventions,
Many of these passages political science, psychology, and discoveries in the natural
focus on the backstories of linguistics, economics, and sciences, which include biology,
individuals who explored education. chemistry, physics, and earth
or developed a new form and space sciences.
of expression as a result of
overcoming some difficulty
or injustice in their lives.

AK OUT. . .
CAREER PATHS PASSAGES DON’T FRE b e a n expert on an
y
e d to
You don’t ne s. Understanding the
ic
This type of passage focuses of these top e passage is much more
on career-related material, purpose of th n knowing a bunch of
a
such as general interest
important th bout the topic of the
a
pieces on trends, issues, and
information
debates within common
career pathways.
passage.

y o u 're g iv e n two facts: lls.


Suppose ce
For example, you might
a s s a g e d e a ls with stem
read about trends in cyber 1. The p .
uasive essay
security, urban planning, 2. It's a pers
or modern library ou
organization practices. h m o re u s e ful. It tells y
c
Fact 2 is mu okout for the author's
to be on th e lo
v id e n c e h e or she uses
ee
thesis and th
.
to support it

SAT v17.10 Unauthorized copying or reuse of any part of this page is illegal. 121
(SAT Teacher Edition)
122
Concepts on the SAT Writing Section

Expression of Ideas: Standard English Convention:


Development Sentence Structure
Thesis, Topic Sentences, Claims, etc. Sentence Formation
Use of Evidence Sentence Boundaries
Relevance Subordination and Coordination
Quantitative Information Parallel Structure
Modifier Placement
Inappropriate Shifts in Construction
Shifts in Verb Tense, Mood, and Voice
Organization Shifts in Pronoun Person and Number
Logical Sequence
Introductions
Conclusions
Transitions Conventions of Usage
Pronoun Clarity
Possessive Determiners
Agreement Issues
Effective Language Use Frequently Confused Words
Precision of Word Choice Logical Comparisons
Economy of Word Choice Conventional Expression
Style and Tone
Syntax

Conventions of Punctuation
End-of-Sentence Punctuation
Quantitative Information Within-Sentence Punctuation
Tables, Charts, Graphs, and Figures Possessive Nouns and Pronouns
Items in a Series
Nonrestrictive and Parenthetical Elements
Unnecessary Punctuation

*Spelling is not tested on the SAT. You'll never be asked


to identify the proper spelling of a word on the Writing
section of the SAT.*

SAT v17.10 Unauthorized copying or reuse of any part of this page is illegal. 123
(SAT Teacher Edition)
How to Raise Your SAT Writing Score
1. Learn to practice effectively by reading and following the advice on the page 13.

2. Don't skim the passage. Read the passage, using active reading techniques.

3. Consider the scope of the question before you try to answer it. Does the question
require you to consider more than a single sentence? Make sure the answer you
choose doesn't fix one sentence and mess up another one.

4. When you spot an error in an underlined section of the passage, solve the error in
your head BEFORE you even look at the answer choices. Remember, all but one of
the answer choices are wrong, and they're designed to confuse you! Do your best
to solve the problem in your head before you look at the answers.

5. Manage your time. You've only got 35 minutes to answer 44 questions. That means
that you've only got 48 seconds per question. That may not sound like much time,
but it's actually more than you'll need for most questions, which is good because
you'll need to "bank time" on those easy questions, so you'll have enough time to
knock out the difficult ones.

6. Keeping your momentum is an important part of a good approach to the SAT


English section. This is true for a couple of reasons. Obviously, you need to move
quickly in order to finish the test, but when you get stuck on a hard problem, you're
actually better off returning to that problem later with fresh eyes.

7. Check the answer choices for clues as to what's being tested on that problem. For
example, if answer choices use plural verbs and some use singular verbs, then that's
big hint that the problem deals with subject verb agreement.

8. Don't be afraid to choose NO CHANGE. If you've read a problem a few times and
you can't see any way to improve the underlined text, then go ahead and choose
NO CHANGE. It's sometimes the correct answer.

9. Never leave a question blank! There's no penalty for guesses or wrong answers, so
use process of elimination, do your best to find the right answer, and make sure
you've bubbled in an answer on every problem before time expires.

124
Process of Elimination for SAT Writing
Eliminate any answer choice that:
is grammatically incorrect
fails to communicate an idea clearly
includes unnecessary punctuation
includes unnecessary words or phrases
includes information that's irrelevant to the passage

Do not eliminate an answer choice that:


is grammatically correct
communicates an idea clearly
includes NO unnecessary punctuation
includes NO unnecessary words or phrases
includes only information that's relevant to the passage

A Right Answer Must Pass Three Tests:


1. Listen to your answer. The simplest sounding answer is often correct, but not if
that simplicity comes at the expense of clarity. Listening to an answer is a great
way to identify sentence fragments and adjective/adverb switches.

2. Look at your answer. Your ears can fool you, especially on questions involving
homophones (e.g. they're, their, and there), so be sure to use your eyes to spot
grammatical mistakes that you can't hear. Remember the grammar rules in this
book.

3. Consider your answer. If it sounds good and looks good, then it may be the
right answer. Just to be sure, take a moment to think about your answer. Check
your answer to make sure it actually makes sense. Consider the commonly
occurring errors in this section like subject verb agreement, unnecessary
punctuation, and idiom errors.

SAT v17.10 Unauthorized copying or reuse of any part of this page is illegal. 125
(SAT Teacher Edition)
15 Essential Grammar Concepts
1. Mark Up the Question
Always underline the words NOT, LEAST, and EXCEPT in the question.

2. Basic Punctuation
Period = Semicolon = Comma + Conjunction (FANBOYS or Subordinate)
FANBOYS = for, and, nor, but, or, yet, so
3. Subject Verb Agreement
Use the verb to find the subject by asking "Who or what performed this action?"
Then check for agreement in number. Singular verbs end in -s and must pair with
singular subjects. Plural verbs do not end in -s and must pair with plural subjects.

George runs. Michael runs. The two boys run.

4. Separating Non-essential Clauses


A non-essential clause is just extra information that the rest of the sentence
does not depend upon. For the purposes of separating a non-essential clause
from the rest of the sentence, remember:
Colon + Non-Essential Clause
Dash + Non-Essential Clause
Comma + Non-Essential Clause + Comma
Dash + Non-Essential Clause + Dash

5. Non-essential Lists or Explanations at the end of sentences


Complete sentence + Colon + List of items
Complete sentence + Colon + Extra information

6. -ING Words (Gerunds)


-ing words are NEVER verbs. They cannot be the action word in a sentence.
-ing words (especially having or being) usually result in wordy sentences.

7. Homophones and Apostrophes


Its = Possessive form of it It's = It is
Their = Possessive form of they They're = They are There = a location

126
8. Prepositions
Prepositions establish a spatial, temporal, or logical relationship between
two nouns in a sentence. Memorize the "Plane Test" and how to use it. Any
word that can logically be used to complete the following sentence is almost
certainly a preposition.

The plane flew __________ the clouds.

9. Unnecessary Commas
Avoid answer choices with a comma before he, she, they, or it. Also avoid
answer choices with a comma immediately before or after a preposition or the
word that.

10. Pronoun Switches


"You" and "one" mean different things. They're not interchangeable. Watch out for
sentences that begin with "one," then switch to "you" or "a person."

11. Pronouns for People/Things/Places/Times


who = people which = things where = places when = times that = things

12. Subjective & Objective Cases


Use who before the verb. Use whom after a preposition.
Who won the tournament? They gave the trophy to whom?

13. Parallelism in Lists and Word Pairs


All items in a list must match. (verb, verb, verb or noun, noun, noun)
Some word pairs require parallel construction (e.g. not only this but also that)
I ate a lasagne, danced a jig, and watched a movie.
I love eating lasagne, dancing jigs, and watching movies.

14. Comparisons
Nate is tall (1). Jim is taller than Nate (2). I am the tallest in the group (3+).

15. Could Have / Would Have / Should Have


Never use "of" in place of "have" in any of the above constructions.
Incorrect: I could of won it all!
Correct: I could have won it all!

SAT v17.10 Unauthorized copying or reuse of any part of this page is illegal. 127
(SAT Teacher Edition)
USE THE ANSWER CHOICES AS CLUES!

SAT English questions would be a lot easier if each problem told you exactly which concept
that problem tests. Wouldn't that be great? Well you're in luck, my friend!

Begin by reading the text to see if you can spot the error on your own. There's no need to panic
if you can't. It's not a big deal. It happens to everyone. But, unlike everyone else, you've got a
sweet strategy to employ when you get stumped. You can use the answer choices to figure out
what the question is testing.

Direct your attention to the answer choices. You'll probably see something like this:

A) NO CHANGE
B) has done
C) did
D) does

What do you notice about the answer choices? If the verb tense is different in three of the
answers, then you're almost certainly being tested on your ability to spot a tense error. Cool,
huh? Let's check out another one.

A) NO CHANGE
B) was done
C) were done
D) is done

This time, you'll see plural and singular verbs. That should tell you that you're being tested on
your ability to spot subject verb agreement. Makes the problem a lot easier, huh? Last one.

A) NO CHANGE
B) however,
C) on the contrary,
D) therefore,

Looks like we've been given a bunch of conjunctions. Because conjunctions are used to
connect ideas in a logical manner, we need to figure out how the ideas being connected are
logically related. Then, our answer should be obvious. Also, notice that answers B and C can be
eliminated because they're interchangeable. They can't both be right, so they must both be
wrong.

128
Writing Unit #1
Homework Drill #1

SAT v17.10 Unauthorized copying or reuse of any part of this page is illegal. 129
(SAT Teacher Edition)
130
ER:
REMEMB answers ing
r y o u r
Ente in stant sc
or
e f o r
onlin olutions!
v id e o s
SAT v17.10 and
Unauthorized copying or reuse of any part of this page is illegal.
(SAT Teacher Edition)
131
Writing Unit #1
Homework Drill #2

132
SAT v17.10 Unauthorized copying or reuse of any part of this page is illegal. 133
(SAT Teacher Edition)
ER:
REMEMB answers ing
r y o u r
Ente in stant sc
or
e f o r
onlin olutions!
v id e o s
134 and
Writing Unit #1
Homework Drill #3

SAT v17.10 Unauthorized copying or reuse of any part of this page is illegal. 135
(SAT Teacher Edition)
136
SAT v17.10 Unauthorized copying or reuse of any part of this page is illegal. 137
(SAT Teacher Edition)
138
2 Parts of Speech 140

Adjectives vs. Adverbs 149

Verb Tense 153


Writing Unit #2

Subject Verb Agreement 163

Pronoun Agreement and Clarity 170

Homework Drills:

1 2 3
181 184 187

SAT v17.10 Unauthorized copying or reuse of any part of this page is illegal. 139
(SAT Teacher Edition)
Parts of Speech

140
SAT v17.10 Unauthorized copying or reuse of any part of this page is illegal. 141
(SAT Teacher Edition)
ER:
REMEMB answers ing
r y o u r
Ente in stant sc
or
e f o r
onlin olutions!
v id e o s
142 and
Parts of Speech Review
1. Verbs
Verbs indicate actions or states of being.

to enchant to dominate to impress to champion to revile


to be to coin to listen to provide to insist
to cheer to purchase to seem to appear to collapse
to eat to sit to invite to jog to repair

The examples above are all in the infinitive form (to + verb), but we must conjugate a
verb to indicate who or what is performing the action. The noun that's doing or being
something is the subject of the verb. The tense of the verb tells us when the action
occurred. Below, we've conjugated the regular verb to vote.

Conjugating regular verbs is easy:

Present: I vote today.


Present Perfect: I have voted eleven times since this morning.
Simple Past: I voted yesterday by mail.
Past Perfect: I had voted before the final debate.
Conditional: I would vote for you if you were on the ballot.
Past Conditional: I would have voted for you if I had found the polling place.
Future: I will vote tomorrow, and it will make a difference.
Future Perfect: I will have voted twenty times before the winner is announced.

Because irregular verbs are a bit trickier, they tend to show up on the SAT quite a bit.
Two in particular, to be and to have, are tested all the time. Check it out.

to be to have
Singular Plural Singular Plural
I am We are I have We have
You are You (pl.) are You have You (pl.) have
He, She, It, One is They are He, She, It, One has They have

The number of a verb tells us whether it is singular or plural. That's important to know,
so you can check for subject verb agreement on SAT problems.

Singular: I, you, he, she, it, each, one, anyone, everyone

Plural: We, you, they

SAT v17.10 Unauthorized copying or reuse of any part of this page is illegal. 143
(SAT Teacher Edition)
2. Nouns
Nouns indicate people, places, objects, and ideas, and can always be preceded by
the articles a(n) or the. Proper nouns refer to specific people and places.

research, dinosaurs, cheesecake, conflict, Martin Van Buren, Uzbekistan

Research indicates that dinosaurs rarely ate cheesecake.


Due to a scheduling conflict, Martin Van Buren was unable to visit Uzbekistan.

3. Pronouns
Pronouns take the place of nouns to make writing sound less tedious.

1st Person Singular = I 1st Person Plural = we


2nd Person Singular = you 2nd Person Plural = you
3rd Person Singular = he, she, it, one 3rd Person Plural = they

Dave went to Dave's local market to buy Dave a box of Dave's favorite cereal.
Dave went to his local market to buy himself a box of his favorite cereal.

Arthur is a big fan of horror movies. He watches them every night.

4. Adjectives
Adjectives modify nouns and pronouns.

disappointed, huge, green, new, heavy, first, foolish

Lisa felt disappointed to see a huge, green stain on her new sofa.
Heavy losses on the first day led the knights to question their foolish king.

5. Adverbs
Adverbs modify verbs, adjectives, and other adverbs. They often end with -ly.

hungrily, loudly, triumphantly, abruptly, politely, well

Michelle ate hungrily, belched loudly, and smiled triumphantly.


Brad abruptly flagged down a waiter and politely asked for the check.
The date did not go well.

144
6. Prepositions
Prepositions indicate spatial, temporal, or logical relationships between words in a
sentence. Most often prepositions tell the reader where two nouns are located relative
to one another. Sometimes, prepositions tell the reader when one event occurred
relative to another event. Occasionally, prepositions tell the reader why events took
place.

despite, into, toward, on, near, while, in

Despite the warning, the plane flew into the clouds toward the rainbow.
The book sat on the shelf near the fireplace while the phone rang in the kitchen.

7. Conjunctions
Conjunctions indicate relationships between words, phrases, and clauses.

for, and, but, or, yet, so, although, when, because, since

I'm very hungry, but I'm nowhere near hungry enough to eat cauliflower.
I've never eaten cauliflower, and I see no reason to start now.
Although I'm not actually allergic to it, I just tell everyone that I am.

SAT v17.10 Unauthorized copying or reuse of any part of this page is illegal. 145
(SAT Teacher Edition)
Parts of Speech

146
SAT v17.10 Unauthorized copying or reuse of any part of this page is illegal. 147
(SAT Teacher Edition)
ER:
REMEMB answers ing
r y o u r
Ente in stant sc
or
e f o r
onlin olutions!
v id e o s
148 and
Adjectives vs. Adverbs

SAT v17.10 Unauthorized copying or reuse of any part of this page is illegal. 149
(SAT Teacher Edition)
ADJECTIVE/ADVERB ERROR
The sentence uses an adjective to modify a verb.

Wrong: Jeremy spoke loud.


Right: Jeremy spoke loudly.

Wrong: The deliciously eggplant fell.


Right: The delicious eggplant fell.

ADJECTIVES
Adjectives answer questions about nouns (or pronouns):
Which one? The green car drove away.
What kind? The fearsome bear growled menacingly.
How many? He offered numerous excuses.
An adjective will usually come immediately before the word it describes.
An adjective can be used to describe a state of being, such as to be, to
become, to appear/seem, in which case it will appear after the verb.

Bob was happy. Lisa seemed angry.

An adjective CANNOT be used to describe a verb! LINKING VERBS


Some verbs, called linking verbs,
ADVERBS link the subject to an adjective:
to be (am, are, is, was, were), to
Adverbs answer questions about verbs: seem, to taste, to smell.
Where? The hedgehog ran outside. Some verbs can be both linking
When? Harold sold his toaster recently.. or action verbs, depending on
How? Agatha stood up quickly. the context of the sentence:
To what extent? Shelia is profoundly wrong.
The soup tastes spicy!
I taste the soup.
Adverbs can also describe the intensity of adjectives
and adverbs.
Many (but not all) adverbs end in –ly.
An adverb CANNOT be used to describe a noun or pronoun!

BASIC SKILLS PRACTICE

1 rudely
The fussy child behaved ____________ to waiters. (rude/rudely)

2 heatedly by scientists. (heated/heatedly)


The geology of Hawaii has always been contested ____________

3 graceful panda begin to dance. (graceful/gracefully)


I grabbed my camera as I saw the ____________

150
ADJECTIVES AND ADVERBS GET SWITCHED ALL THE TIME

Watch for sentences with a pair of underlined words, the first of which should be an adverb and the
second of which should be an adjective.

The presence of tornadoes on the moon is extremely rarely.

You'll be given a number of answer choices that fumble the words back and forth, getting one or the
other piece right, but the correct answer will correctly supply an adjective to modify a noun, pronoun,
or state of being.

The presence of tornadoes on the moon is extremely rare.

In this example, the noun presence is described as rare. To what extent? extremely rare!

If you're pretty good with standard English, then go ahead and trust your ear on these.

WATCH OUT FOR SNEAKY ADJECTIVE/ADVERB SWITCH QUESTIONS

Some adjective adverb switch questions will try to trick you with an obvious switch that takes your
attention away from the other concepts. Meanwhile, the answer choices will deal with punctuation
issues, such as semicolons or commas.

In this case, we don't need an adverb. We should use the adjective contagious to modify respiratory
infection as opposed to the adverb contagiously to modify how respiratory the infection is. It's also
worth noting that the comma in answer choice (B) is unnecessary because the adjectives cannot be
separated by the word and. (D) is clearly wrong because the semicolon is not followed by a complete
idea. This question may look like it's testing your punctuation skills, but it also requires you to
consider the rules that govern adjectives and adverbs.

REMEMBER THAT ADJECTIVES AND ADVERBS ARE MODIFIERS

Modifiers, such as adjectives and adverbs, must be placed directly next to the words they modify.
Because of that grammar rule, the SAT is pretty picky about the placement of adjectives and adverbs.
As a result, whenever you're asked where to put a single word within a sentence, the correct answer
usually hinges on making sure the correct word is being modified. Keep that in mind.

SAT v17.10 Unauthorized copying or reuse of any part of this page is illegal. 151
(SAT Teacher Edition)
Adjectives vs. Adverbs

152
Verb Tense

SAT v17.10 Unauthorized copying or reuse of any part of this page is illegal. 153
(SAT Teacher Edition)
154
SAT v17.10 Unauthorized copying or reuse of any part of this page is illegal. 155
(SAT Teacher Edition)
GET TO KNOW YOUR TENSES

The present tense is used to indicate that an action is occurring now (or the events of a story).
Mario sits in detention.

The present progressive is used to emphasize that an action is happening right now.
The children in detention are missing the ice cream party.
is/are + (-ing)

The present perfect is used for actions that began in the past and are still continuing in the present or
during a time period that hasn't completed or when the time frame is not specified.
Mario has missed many ice cream parties in the past. Mario has missed four ice cream parties this year.
has/have + past participle has/have + past participle

The simple past is used for actions that began and ended in the past.
Mario disrespected his teacher.
(-ed) for regular verbs

The past perfect is used to help establish the order of two events that began and ended in the past.
By the time Mario was released, all the ice cream had melted.
had + past participle

The present participle can be used to modify actions in the past, present, or future.
Sobbing uncontrollably, Mario was inconsolable.
Laughing maniacally, Mario plots his revenge.
Smiling menacingly, Mario will make the biggest ice cream sunday that anyone has ever seen!

WATCH FOR UNNECESSARY TENSE SWITCHES


The vast majority of questions dealing with tense errors test whether you can spot an
unnecessary tense switch. Follow this one simple rule, and you'll do just fine:
Unless the context of the sentence establishes a clear reason to change tense,
an underlined verb should match the tense established by other verb(s) in the
sentence or paragraph.

Make sure the tense of the underlined portion agrees with the part of the sentence or
paragraph that you cannot change.

Use the answer choices to find out what's being tested on the problem.
If the answer choices include single and plural verbs, then you're probably
being asked to identify and resolve a subject verb disagreement.

If the answer choices do NOT include both single and plural choices, then
you're probably being asked to eliminate a tense error, such as a tense switch.

156
THE MOST COMMON TENSE ERRORS ON THE SAT

1. Past participle used in place of simple past tense


The simple past tense can stand alone, but the past participle can only be used after a form of to
have or to be.

Wrong: Yesterday, Bart eaten dinner.


Right: Bart ate dinner.

2. Past participle used without "to have" or "to be"


The sentence requires the past participle but omits the helping verb before it. This is sometimes
called a "helpless past participle" because it lacks a helping verb.

Wrong: Since last week, Charles flown to New York and back.
Right: Since last week, Charles has flown to New York and back.

3. Simple past replaces past participle


Verbs in the simple past tense can stand alone. They don't require a helping verb, such as to have or
to be.

Wrong: As a child, I has went to the amusement park every summer.


Right: As a child, I went to the amusement park every summer.

4. Past participle without "-ed"


This is a pretty clear error. You've got a regular verb in the past participle tense without an -ed.

Wrong: The children were invite to the soccer tournament.


Right: The children were invited to the soccer tournament.

BE VERY SKEPTICAL OF ANSWERS THAT USE THE PHRASE


""WOULD HAVE" OR "WILL HAVE"
Answer choices that include the phrases "would have" or "will have" are nearly always wrong
because they tend to create awkward, wordy sentences and introduce tense switch errors.
Used properly, "would" has the opposite effect of the word "had" in establishing the order of
events in the timeline of a sentence.

As the director finished his film, he knew it would win an award.


Though Nate would regret the decision, he devoured his dessert in record time.

The word "having" is similar to "had" but acts as a modifier, not a verb.
Having arrived at the airport, Danielle checked her duffle bag.
Having acknowledged the mistake, Michael offered the man a refund.
SAT v17.10 Unauthorized copying or reuse of any part of this page is illegal. 157
(SAT Teacher Edition)
REGULAR VERBS
Regular verbs follow the basic rule: add an –ed ending to
convert a verb to the past tense form. Regular verbs like
jump are the same in the past tense and past participle forms.

Present Simple Past Past Participle


taste tasted (have) tasted
shout shouted (have) shouted
laugh laughed (have) laughed

IRREGULAR VERBS
Irregular verbs are tricky. They do not follow the basic “add
an -ed” rule. Be careful when dealing with irregular verbs.

Present Simple Past Past Participle


fly flew (have) flown
give gave (have) given
cut cut (have) cut

SKILLS PRACTICE
Circle the correct verb phrase.

2.1 I’m pretty sick of lasagna; we (had ate/have eaten) it every night this week.

2.2 If not for the final exam, Paul and Todd (would have done/would have did) better in our chemistry class.

2.3 In the novel I’m reading, the protagonist (goes/had gone) to question the suspect about the crime.

2.4 Darkness had already (fallen/fell) by the time it began to rain.

158
COMMON IRREGULAR VERBS
Present Simple Past Past Participle
be was/were (have) _________
been
become _________
became (have) become
begin began (have) _________
begun
bring _________
brought (have) brought
buy bought (have) _________
bought
come came (have) _________
come
do _________
did (have) _________
done
drink _________
drank (have) drunk
eat ate (have) _________
eaten
fall fell (have) _________
fallen
feel _________
felt (have) _________
felt
fight _________
fought (have) fought
forget forgot (have) forgotten
give gave (have) _________
given
go went (have) gone
have _________
had (have) _________
had
know knew (have) known
make made (have) _________
made
put _________
put (have) put
say said (have) _________
said
show _________
showed (have) _________
shown
sing sang (have) _________
sung
sleep _________
slept (have) slept
speak spoke (have) _________
spoken
swim _________
swam (have) swum
take took (have) _________
taken
teach _________
taught (have) taught
think _________
thought (have) _________
thought
wake woke (have) _________
woken
write wrote (have) _________
written

REMEMBER
Watch out for irregular verbs. They do NOT follow the standard rules of verb conjugation.

Irregular verbs have two different forms for simple past tense and past participle.

ONLY use helping verbs like had or have when you’re using the past participle form of the verb.

SAT v17.10 Unauthorized copying or reuse of any part of this page is illegal. 159
(SAT Teacher Edition)
Verb Tense

he
f e w m ore on t
Try a ew
x t p a g e . It's a n f the
ne o
p ic , b u t it's part
to
sage.
same pas
160
SAT v17.10 Unauthorized copying or reuse of any part of this page is illegal. 161
(SAT Teacher Edition)
162
Subject Verb Agreement

SAT v17.10 Unauthorized copying or reuse of any part of this page is illegal. 163
(SAT Teacher Edition)
SUBJECT VERB AGREEMENT

The first thing you should know about Subject Verb Agreement problems is that they're virtually
guaranteed to appear three or more times on every SAT. With that in mind, you should consider this an
opportunity to quickly pick up a few points.

Quick review: every sentence must have a subject and a verb. The subject and the verb must agree.

A singular subject requires a singular verb.


A plural subject requires a plural verb.
Two singular subjects joined by the word and require a plural verb.

“VERB YOUR WAY TO THE SUBJECT”


1. Identify the verb (action word) in the sentence and underline it.
2. Find the subject of the verb and underline it. To find the subject, ask who
(or what) does the verb?
3. Read the subject and verb together aloud.
If this sounds funny, then the subject and the verb disagree.


correct: The knight runs fast. “ knight runs


correct: The dragons run faster. “ dragons run


incorrect: The dragon catch the knight. “ dragon catch


incorrect: The dragons catches the knight. “ dragons catches

BASIC SKILLS PRACTICE


Use your strategy on these super easy practice problems.

1 The campfire burn brightly. campfire ___________
“___________ burn Agree / Disagree

2 The pirates yells at me. pirates
“___________ yells
___________ Agree / Disagree

3 My laundry smell dirty. laundry ___________
“___________ smell Agree / Disagree

4 The panda smiles at me. panda
“___________ smiles
___________ Agree / Disagree

164
GET RID OF THE JUNK
Don’t be afraid of long sentences. Identify the
prepositional phrases and isolate the core sentence!
FIND THE CORE SENTENCE
DO NOT trust your ear until you've identified the
prepositional phrases. Subjects and verbs can NEVER be inside
prepositional phrases.
Read only the core sentence aloud, and then trust
your ear. To find the subject and verb easily,
draw brackets around the prepositional
phrases and skip over them when you
reread the sentence.

THE COUNTABILITY EXCEPTION


IDENTIFYING PREPOSITIONS: THE CLOUD TEST If the noun in the prepositional phrase is
Prepositions are words that show the relationship between countable, use a plural verb:
two words in a sentence. If you’re not sure if a word is a
preposition, try using the cloud test.
Some of the bottles are on the floor.

LOCATION
If the noun in the prepositional phrase is
5 beneath
The airplane flew _______________ the clouds. uncountable, use a singular verb:
6 near, etc.
The airplane flew _______________ the clouds.
Some of the milk is on the floor.
7 past, etc.
The airplane flew _______________ the clouds.

TIME
8 after
The airplane flew _______________ the clouds.
9 before, etc.
The airplane flew _______________ the clouds.

LOGICAL
despite
10 The airplane flew _______________ the clouds.

11 The airplane flew because of, etc. the clouds.


_______________

BASIC SKILLS PRACTICE


Underline the prepositional phrases. Then read the core sentence aloud to see if the subject and verb agree.

12 The pudding inside the DVD players were put there by mistake. Agree / Disagree

13 Someone from the headquarters on Main Street wish to discuss details of the television show. Agree / Disagree

14 Without a thought, Anna drove past the valet and into the movie theater on Sycamore Street. Agree / Disagree

16 The feelings of disappointment among the archers in the courtyard was quite noticeable. Agree / Disagree

SAT v17.10 Unauthorized copying or reuse of any part of this page is illegal. 165
(SAT Teacher Edition)
WATCH FOR SENTENCES WITH INFINITIVE/GERUND SWITCHES

Infinitive = to + verb Gerund = verb + ing


to dance dancing
Sometimes the SAT will give you a sentence in which the infinitive form of a verb has been replaced by a
gerund. These can be tricky to spot because there is no firm rule to tell you which one should be used in a
given sentence. Let's look at a few examples.

Incorrect: Dave has raised $100,000 supporting cancer research.


Correct: Dave has raised $100,000 to support cancer research.

It may appear that these two sentences mean the same thing, but if you really think about it, they're
slightly different. In the first one, Dave has raised the money while supporting research. In the second
one, Dave has raised the money to support future research. This is a subtle difference, but it's significant.

Sometimes, switching an infinitive for a gerund can create a new error if you don't also place the correct
preposition in front of the gerund. These can be super tricky to spot, so be careful.

Incorrect: The senator was praised for his prudence declining the new spending bill.
Correct: The senator was praised for his prudence in declining the new spending bill.

BASIC SKILLS PRACTICE


Supply the correct preposition for each of the following verbs.

2.1 destined __________


to win 2.9 argue __________
with someone

2.2 adapt __________


to changes 2.10 stare __________
at the sun

2.3 triumph __________


over adversity 2.11 protect __________
against disaster

2.4 thrilled __________


by/with the results 2.12 based __________
on the proposal

2.5 dream __________


of peace 2.13 escape __________
from captivity

2.6 depend __________


upon/on someone 2.14 apply __________
for the job

2.7 charged __________


with a crime 2.15 true __________
to his promise

2.8 relevant __________


to the topic 2.16 prohibited __________
from disrupting

166
Subject Verb Agreement

SAT v17.10 Unauthorized copying or reuse of any part of this page is illegal. 167
(SAT Teacher Edition)
168
SAT v17.10 Unauthorized copying or reuse of any part of this page is illegal. 169
(SAT Teacher Edition)
Pronoun Agreement and Clarity

170
SAT v17.10 Unauthorized copying or reuse of any part of this page is illegal. 171
(SAT Teacher Edition)
PRONOUNS MUST AGREE IN NUMBER WITH THE WORDS THEY REPLACE

Pronouns take the place of nouns to prevent writers from having to repeat the same noun over and
over again as they write. Check it out.

Redundant: When pronouns are used correctly, pronouns replace other nouns in a sentence.
Correct: When pronouns are used correctly, they replace other nouns in a sentence.

Pronouns are a great way to make your writing sound more sophisticated, but you must be careful
when using them. Just as with subjects and verbs, pronouns must agree with their antecedents (the
words they've replaced). Disagreement between a pronoun and its antecedent can cause confusion.

Incorrect: I love to eat blueberry pudding, but I hate how long they take to prepare.
Correct: I love to eat blueberry pudding, but I hate how long it takes to prepare.

Pronoun disagreement errors can be pretty obvious, but sometimes they're subtle. Check this out.

Incorrect: I love to eat blueberries, but I hate how it stains my carpet.


Correct: I love to eat blueberries, but I hate how they stain my carpet.

Pretty subtle, right? How about this one?

Incorrect: I washed a batch of blueberries, and put them in the refrigerator.


Correct: I washed a batch of blueberries, and put it in the refrigerator.

Also keep in mind that pronoun disagreement often involves an antecedent in one sentence and a
pronoun later in the next sentence or later in the paragraph. A pronoun can only refer to a word that
actually appears in the passage.

Incorrect: Grandma's recipe for blueberry pie is a secret. We have been baking them for years.
Correct: Grandma's recipe for blueberry pie is a secret. We have been following it for years.

172
GRAMMAR RULE!
Always use a singular pronoun with a singular noun.
Always use a plural pronoun with a plural noun.

THESE WORDS AND PHRASES


PAIR WITH PLURAL VERBS:
we, us,

THESE WORDS AND PHRASES they, them, you (as in “you guys”)
PAIR WITH SINGULAR VERBS: these, those
few, both, several, many, most,
I, me, mine, my
a number of
you, your, yours
he, him, his
she, her, hers
it, its, this, that, anything
each, either, neither, every
-one: one, everyone, anyone, someone, no one,
-body: everybody, anybody, somebody, nobody
the amount of, the number of

BASIC SKILLS PRACTICE


Underline each pronoun and the noun it refers to. When you find a pair that disagrees,
correct it in the space provided.

1 I love popcorn but they make me thirsty.


I love popcorn but it makes me thirsty.
2 I dropped my goggles, but it didn’t break.
I dropped my goggles, but they didn’t break.
3 The stone statues outside the building look as if it took years to carve.
The stone statues outside the building look as if they took years to carve.
4 The team has been in a slump ever since they cancelled weekly practices.
The team has been in a slump ever since it cancelled weekly practices.
5 Water makes our lives easier; life without it would be difficult.
(correct as written)
6 Economics is a challenging class, but they are very important.
Ecomonics is a challenging class, but it is very important.

SAT v17.10 Unauthorized copying or reuse of any part of this page is illegal. 173
(SAT Teacher Edition)
TIPS & TRICKS
Pronoun Case Errors will often show up more than once on an SAT.
Make sure you understand how to fix these errors.

If a pronoun tells who does the action, use the subjective case.
If a pronoun tells what or whom the action is performed on, use the objective case.
If a pronoun follows a preposition, use the objective case.
When in doubt, go with who.

SUBJECTIVE CASE

I punch the magician.


You punch the magician.
(He/She/It) punches the magician.
We punch the magician.
They punch the magician.
OBJECTIVE CASE
Who punches the magician?
The magician punches me.
The magician punches you.
The magician punches (him/her/it).
The magician punches us.
The magician punches them.
The magician punches whom?

BASIC SKILLS PRACTICE


Identify the incorrect pronoun and its case. Write the correct pronoun and its case below.

1 In the case of Carlos and I, a little preparation would have gone a long way.
In the case of Carlos and me, a little preparation would have gone a long way.

2 Him and Lisa left early because they can never find their car.
He and Lisa left early because they can never find their car.

3 During the talent portion, the judges were forced to choose between the puppeteer and he.
During the talent portion, the judges were forced to choose between the puppeteer and him.

4 The other geniuses and me will be celebrated at a special awards show in the auditorium.
The other geniuses and I will be celebrated at a special awards show in the auditorium.

5 Whom was the last person to use the bathroom?


Who was the last person to use the bathroom?

174
TIPS & TRICKS
DO NOT use that or which to refer to a person or people.
When a pronoun refers back to a person or people use: who or whom.

WHEN & WHERE ARE NOT PRONOUNS

Example:
1.1 Wrong: A home run is when the batter hits
the ball over the outfield fence.
Wrong: A home run is where the batter hits
the ball over the outfield fence.
Right: A home run is a situation in which the
________________________________
batter hits the ball over the outfield fence.
________________________________

USE THAT OR WHICH FOR THINGS

The relative pronoun who refers to people only; which refers to things only; that is
used to refer to things. (Informally, that can refer to people but not on the SAT. )

Example:

2.1 Wrong: Bears who eat wild berries smell nice.


Right: Bears that eat wild berries smell nice.
_______________________________

USE WHO OR WHOM FOR PEOPLE

Example:
3.1 Wrong: Painters which use watercolors are very talented.
Wrong: Painters that use watercolors are very talented.
Right: Painters who use watercolors are very talented.
__________________________________________

BASIC SKILLS PRACTICE


Circle the correct word or phrase.

1 Mitch was the only person that / who stayed behind to fight the swamp creature.

2 Dante’s jaw, that / which / who he kept flapping, reminded me of a wind-up toy.

3 The only camper that / which / who volunteered to investigate the scary noise was Ned.

4 According to my research, a hung jury is when / a situation in which / how the jury
cannot reach a verdict.

5 My putter is definitely the golf club that / who is best for throwing in anger.

SAT v17.10 Unauthorized copying or reuse of any part of this page is illegal. 175
(SAT Teacher Edition)
KEEP YOUR PRONOUNS CONSISTENT
If you starts a sentence, you had better finish it.
If one starts a sentence, one had better finish it.

YOU & ONE DON’T MIX

Wrong: If you switch pronouns in a sentence, one has committed an error.

Wrong: If one switches pronouns in a sentence, you have committed an error.

Right: If you do not switch pronouns in a sentence, you have not committed an error.

Right: If one does not switch pronouns in a sentence, one has not committed an error.

BASIC SKILLS PRACTICE


Fix these sentences by resolving the Pronoun Switch errors.

1 If one is happy, you are smiling.


If one is happy, one is smiling.
If you are happy, you are smiling.
2 If you are sad, one is frowning.
If you are sad, you are frowning.
If one is sad, one is frowning.
3 One should not buy a car if you do not have enough money for gas.
One should not buy a new car if one does not have enough money for gas.
You should not buy a new car if you do not have enough money for gas.
4 You should purchase a fancy sweater if one wishes to get better at tennis.
You should purchase a fancy sweater if you wish to get better at tennis.
One should purchase a fancy sweater if one wishes to get better at tennis.
5 Despite your rhythm, you were actually pretty funny looking when one
attempted to dance.
Despite your rhythm, you were actually pretty funny looking when you attempted
to dance.

176
TIPS & TRICKS
Watch for pronouns that are underlined. The question may
test your ability to spot an ambiguous pronoun.
If a sentence does contain an underlined pronoun (it, they,
them) and you cannot identify the noun it refers to, you’re
dealing with an ambiguous pronoun.

DON’T ASSUME ANYTHING


When you come across a pronoun, never assume from the context
that you know which noun it refers to.

Example:
The blimp hit the building, and then it popped.

What popped?
A. the blimp
B. the building
C. the housing bubble
D. Timmy
E. Cannot be determined from this sentence.

The answer is E, cannot be determined from this sentence. It is an


ambiguous pronoun. The reader cannot be sure what it refers to.
It could be the blimp or the building.

SKILLS PRACTICE
Each of the following problems has more than one possible meaning.
Circle the ambiguous pronoun.

1 Bernard spoke to Mr. Gonzalez before he lost his temper.


Bernard spoke to Mr. Gonzalez before Bernard lost his temper.
Bernard spoke to Mr. Gonzalez before Mr. Gonzalez lost his temper.
2 In my autobiography, it says repeatedly that I’m a genius.
My autobiography states repeatedly that I am a genius.
3 After I told Bill the truth, I told Lionel, which infuriated him.
After I told Bill the truth, I told Lionel, which infuriated Bill.
After I told Bill the truth, I told Lionel, which infuriated Lionel.
4 Though Neisa worked at the ice cream shop for a year, she never tried it.
Though Neisa worked at the ice cream shop for a year, she never tried the ice cream.
5 The detectives questioned the mob bosses, and they were angry.
The detectives were angry as they questioned the mob bosses.
The detectives questioned the angry mob bosses.

SAT v17.10 Unauthorized copying or reuse of any part of this page is illegal. 177
(SAT Teacher Edition)
Pronoun Agreement and Clarity

178
SAT v17.10 Unauthorized copying or reuse of any part of this page is illegal. 179
(SAT Teacher Edition)
180
Writing Unit #2
Homework Drill #1

SAT v17.10 Unauthorized copying or reuse of any part of this page is illegal. 181
(SAT Teacher Edition)
182
SAT v17.10 Unauthorized copying or reuse of any part of this page is illegal. 183
(SAT Teacher Edition)
Writing Unit #2
Homework Drill #2

184
SAT v17.10 Unauthorized copying or reuse of any part of this page is illegal. 185
(SAT Teacher Edition)
186
Writing Unit #2
Homework Drill #3

SAT v17.10 Unauthorized copying or reuse of any part of this page is illegal. 187
(SAT Teacher Edition)
188
SAT v17.10 Unauthorized copying or reuse of any part of this page is illegal. 189
(SAT Teacher Edition)
190
3 Sentences & Fragments 193

Run-On Sentences 201

Colons, Dashes, and Semicolons 207


Writing Unit #3

Commas and
Non-Essential Clauses 211

Possessives, Plurals, and


217
Contractions

Homework Drills:

1 2 3
222 225 228

SAT v17.10 Unauthorized copying or reuse of any part of this page is illegal. 191
(SAT Teacher Edition)
192
Sentences & Fragments

SAT v17.10 Unauthorized copying or reuse of any part of this page is illegal. 193
(SAT Teacher Edition)
SENTENCE FRAGMENT ERROR
The sentence lacks a subject/verb pair or does not
express a complete idea.

THE RULES OF COMPLETE SENTENCES


A sentence must have at least one independent clause.
An independent clause must have a subject and a conjugated verb that
corresponds with the subject. In other words, an independent clause
must express a complete idea.
If a sentence contains more than one independent clause, the
clauses must be joined correctly to avoid a run-on sentence or logical
inconsistency.

How to Build a Simple Sentence


Sentences can be as simple as a stand-alone independent clause: subject + conjugated verb.
Sentence: Anna begins.
Simple sentences can get very complicated when we add additional ingredients:
Sentence: In her art studio, concentrating intently as she mixes the colors on her palette, Anna begins to
envision the subject of her painting - perfect blue buildings - which she will bring to life in vivid
detail, using fancy imported oil paints on canvas.

How to Build a Sentence Fragment


Regardless of its length, a string of words without an independent clause is always a fragment.
Fragment: In her art studio.
Fragment: Concentrating intently as she mixes the colors on her palette.
Placing the subject + conjugated verb within a non-essential clause results in a fragment as well.
Fragment: Which she will bring to life in vivid detail.

How to Spot a Sentence Fragment on the SAT English Section


Familiarize yourself with the various parts of speech, dependent clauses, and independent clauses.
Make sure you understand how they can interact to form complete sentences and sentence fragments.

You'll notice in the pages ahead that "w-words" have a way of making an otherwise complete sentence into a
fragment. Be on the lookout for "w-words."

194
TRICKY COMPLETE SENTENCES THAT FEEL LIKE FRAGMENTS

A Pronoun May Be Used as the Subject of a Complete Sentence.


Sentence: It rains.
Remember that nouns can be replaced with pronouns without affecting the completeness of a sentence.
Some students read a sentence with a pronoun such as he, she, it, or they, and conclude that the sentence
cannot be complete unless the subject is specifically named within the sentence. In reality, the sentence has
a subject (it) and a conjugated verb (rains).

If you cannot with certainty identify the noun that a pronoun replaced, then you're dealing with an
ambiguous pronoun. That's an error of a different sort, which we'll deal with when we talk about pronouns.

A Pronoun May Imply Words That Aren't Written on the Page.


Sentence: Some complain.
Out of context, these sentences don't make much sense. Presumably, we're dealing with some people, but
it's not really clear. Issues with clarity aside, we've still got a complete sentence because we've got a subject
(some) and a conjugated verb (complain).

TRICKY FRAGMENTS THAT FEEL LIKE COMPLETE SENTENCES


Some sentences look or feel complete, but they're actually fragments. These are more common and more
difficult than the examples above of complete sentences that look or feel incomplete. Let's take a look.

Pronoun + "of which" or "of whom" = Dependent Clause = Sentence Fragment


Fragment: Those of whom went to the beach were given warnings.
Fragment: All of which were ignored.
When an indefinite pronoun is followed by of which or of whom, it creates a dependent clause. Remember
that a dependent clause cannot stand alone as a complete sentence. Also, consider these two sentences in
the context of one another. In the box above, we saw that an ambiguous pronoun can give the impression
that a complete sentence is incomplete. Now we're seeing the opposite. A sentence fragment that supplies
useful information can cause you to overlook the sentence fragment's incompleteness. In the example
involving the beach and warnings, each sentence is clearly a fragment when considered by itself, but
together, they seem to form a complete idea. Don't fall for this trick.

Use This Information to Identify Run-on Sentences


Run-on: The dance was attended by 200 students, none danced.
(Two complete ideas joined by a comma).
Sentence: The dance was attended by 200 students, none of whom danced.

SAT v17.10 Unauthorized copying or reuse of any part of this page is illegal. 195
(SAT Teacher Edition)
ADVERBS HAVE NO GRAMMATICAL AFFECT ON A SENTENCE
Of course, you should watch out for misplaced modifiers, but those are logical errors. Adverbs modify verbs
and clauses. Adverbs are never to blame for creating sentence fragments. If you've got a complete sentence,
adding an adverb cannot turn it into a fragment. If you've got a fragment, adding an adverb cannot turn it
into a complete sentence.
Complete Sentence: We bought a small bird.
Complete Sentence: Luckily, we bought a small bird.
Fragment: Bought a small bird.
Fragment: Quickly bought a small bird.

NON-ESSENTIAL CLAUSES: WATCH FOR THE "W-WORDS"


Unnecessary information can be added between the subject and the verb, making it difficult to spot a
number of commonly occurring SAT errors. It's especially successful at disguising sentence fragments, subject
verb disagreement, pronoun disagreement, and tense switches. The information that's added may be useful,
but it's not essential to the sentence, meaning you should make sure the sentence functions as a complete
sentence and makes sense without that extraneous information.

Non-essential clauses often begin with one of the "w-words" we mentioned earlier, such as whose, whom,
which, and where.
Complete Sentence: Michael, who has lived in Newport all his life, finally visited the Frozen Banana Stand.
Complete Sentence: The Frozen Banana Stand, which is located on Marine Avenue, is a local institution.

The most important take-away from this lesson is that the main verb of a sentence cannot show up inside
a non-essential clause. That's because the verb inside the non-essential clause pairs with the "w-word" that
started the non-essential clause, leaving you with a fragment.
Fragment: The frozen banana, which is delicious.
Fragment: Michael, whose car is a rather unorthodox piece of machinery.

APPOSITIVES: NON-ESSENTIAL CLAUSES WITHOUT THE "W-WORDS"


Sometimes non-essential clauses, extra information about the subject of the sentence, don't begin with
"w-words." We'll stick with Michael and the Frozen Banana Stand to make this point about appositives.

Sentence: Michael, a lifelong resident of Newport, finally visited the Frozen Banana Stand.
Sentence: The Frozen Banana Stand, located on Marine avenue, is a local institution.
These non-essential clauses don't have to show up in the middle of the sentence. In fact, you can start or end
a sentence with an appositive.
Sentence: A lifelong resident of Newport, Michael finally visited the Frozen Banana Stand.
Sentence: Marine avenue is home to the Frozen Banana Stand, a local institution.

196
-ING WORDS ARE NOT VERBS.
A verb has two participles:
The present participle is formed by adding -ing to the end of the verb.
to request requesting
to announce announcing
to breakdance breakdancing

The past participle is formed by adding -ed or -n to the end of the verb (except with irregular verbs).
to request requested
to announce announced
to breakdance breakdanced

A participle phrase begins with a participle and can be in either the present or past tense.
Most of the time, participle phrases involve the present tense (-ing form).
Sentence: Requesting everyone's attention, Theodore began his speech.
Sentence: Announcing his candidacy, Theodore concluded his speech.
Sentence: Breakdancing his way out of the room, Theodore felt confident he'd be elected.

To form the past tense, we can use the present participle having + past participle of the main verb.
Sentence: Having requested everyone's attention, Theodore began his speech.
Sentence: Having announced his candidacy, Theodore concluded his speech.
Sentence: Having breakdanced out of the room, Theodore felt confident he'd be elected.
It's worth noting that a participle phrase can show up at the middle or end of a sentence too.
Sentence: Theodore began his speech, having breakdanced his way into the room.
Sentence: Theodore, having ignited a breakdancing craze, concluded his victory speech.

Participle phrases cannot stand alone as sentences. Check it out.


Fragment: Having denied the rumors of a fund raising scandal.
Fragment: Theodore, having been forced to resign.

ANSWER CHOICES THAT CONTAIN --ING WORDS ARE USUALLY WRONG.


Answer choices that feature -ing words are usually wrong. The reason is simply that they're difficult to
incorporate into sentences without making a wordy mess. The two most commonly tested -ing words on the
SAT are having and being. You should be very skeptical of any answer choice that includes one of those two
words. The correct answer to this type of problem often includes one of the conjugated forms of being.

present past

Singular is was

Plural are were

SAT v17.10 Unauthorized copying or reuse of any part of this page is illegal. 197
(SAT Teacher Edition)
CONJUNCTIONS
Coordinating conjunctions join two independent clauses. Make sure you memorize the seven coordinating
conjunctions. It's easy! Just remember FANBOYS.

For, And, Nor, But, Or, Yet, So

It's worth mentioning that a single clause that begins with a FANBOYS conjunction can technically stand
alone as a complete sentence. Professional authors will sometimes employ this technique, but you will never
see this construction on the SAT.

So, let's say you're trying to choose between two answer choices. One begins with a FANBOYS conjunction.
The other does not. Go with the one that does not. On the SAT, that's always the safer bet.

Subordinating conjunctions join an independent clause to a dependent clause. These are a bit trickier than
coordinating conjunctions because they create dependent clauses, which, as we've discussed, cannot stand
alone as complete sentences. As a result, many teachers discourage students from starting sentences with
words like because. In reality, they're being overly cautious, and they're encouraging students to become
boring writers. They're afraid that students will just stick because on to the front of an independent clause,
creating a sentence fragment.

Complete Sentence: Ernesto had learned to swim.


Fragment: Because Ernesto had learned to swim.
Complete Sentence: Because Ernesto had learned to swim, he was no longer nervous in the water.
There are lots of subordinating conjunctions, and they all work the same way. Any one of the following words
can be used to start a sentence, as long as the sentence contains an independent clause in addition to the
dependent clause created by the addition of the subordinating conjunction.

after if that where


although in order to though whereas
as once unless whether
because rather than until while
before since when why
even though so that whenever

Sentence: In order to be allowed in the deep end of the pool, children must pass a swimming test.
Sentence: Because Ernesto had learned to swim, he was allowed to go in the deep end of the pool.
Sentence: Though Ernesto later forgot how to swim, he will always have his memories of that afternoon.

198
Sentences & Fragments

SAT v17.10 Unauthorized copying or reuse of any part of this page is illegal. 199
(SAT Teacher Edition)
200
Run-On Sentences

SAT v17.10 Unauthorized copying or reuse of any part of this page is illegal. 201
(SAT Teacher Edition)
202
RUN-ON SENTENCE ERROR:
Two or more complete ideas are jammed together into one
sentence without the proper conjunction or punctuation mark.

Wrong: Mom knitted me a hat I wore it to prom.


Right: Mom knitted me a hat, and I wore it to prom.

TIPS AND TRICKS FOR RUN-ONS


When you see an underlined comma, check to see if the comma combines two independent
clauses (complete ideas). If it does, then the correct answer must repair the run-on using one of
the following three methods:

“Un-complete” one of the two complete ideas (create a dependent clause).


Add a FANBOYS conjunction: For, And, Nor, But, Or, Yet, So
Replace the comma with a semicolon.
Add a subordinating conjunction.

“Un-complete” an idea:
Wrong: Aaron has a sports car, he drives too fast.
Right: Aaron has a sports car, which he drives too fast.

Add a FANBOYS conjunction:


Wrong: Carla is really funny, she is one of my favorite people.
Right: Carla is really funny, so she is one of my favorite people.

Use a semicolon:
Wrong: I did not go to the party alone, Bill came with me.
Right: I did not go to the party alone; Bill came with me.

Add a subordinating conjunction:


Wrong: Ralph did not sleep last night, he was nervous.
Right: Ralph did not sleep last night because he was nervous.

SAT v17.10 Unauthorized copying or reuse of any part of this page is illegal. 203
(SAT Teacher Edition)
BASIC SKILLS PRACTICE
Repair the following run-on sentences using the methods you’ve learned.

1 The Volga River is over 2300 miles long, it is the longest river in Europe.
The Volga River is over 2300 miles long and the longest river in Europe.
2 In his lifetime, Thomas Edison registered 1093 patents, these included one for a flying machine.
In his lifetime, Thomas Edison registered 1093 patents, including one for a flying machine.
3 Nicholas scored 22 points, Shauna only had 15 points, she did have 11 rebounds and 2 steals, however.
Nicholas scored 22 points; Shauna only had 15 points, but she did have 11 rebounds and 2 steals.
4 Poodles are fragile and pet owners need to realize this and they need to be careful.
Poodles are fragile, so pet owners need to realize this and be careful.

A VERY POWERFUL TOOL FOR THE PROCESS OF ELIMINATION


As we mentioned earlier, you can separate two independent clauses with a semicolon, period, or
a comma + FANBOYS conjunction.
Now, because that's true, these three "separators" are essentially equal in the eyes of the SAT.
This presents a problem for test makers. You see, if any one of these methods would resolve the
error in a given sentence, then so could the other two!
How does that help?
Well, if you see more than one of them in the answer choices, then they must be incorrect!

Lisa and Deb finished the triathlon exhausted, she collapsed from exhaustion.
A) exhausted; she
B) exhausted. She
C) exhausted, and she
D) exhausted; Deb

Think about it. There cannot be more than one correct answer, and they would all be correct if
one of them is correct; therefore, you can safely eliminate all of them! In the above example, that
just leaves us with answer choice D.
There is one exception to this rule, which occurs when dealing with however and therefore at the
beginning of a clause. Those two often follow a semicolon rather than a period, though both are
technically correct.
That's a powerful tool for process of elimination!

204
Run-On Sentences

SAT v17.10 Unauthorized copying or reuse of any part of this page is illegal. 205
(SAT Teacher Edition)
206
Colons, Dashes, and
Semicolons

SAT v17.10 Unauthorized copying or reuse of any part of this page is illegal. 207
(SAT Teacher Edition)
DON'T USE A COLON UNLESS...
it follows an independent clause and introduces a list, explanation, short phrase, or examples.
List: Mitch has several cars: a Ford Pinto, a Volkswagen Beetle, and a Pontiac Aztek.
Explanation: The Pontiac Aztek is his preferred vehicle in every way except one: aesthetics.
Short Phrase: Mitch only cares about one thing: its beautiful custom paint job.
Example: The car was slammed by Mitch's wife: she called it ugly.

A DASHING GRAMMAR LESSON ABOUT DASHES


The dash — the bow tie of style — stands out. It adds a non-essential clause — and may inject surprise,
irony or a break in syntax. Sometimes — check out The New York Times — writers employ dashes to insert
a sentence within a sentence. A dash or two also excel for lists of items. If you see a dash, think not only
about punctuation but also about subject-verb agreement: remember that a non-essential clause cannot
be the subject of the sentence. Here are examples of dashes, all correct except the third one.

As easy as 1, 2 — 1. Adding to the beginning (1 dash), middle (2 dashes) or end (1 dash) of a sentence.
Burj Khalifa and Tokyo Skytree — each of these structures was built recently. (Core: “Each was built.”)
Use two dashes — not a dash and a comma — when the non-essential clause is in the middle.

Incorrect: Use two dashes — not a dash and a comma, when the non-essential clause is in the middle.
Burj Khalifa and Tokyo Skytree were built in this millennium — in Dubai and Tokyo, respectively.
(The final comma and “respectively” are part of the ending, non-essential clause, so one dash is
correct).

Indicating an element of surprise. You won’t likely be tested on the nuances below, but the examples
can help you remember the uses of a dash — because dashing to the library isn’t your top vacation idea.
She wanted one thing out of her vacation — to dash to the library. (Surprising!)
She wanted one thing out of her vacation: relaxation. (Expected, so a colon is appropriate)

Inserting an independent clause in the beginning, middle or end of a sentence


He apologized for offending me — but he continued to do so. A comma would work, too.

Separating a list that contains commas


The cities on the tour — Topeka, Tallahassee, and Tarzana — all begin with the letter T.
(Commas, instead of dashes, would be grammatically correct but confusing.)

SEMICOLONS JOIN COMPLETE IDEAS THAT ARE CLOSELY RELATED


Semicolons are covered in depth elsewhere in this book, but they're used to join two independent
clauses (complete ideas) without a FANBOYS conjunction. Semicolons are only used when the ideas
they separate are closely related.

As a rule of thumb, remember that a period = semicolon = comma + FANBOYS conjunction.

208
Colons, Dashes, and
Semicolons

SAT v17.10 Unauthorized copying or reuse of any part of this page is illegal. 209
(SAT Teacher Edition)
210
Commas and
Non-Essential Clauses

SAT v17.10 Unauthorized copying or reuse of any part of this page is illegal. 211
(SAT Teacher Edition)
ESSENTIAL AND NON-ESSENTIAL CLAUSES

Non-essential words and phrases can be removed from the sentence without affecting the
essential meaning of the sentence. These words can be eliminated without interfering with the
meaning of the core sentence.

Correct: Alan Sheppard, who was the oldest astronaut in the program,
commanded the Apollo 14 mission.

The above sentence contains a non-essential clause that's surrounded by commas.

Non-essential clauses often begin with one of these words: who, whose, which, or where. If you
spot one of those words or what you think may be a non-essential clause, take a second to
check whether the clause is essential to the meaning of the core sentence.

Consider where in the sentence the clause should begin and end. Often you'll be asked to
choose among answer choices that will determine the end of a non-essential clause. Where
you choose to place the comma will decide whether the non-essential clause is correctly
"set off " from the rest of the core sentence.

NON-ESSENTIAL CLAUSES MUST BE SURROUNDED BY COMMAS.

Incorrect: Alan Sheppard who was the oldest astronaut in the program
commanded the Apollo 14 mission. (no commas)

Incorrect: Alan Sheppard who was the oldest astronaut in the program,
commanded the Apollo 14 mission. (no comma before)

Incorrect: Alan Sheppard, who was the oldest astronaut in the program
commanded the Apollo 14 mission.

Core Sentence: Alan Sheppard commanded the Apollo 14 mission.

Talk with your tutor about specific strategies. You may also want to experiment with using your
pencil to draw a line through non-essential clauses. Because crossing out non-essential clauses
can help you visualize where punctuation (mainly commas) needs to be placed, this technique
will help you to identify the core sentences in the passage. Don't scribble or write any darker
than necessary. The goal is not to make the words illegible. You're just trying to help yourself
see what's going on in the sentence.

212
BE CAREFUL WITH TRANSITION WORDS
As you're probably aware, you can start a sentence with a transition word.

Correct: However, Jim Lovell is one of only three men to travel to the Moon twice.

The word however, can also be placed later in the sentence to create a non-essential clause.

Correct: Jim Lovell, however, is one of only three men to travel to the Moon twice.

The commas surrounding however tell us that the sentence will still make sense if we remove
the word however.

Core sentence: Jim Lovell is one of only three men to travel to the Moon twice.

You've got to be careful with the double commas, so you don't make it look as if an essential
clause is non-essential.

Incorrect: Jim Lovell is one of only three men to travel to the Moon twice, however,
he never set foot on it.

You've also got to be careful with the double commas so you don't mistake an essential clause
for a non-essential clause. If we cross out the clause we think may be non-essential, it becomes
really obvious that something is wrong.

Incorrect: Jim Lovell is one of only three men to travel to the Moon twice, however,
he never set foot on it.

That leaves us with two independent clauses joined without any punctuation to join or
separate them.

If you think you're dealing with a non-essential clause, cross it out. If the remaining sentence
functions, then you've got a non-essential clause. If it does not make sense, or a comma splice
is created, then you need to use a semicolon or period between the ideas.

TWO COMMAS DO NOT ALWAYS INDICATE A NON-ESSENTIAL CLAUSE

Don't assume that every bunch of words surrounded by a pair of commas is non-essential.

Sentence #1: Released in 1995, Apollo 13 stars Tom Hanks as Jim Lovell, and was
(NOT non-essential) directed by Ron Howard.

Sentence #2: Released in 1995, Apollo 13, which stars Tom Hanks as Jim Lovell, was
(non-essential) directed by Ron Howard.

Core Sentence: Apollo 13 was directed by Ron Howard.

SAT v17.10 Unauthorized copying or reuse of any part of this page is illegal. 213
(SAT Teacher Edition)
ESSENTIAL AND NON-ESSENTIAL NAMES AND TITLES

When a name or title appears in the middle of a sentence (not first or last), then you've really only
got two options: two commas or none at all.

Correct: Ron Howard, director of Arrested Development, got his start on The Andy
Griffith Show.

The above sentence contains a non-essential clause that's surrounded by commas. If we remove
everything between the commas, we'd be left with:

Core sentence: Ron Howard got his start on The Andy Griffith Show.

The best way to determine whether the commas are necessary is to follow the same technique
we used earlier: cross out the word or clause and see if the sentence makes sense without it. If the
sentence still makes sense, then the word or phrase is non-essential and we do need the commas.
If the sentence no longer makes sense without the word or clause, then we do not need the
commas.

Incorrect: Tom Hanks and his pal, Ron Howard teamed up again on The Da Vinci Code.

Incorrect: Tom Hanks and his pal Ron Howard, teamed up again on The Da Vinci Code

Correct: (okay) Tom Hanks and his pal, Ron Howard, teamed up again on The Da Vinci Code.

Correct: (better) Tom Hanks and his pal Ron Howard teamed up again on The Da Vinci Code.

Because the name Ron Howard appears in the middle of the sentence, we must choose whether
to use two commas or none at all. We can't go half way and use a single comma before or after
the name Ron Howard.

This is where it gets a little tricky. If we do use two commas, then the sentence seems to say that
Tom Hanks has only one pal, Ron Howard. That may be grammatically correct, but it's pretty
unlikely that the sentence intends that meaning. If we use no commas at all, then the sentence
makes more sense because it means that Tom Hanks has a number of pals, one of whom is named
Ron Howard.

Incorrect: Celebrated filmmaker, Steven Spielberg is another frequent collaborator.

Incorrect: Celebrated filmmaker, Steven Spielberg, is another frequent collaborator.

The first sentence is wrong because the name occurs in the middle of the sentence, which means
we need either two commas or none at all. The second sentence leaves us with an ungrammatical
sentence if we treat the name as an non-essential clause and cross it out. Thus, we'll have to go
without commas.

Correct: Celebrated filmmaker Steven Spielberg is another frequent collaborator.

214
Commas and
Non-Essential Clauses

SAT v17.10 Unauthorized copying or reuse of any part of this page is illegal. 215
(SAT Teacher Edition)
216
Possessives, Plurals,
and Contractions

SAT v17.10 Unauthorized copying or reuse of any part of this page is illegal. 217
(SAT Teacher Edition)
POSSESSIVE, PLURAL, AND
CONTRACTION WITH A VERB
This error occurs whenever a possessive is used
in place of a plural noun or vice versa.
A variation on this error occurs when a
contraction with a verb is used in place of
either a plural noun or a possessive.

HOW TO CREATE PLURAL NOUNS AND POSSESSIVE NOUNS


To form a plural of a noun, add -s (or -es if the singular noun ends in -s).

Correct: The views are breathtaking. (more than one view)


Correct: The mob bosses met to discuss which candidate to endorse. (more than one mob boss)

DO NOT attempt to create a plural noun by adding an apostrophe. It won't work!

Incorrect: The view's are breathtaking.


Incorrect: The mob boss's met to discuss which candidate to endorse.

To form the possessive of a singular noun, add -'s (even if the singular noun ends in -s.)

Correct: The doctor's stethoscope is missing.


Correct: The hippopotamus's birthday party begins at 4:00.

To form the possessive of a plural noun, add -s' (or -es' if the singular version ends in -s.)

Correct: Many of the mothers' recommendations to the city council were ignored.
Correct: The family of walruses' trip to Alaska got off to a strange start.

PRONOUNS: POSSESSIVE, PLURAL,


AND CONTRACTION WITH A VERB
These concepts are tested very frequently on the SAT, so
make sure you show up prepared to correct this type of
error.

Seriously, you should expect to see two - or even three - of


these on your SAT test, so put in the time!

218
ITS VS IT'S
Its = possessive of it
Have you seen the size of its tentacles?!

It's = it is
It's the most wonderful time of the year.

Its' = not a word (The plural of its is their).


Don't use its' when you mean to use their.

THERE VS THEIR VS THEY'RE


There = a place.
Nashville is nice, but I've never been there.

Their = possessive of they.


The bandits forgot to wear their masks.

They're = they are


Try the steak and potatoes; they're delicious.

WHOSE VS WHO'S
Whose = the possessive of who.
Whose running shoes are these?

Who's = who is.


Who's responsible for this disaster?

SAT v17.10 Unauthorized copying or reuse of any part of this page is illegal. 219
(SAT Teacher Edition)
Possessives, Plurals,
and Contractions

220
SAT v17.10 Unauthorized copying or reuse of any part of this page is illegal. 221
(SAT Teacher Edition)
Writing Unit #3
Homework Drill #1

222
SAT v17.10 Unauthorized copying or reuse of any part of this page is illegal. 223
(SAT Teacher Edition)
224
Writing Unit #3
Homework Drill #2

SAT v17.10 Unauthorized copying or reuse of any part of this page is illegal. 225
(SAT Teacher Edition)
226
SAT v17.10 Unauthorized copying or reuse of any part of this page is illegal. 227
(SAT Teacher Edition)
Writing Unit #3
Homework Drill #3

228
SAT v17.10 Unauthorized copying or reuse of any part of this page is illegal. 229
(SAT Teacher Edition)
230
4 Informational Graphics 232

Logical Transitions 241

Relevance: Adding & Removing Info 247


Writing Unit #4

Reordering a Sentence
or Paragraph 253

Homework Drills:

1 2 3
260 263 266

SAT v17.10 Unauthorized copying or reuse of any part of this page is illegal. 231
(SAT Teacher Edition)
Informational Graphics

232
SAT v17.10 Unauthorized copying or reuse of any part of this page is illegal. 233
(SAT Teacher Edition)
BAR CHA
RTS:
ts are
Bar Charts
Bar char WORK
tool Y:
a great ATICALL
hting SYSTEM he
for highlig e g in b y reading t
values, B t or
specific t to righ
han A chart lef his
rather t ottom. T
top to b
trends.
100 ou make
will help y s

VLDL Concentration (mg/dl)


don't mis
sure you ails
rtant det
75 any impo
IS on this nships
THE Y-AX at or relatio .
t begins variables
bar char ul between
be caref 50
zero, but ts
bar char
because a rt
ve out p
often lea ey
25
xis so th
of the a ES:
us more GRID LIN
can foc er that t
he
on the 0 Rememb
narrowly 0 0 100 500 are ther
e
portion. gridlines and
relevant WT Min ou read
to help y t.
Dose of NF-19 (ppm) e r p r e t the char
in t
B
D DATA
GROUPE :
40
CHARTS
ON BAR
LDL Concentration (mg/dl)

k
er to loo 500 ppm
Rememb o ng 30
0-1
ends am
Polyp volume (mm3)

at the tr
100 ppm
ithin an
groups w 1.1-2 0 ppm
nt.
experime 20
2.1-3
the
ample to
In the ex roup 10
0 ppm g
right, the
3.1-4
mors of
had 18 tu3 while the
,
0 - 1 mm
0 4.1+
g r o u p had 0 0 100 500
500ppm
this size. WT 0 10 Min 20
only 2 of 30 40
Dose of NF-19 (ppm)
Number of tumors
C
100
TION
COMBINA
HDL Concentration (mg/dl)

100 5000
:
GRAPHS
Population Size (% max)

75
Dioxin Load (ppt)

80 40 4500
sets are
Two data te 100
n separa
plotted o 50
60 4000 Population Size
ne on the
Number of polyps

30
y-axes, o he Dioxin Load
75 0 ppm
% Total Cholesterol

one on t
left and 40
25 3500 100 ppm
right. 20
500 ppm
20 3000 50
to note
Be sure
0
is goes
0 10 0 100 500
which ax t! 0 2500
h datase WT Min 25
with whic
es

ls
s
its

xe

Dose of NF-19 (ppm)


w
ol

0
bb

Fo

O
V

0-1 1.1-2 2.1-3 3.1-4 4.1+


Ra

234
0
0 0
Figure 1
Stacked Bar and Pie Charts
Parts of a Whole
BARS:
A
• Elements of Stacked Bar and Pie STACKED p all the
b a r s show u
charts ALWAYS add to 100%. Stac k e d ke sure
t h e S A T, so ma
time on hem.
h o w t o read t
you know
ine ac-
x p e r iment, ur gen
In th is e
0 %. o f the nitro
or 8
counts f , f e c es make
up
m p le Key es up
in the sa w ater mak
g r e y
10 ., and
% Urine
ining 10 . %
the rema Feces
Greywater
Car Exhaust
Road Wear
SAME Average
100 2.2 8.1 16.1 76.6 100
DATA % Total from source

% Total from source


80 80

60 60

Key 40 40
Urine
Feces Key
Greywater 20 20
Car Exhaust Combined
Road Wear Separated
0 0
100 100 100 100
ad
en

nc

H
s

B
ou

PA

PC
Le
og

Zi
% Total from source

% Total from source

or

80 80 80 80
Material released (kg)itr

ph

Material released (kg)


N

os
Ph

60 60 60 60
Figure 1
40 40 40 40

20Plan to see two infographics


20 on the Reading
20 Test. 20
First, take a moment to familiarize yourself with the data presented in the
0 0 0 0
infographic. As you do, pay close attention to any axis labels or keys.
ad

ad
en

nc

en
H

nc
s

H
s
B

B
ou

ou
PA

PA
PC

PC
Le

Le
og

Zi

og

Zi
or

or
itr

itr
ph

ph
N

Next, consider how the data presented in the infographic relates to the
os

os
Ph

Ph

informationFigure
in the1 passage. Sometimes an infographic and the passage
Figure 2 it's
Table 1
paired with are barely related. Other times, the data in the infographic is directly
referenced in the passage.
Chemical in Effect on human health or
In these cases, you'll often see a question with a set of claims in the
wastewater the answer
environment
choices. The question will ask you which claim is best supported by the data in
Nutrients Algae overgrowth,
the chart, graph, or table.
(Nitrogen, Phosohorous) death of fish
Table 1
Heavy metals Human cancers,
Chemical in Effect on human health or (Lead, Zinc) birth disorders
wastewater the environment
Nutrients
Aromatic
Algae overgrowth,
(Nitrogen,
SAT v17.10 Phosohorous) death of fish hydrocarbons
Unauthorized copying or Skin
reuse of any part of this page is illegal. and nerve disorders 235
Heavy metals Human cancers, (PAH, PCB)
(SAT Teacher Edition)

(Lead, Zinc) birth disorders


Tables
W:
TION RO
LE TITLE
: INFORMA ave
THE TAB ble will h
en't Every ta
These ar column
seful, a row or
always u (or both
) with
're ion.
but they Table 2 informat
orth essential
always w
reading! Mass Mass of
of sodium Initial Final
water hydroxide temperature temperature
Cup (g) (g) (°C) (°C) q (J)
1 100 2 22 25 1200
BONUS 2 100 4 22.5 28.5
TION: UNITS OF
INFORMA 3 100 6 23 32 5400
bles EMENT:
Some ta 4 100 8 23 35 MEASUR
xtra ut for
include e Watch o
Example (Cup 1): q = 100g*(4 J/g°C)*3°C = 1200 J that use
info that answers
find g units
you may the wron
e this. urement.
helpful, lik of meas

Table Strategies
• Note which values do not change. "Mass of water" and "Initial Temperature" don't vary
(much). Feel free to cross out information that doesn't help you understand what's
going on.
• Look for patterns in the data. "Mass of sodium hydroxide" changes in an orderly way.
It is probably the independent variable in this experiment. Both "Final temperature"
and "q" seem directly related to it. Why?

Solubility of Ionic Compounds


A
TIVE DAT
QUALITA n in Ca2+ Fe3+ Ag+ Pb2+
be show
can also a t
his form F- White : Soluble
tables. T ons
p lifi e s comparis
sim of Cl- Black : Not
not determined
Determined
elements
between
t. Light Grey : Slightly Soluble
a datase Br-
Dark Grey : Insoluble
I-
OH-
SO42-
CO32-

236
Table-Graph Conversions

Simplify the table whenever you can.


Tables often contain excess information that isn't required for most problems. Look for
the important information. Then consider making a quick graph to see relationships.

Table 1 Table 2
Brightness (lumens) Time until breakage (seconds)
Filament Table 1 Filament Table 2
Diameter 0.3 mm 0.5Brightness
mm (lumens)
0.7 mm 0.9 mm Diameter Time 0.3until
mm 0.5 mm 0.7 mm
breakage (seconds)
0.9 mm
Trial 1Filament 85 65 32 10 Filament
Trial 1 2.5 8.2 15.1 45.5
Trial 2Diameter 810.3 mm 0.5
65 mm 0.734
mm 0.9 mm8 Diameter
Trial0.32 mm 0.5
2.8mm 0.7 8.6 mm 0.9 mm 17.9 94.1
Trial 1 85 65 32 10 Trial 1 2.5 8.2 15.1 45.5
Trial 3 Trial 2 86 81 6465 38
34 8
10 Trial 2
Trial 32.8 0.2
8.6
7.9
17.9 94.1
14.0 78.0
Trial 4 Trial 3 83 86 6264 31
38 10 10 Trial 3Trial 40.2 3.0
7.9 7.8
14.0 78.018.2 107
Trial 5 Trial 4 85 83 6562 33
31 10 9 Trial 4Trial 53.0 2.6
7.8 8.0
18.2 10715.2 58.8
Trial 5 85 65 33 9 Trial 5 2.6 8.0 15.2 58.8
Average 84 64 34 9 Average 2.2 8.1 16.1 76.6
Average 84 64 34 9 Average 2.2 8.1 16.1 76.6

e
to visualiz
Attempt a
sketch
or even ake
c h art to m it clear
b a s ic
understa
nd g r a p h makes
T h e able
sure you the d a t a from T
ionships in that t h e
(i.e. linear
ly)
the relat ir e c t ly
1 are d rom
table.
t e d , w h ile data f
rela
re not.
the table
s: Table 2 a
Compare ve
e they ha
Looks lik
Filament
the same
and both
Diameter ge
an avera
also have
he end.
row at t
DIRECTLY (LINEARLY) RELATED (ADJ):
The value of y is a MULTIPLE OF x, basically. Put into algebra, it
looks like y = kx, where k is any number. The slope can go up or
down, but the plot is always a straight line.

INVERSELY RELATED (ADJ):


The value of y is some number DIVIDED BY x, basically.
Algebraically, this means y = k/x, where k is any number. With
inversely related data, y rises as x falls, and falls as it rises.
The plot is never a straight line.

SAT v17.10 Unauthorized copying or reuse of any part of this page is illegal. 237
(SAT Teacher Edition)
Extrapolate/Interpolate
t math
w o r d s are jus
Thes e AT is
A s f a r as the S
jargon. need to
r n e d , you just EXTRAPOLATE (V):
conc e data
r a t r e nd in the
look fo e your
Infer an unknown value based on trends in data. When
fig u r e out wher extrapolating, the unknown value lies outside the range of the
and
uld be. dataset.
value wo
INTERPOLATE (V):
Infer an unknown value based on trends in the data. This is
exactly like extrapolating, except that the unknown value lies
within the range of data that you have.

LATION: w could
EXTRAPO a t t e rplot, ho
s c
Given this ntration
t im a t e t he conce
you es r?
Cation
0 m u n der wate Anion
10
of X at 104
103 of
entration
Concentration of X

The conc 103 and 10 .


4

w e e n
102 X is bet apolated!
Cation extrAnion
A 101 You just
1040 104
10 Sodium
Potassium 10-13 103
Concentration of X

10
1 10 100 1020.1 1 10 100 102
Meters under water
10 1 101
s a r e r 10in4g.
is
lue
um The y-va ce
n 03traChloride
e n d t h e line, the10 10 e
100
Ex t the lin
m up to10-1 Potassium
from 100 h e y -a x2is. 10-1
r t o t 10
and ove 0.1 1 10 100 0.1 1 10 100
B 1 Meters under water
10
1040 104
10 Magnesium Chloride
10-13 103
r the
iq u es 10 Solubility (g per 100 ml water) s t look ove
e t e c h n
10 Sur e ! F ir s info.
te exces
1 Thes10 100 20.1 1 10 100 2
t o o. 10 in a
meq/L

a b le s o e lim
work in
t Cerium table t mns 2-4
s t im ate n ig n ore colu
e a olumn
1 Sodium (IV) Trisodium 1
Can yo u 10 10 You c
ead on c
10 4
ili t y o f Chloride Sulfate Phosphate Sucrose c u s in s t
the solub fo
2C? 10 0 Carbonate 100 and
t 8 crose.
3 NaCl Ce(SO4)2 Na3PO4 C12H22O11
s e a 10
5, s u
sucro Temp. (g*) (g*) (g*) (g*) in
e trend
10 -1 10-1
1 ok for10th
102
0.1
0°C 36.01 21.410 1.50100 182 0.1 Then lo o .
100 s e
r a t u r e and sucr
tempe
C 1
1020°C 35.8 9.84 8.80 202
1020 104 ate.
1060°C 35.9 3.87 20.9 289
m a k e y our estim
Then , an
olation is
Calcium Carbonate
103
ll in t e r p
That's a
100°C 36.0 0.013 77.0 476
10-1
1010.1 .
1 10 100 1 10 100 102
estimate
meq/L

Salinity (ppt
238 TDS) Salinity (ppt TDS)
101
Figure 1100
100
Informational Graphics

SAT v17.10 Unauthorized copying or reuse of any part of this page is illegal. 239
(SAT Teacher Edition)
Trending Baby Names: 2013-2014
Rank Name 2013 2014 Percent Change
1 Daleyza 485 1461 201.2
2 Gannon 253 747 195.3
3 Everly 812 1854 128.3
4 Paislee 446 915 105.2
5 Elsa 564 1131 100.5
6 Nova 562 1121 99.5
7 Karter 1174 2103 79.1
8 Amina 446 776 74.0
9 Milan 486 748 53.9
10 Ace 545 818 50.1
11 River 827 1223 47.9
12 Cali 1121 1652 47.4
13 August 1039 1520 46.3
14 Finley 569 829 45.7
15 Nash 597 867 45.2
16 Juniper 437 633 44.9
17 Maverick 1294 1862 43.9
18 Kali 867 1247 43.8
19 Rhett 688 975 41.7
20 Hazel 2049 2877 40.4
21 Thiago 561 791 41.0
22 Harvey 402 566 40.8
23 Kayson 397 699 40.6
24 Angelique 548 768 40.1
25 Ember 522 729 39.7
26 Sawyer 685 955 39.4
27 Finley 1093 1498 37.1
28 Adalynn 1841 2512 36.4
29 Arya 1137 1544 35.8
30 Evelynn 539 731 35.6

male name
female name

240
Logical Transitions

SAT v17.10 Unauthorized copying or reuse of any part of this page is illegal. 241
(SAT Teacher Edition)
242
MAKE SURE YOU KNOW HOW TO USE THESE TRANSITION WORDS:

Additive:
In fact Likewise In other words Such as
Indeed Similarly That is For example
Furthermore For Instance
What is more
Additionally

Adversative:
Despite this Although
Even so But
Still While
Nevertheless Whereas

Cause & Effect:


Accordingly
Consequently
Therefore
As a result
Thus
Hence

Sequential:
Subsequently
Thereafter
Meanwhile
Concurrently
At the same time

SAT v17.10 Unauthorized copying or reuse of any part of this page is illegal. 243
(SAT Teacher Edition)
QUESTIONS INVOLVING TRANSITIONS ARE VERY COMMON ON THE NEW SAT

You should expect to see at least five questions testing your ability to choose the word, phrase, or sentence
that creates the smoothest, most logical transition between two elements in a passage.

You'll be asked to do each of the following:

1) Create a transition between two parts of the same sentence.


2) Create a transition between two sentences.
3) Create a transition between two paragraphs.

Your job is simple: Choose the word, phrase, or sentence that clarifies the relationship between the two
elements you've been asked to connect.

This means that you'll first need to identify the relationship between the two elements, so you'll know
what type of word or phrase you're looking for as your answer. For the purposes of SAT English passages,
there are only a few types of relationships between ideas:

Additive: It's important to childproof your home by storing cleaning supplies out of the reach of
children. Furthermore, you should remove all chain saws.

Adversative: Do not feed the bears. However, if they look really cute, then go for it!

Cause & Effect: Rob challenged the Green Knight to a duel. As a result, Rob is no longer with us.

Sequential: Joe ate two tubes of toothpaste. Subsequently, he had fresh breath and a hospital bill.

Remember, this is a two step process. First, you must identify the relationship between the two ideas.
Second, you must choose the answer choice that best establishes the relationship you identified in step 1.

Because the transition that has been used in the passage is usually wrong (and often misleading), it's a
great idea to physically cross out the existing transition, so you're not tempted to "make it work."

A CONCLUSION IS ANOTHER FORM OF TRANSITION.

Transitions tie two parts of the passage together. A conclusion is very much the same, except that it ties
up the passage as a whole. When you're asked to choose the best concluding sentence, make sure that
you consider the passage as a whole, including the tone of the passage. A concluding sentence should
not introduce new information or contradict the rest of the passage, so eliminate any answer choices that
bring in new topics or try to expand the scope of the passage.

244
Logical Transitions

SAT v17.10 Unauthorized copying or reuse of any part of this page is illegal. 245
(SAT Teacher Edition)
246
Relevance: Adding &
Removing Info

SAT v17.10 Unauthorized copying or reuse of any part of this page is illegal. 247
(SAT Teacher Edition)
248
WHEN ADDING OR REMOVING INFORMATION, STAY ON TOPIC!
You should expect to see at least three or four questions asking whether sentences or phrases should be
changed, added, or deleted. These are usually two-part questions because the SAT tends to ask not only
whether an action should be taken but also why the action should be taken.

The best way to attack these questions is by breaking them in half. First, consider whether the information
should be added at all. It's okay to use your gut instincts on this one. If the sentence feels out of place,
then don't add it. If the sentence feels like it adds relevant information, then add the sentence.

Now, you're only half done at this point, but the hardest part is over. You've eliminated two answer choices
just by deciding whether or not to add the sentence. Before you read the answer choices, try to answer
the question in your own words. Why should/shouldn't this sentence be added?

Then, select the answer choice that's closest to the answer you came up with in your own words. If you're
stuck, then read the two remaining answer choices carefully and focus on the differences between them. If
you're still stuck, take a guess and move on.

If you feel the sentence should not be added, consider an answer choice that says something like because
it distracts from the main focus of the paragraph. That's another way of saying, it's irrelevant!

Keep in mind that some sentences just don't belong in some passages. If you really think a sentence is
not directly relevant to the essay, then you should confidently choose the answer choice that says "OMIT
the underlined portion" or "DELETE the underlined portion." Often, a sentence will simply add extra
information that's related to a person or thing in the passage, but not directly related to the focus of the
passage.

Let's consider an example. Suppose you're looking at a passage about the end of Babe Ruth's career, and
a question asks you whether or not to add a sentence about baseball in general: There is some dispute as
to who actually invented the game of baseball. It's related because Babe Ruth played baseball, but it's not
relevant to an essay about the end of his career as a baseball player. The same would go for a sentence
about Babe Ruth's childhood. It's related, but not necessarily relevant. Know the difference.

NEED TO ADD EMPHASIS TO SOMETHING? START BY UNDERLINING IT.


Some SAT questions ask you to revise a sentence in order to better emphasize a point that the writer
is trying to make. Obviously, you can't just underline the part of the passage that you're supposed to
emphasize. That would be way too easy. This is the SAT, after all. That doesn't mean you can't underline
parts of the question to help you make sure you're looking for the right answer.

So, when you see this type of question, begin by underlining the part of the question that tells you what to
emphasize. This will help you keep in mind that you're not necessarily looking for the revision that sounds
best; you're looking for the one that best emphasizes the information you've been asked to emphasize.

MAKE SURE YOU UNDERSTAND WHAT YOU'RE ADDING OR REMOVING.


Some SAT questions tell you that a sentence has been removed and it's your job to figure out what job that
sentence played when it was still part of the passage. Other questions will tell you that a sentence is being
added and you need to identify what role that sentence will play in the passage.

Sentences typically function as examples, transitions, explanations, or descriptions. Again, come up with
an answer in your head before you read the answer choices. Choose the option that's closest to the answer
you came up with in your head.

SAT v17.10 Unauthorized copying or reuse of any part of this page is illegal. 249
(SAT Teacher Edition)
250
Relevance: Adding &
Removing Info

SAT v17.10 Unauthorized copying or reuse of any part of this page is illegal. 251
(SAT Teacher Edition)
252
Reordering a
Sentence or Paragraph

SAT v17.10 Unauthorized copying or reuse of any part of this page is illegal. 253
(SAT Teacher Edition)
254
SAT v17.10 Unauthorized copying or reuse of any part of this page is illegal. 255
(SAT Teacher Edition)
PAY CLOSE ATTENTION WHEN YOU SEE THESE [ ].
When you see brackets (numbered paragraphs or sentences), pay very close attention to the order of the
paragraphs within the essay or the order of numbered sentences within a paragraph.

WHEN YOU'RE ASKED TO MOVE A SENTENCE...


Start by finding the topic sentence of that paragraph. If the topic sentence seems like it shows up a bit
late in the paragraph, then you should look for answer choices that move the topic sentence nearer to the
beginning of the paragraph. Choose the best available answer choice. Simple, right?

Next, consider the flow of the paragraph. Each sentence should follow logically from the sentence before
it. If you notice a break in the flow or a sentence that doesn't follow logically from the sentence before it,
then you've found the key to the problem. You'll either need to move one of the two sentences that don't
seem to flow together, or you'll need to place a third sentence between the two sentences that don't flow
together.

Keep an eye out for ambiguous pronouns that could be clarified by repositioned sentences. Also watch out
for cases where one sentence clarifies or expands upon something that doesn't get mentioned until later
in the passage. These are great signals that a sentence should be moved.

WHEN YOU'RE ASKED TO MOVE A PARAGRAPH...


Start by recapping the essay in your own words. What's it about? How does the writer make his or her
point? What function does each paragraph serve in making that point?

For example, an essay might break down like this:


Main Idea: Occasional forest fires are necessary for the survival of a forest.
Paragraph 1: Examples of fires that were very destructive.
Paragraph 2: Information about early conservation efforts and forest fire prevention.
Paragraph 3: Information about how those efforts actually made the fires much worse.
Paragraph 4: Examples of forests that have thrived after major fires.
Paragraph 5: Conclusion: Today, we know that forests need occasional fires.

Now, reread the paragraph you're asked whether to move. Pay close attention to its relationship to the
paragraph before it, the paragraph after it, and the essay as a whole. It's a very good idea to reread the last
sentence of the paragraph before it. Does it flow into the paragraph you've been asked whether to move?

There's one more trick that may come in handy here. Identify the topic sentence of the paragraph you've
been asked to move. Quickly skim the other paragraphs to see if that same topic is mentioned elsewhere
in the passage. If you find that topic elsewhere, and if you've got the option to place the two paragraphs
adjacent to one another, then do so.

WHEN YOU'RE ASKED TO SPLIT ONE PARAGRAPH INTO TWO PARAGRAPHS...


Start by reading the paragraph carefully. As you do, watch for any point at which the paragraph changes
direction by introducing new ideas or phenomena. Wherever you feel the writer has strayed from the
original topic sentence of the paragraph by introducing a new topic sentence, that's where you should
separate the paragraphs.

Alternatively, you can work backward from the answer choices, plugging each one in to see which one
gives you the cleanest break between the two newly formed paragraphs. Be aware that this method can
be slower.

256
Reordering a
Sentence or Paragraph

SAT v17.10 Unauthorized copying or reuse of any part of this page is illegal. 257
(SAT Teacher Edition)
258
SAT v17.10 Unauthorized copying or reuse of any part of this page is illegal. 259
(SAT Teacher Edition)
Writing Unit #4
Homework Drill #1

260
SAT v17.10 Unauthorized copying or reuse of any part of this page is illegal. 261
(SAT Teacher Edition)
262
Writing Unit #4
Homework Drill #2

SAT v17.10 Unauthorized copying or reuse of any part of this page is illegal. 263
(SAT Teacher Edition)
264
SAT v17.10 Unauthorized copying or reuse of any part of this page is illegal. 265
(SAT Teacher Edition)
Writing Unit #4
Homework Drill #3

266
SAT v17.10 Unauthorized copying or reuse of any part of this page is illegal. 267
(SAT Teacher Edition)
268
5 Diction and Idioms 270

Parallelism and Word Pairs 279


Writing Unit #5

Misplaced Modifiers 282

Redundancy & Wordiness 285

Homework Drills:

1 2 3
290 293 296

SAT v17.10 Unauthorized copying or reuse of any part of this page is illegal. 269
(SAT Teacher Edition)
Diction and Idioms

270
SAT v17.10 Unauthorized copying or reuse of any part of this page is illegal. 271
(SAT Teacher Edition)
DICTION ERRORS
Misuse of a word in place of another word that is related in meaning or sound.
The SAT Writing Section commonly includes two or three questions that test your ability to recognize
when a word has been misused in the context of the passage. The following is a list of words, broken
into pairs and small groups of words that are commonly mistaken for one another. Most of these will
be well known to you, but a few might surprise you.
Read threw through this list to make sure your you're knot not cot caught unprepared on test day.

Accept (v.) to receive Ascent (n.) climb


Except (prep.) with the exclusion of Assent (n.) agreement

Access (n.) the ability to enter; (v.) to enter Assure (v.) to dispel doubts
Excess (n.) a surplus; (adj.) unnecessary Ensure (v.) to guarantee

Addition (n.) the act or process of adding Averse (adj.) having strong feelings of opposition
Edition (n.) one of a series Adverse (adj.) unfavorable or antagonistic

Adopt (v.) to take up, to begin using or following Capital (n.) location of govt; financial resources
Adapt (v.) to modify for use in new circumstances Capitol (n.) government headquarters

Advice (n.) counsel Cite (v.) to quote or document


Advise (v.) to give advice Sight (n.) vision
Site (n.) position or place
Affect (v.) to influence
Effect (n.) a result; (v.) to accomplish Collaborate (v.) to work together
Corroborate (v.) to provide evidence in support
Afflict (v.) to distress
Inflict (v.) to impose upon Complement (n.) something that completes
Compliment (n.) praise; (v.) to praise
A lot (n.) many
Alot (-.) This is not a word. Conscience (n.) sense of right and wrong
Allot (v.) to give out a specific amount Conscious (adj.) awake

Allude (v.) to make reference to Council (n.) a group that consults or advises
Elude (v.) to avoid or escape by speed Counsel (v.) to advise (n.) advice

Allusion (n.) an indirect reference Indiscreet (adj.) lacking discretion or tact


Illusion (n.) a false perception of reality Indiscrete (adj.) not divisible into separate parts

Anecdote (n.) an amusing story Disinterested (adj.) unbiased


Antidote (n.) medicine or remedy Uninterested (adj.) not interested

272
Descent (n.) the act of moving higher to lower Passed (v.) past tense of "to pass," to have moved
Dissent (n.) a difference of opinion; (v.) to disagree Past (n.) belonging to a former time or place

Device (n.) a machine Perspective (n.) point of view


Devise (v.) come up with Prospective (adj.) likely to happen in the future

Elicit (v.) to draw or bring out Precede (v.) to come before


Illicit (adj.) illegal Proceed (v.) to go forward

Eminent (adj.) famous, respected Precedent (n.) something that serves as a guide
Immanent (adj.) inherent or intrinsic President (n.) one who presides over a group
Imminent (adj.) ready to take place
Prescribe (v.) to designate or order the use of
Emit (v.) to give off Proscribe (v.) to denounce or condemn
Omit (v.) to leave out
Principal (adj.) most important (n.) the authority
Faze (v.) to disturb ones composure Principle (n.) a general or fundamental truth
Phase (n.) a stage in a process
Quote (v.) to cite
Flaunt (v.) to show off Quotation (n.) something that is quoted
Flout (v.) to treat with contempt
Respective (adj.) particular to one among a group
Gambit (n.) a careful strategy Respectful (adj.) showing politeness or deference
Gamut (n.) the complete range Respected (adj.) held in high esteem

Imply (v.) to indicate or suggest Stationary (adj.) standing still


Infer (v.) to derive by reasoning; conclude Stationery (n.) writing paper

Ingenious (adj.) brilliant; very smart Weather (n.) atmospheric conditions (v.) to endure
Ingenuous (adj.) candid, sincere Whether (conj.) if it be the case

Its (pronoun) of or belonging to it Than (prep., conj.) use with comparisons


It's (contraction) contraction for "it is" Then (adv.) at that time, or next

Lead (n.) a type of metal Their (pro.) possessive form of they


Led (v.) past tense of the verb "to lead" There (n.) a location
They're - contraction for "they are"
Lie (v.) to lie down; (v.) to give a false account
Lay (v.) (followed by a noun) lay an object down Through (adj.) by means of; finished; into or out of
Threw (v.) past tense of throw
Lose (v.) to misplace or not win Thorough (adj.) careful or complete
Loose (adj.) to not be tight Though (adv.), ( conj.) however; in spite of

Medal (n.) an award To (p.) toward


Meddle (v.) to interfere Too (adv.) also, or excessively
Metal (n.) a mineral substance Two (n.) (adj.) the number 2
Mettle (n.) fortitude, strength of character

SAT v17.10 Unauthorized copying or reuse of any part of this page is illegal. 273
(SAT Teacher Edition)
IDIOM ERROR
Misuse of an expression that does not mean what its literal definition suggests.
On the SAT, idioms are primarily tested on phrases that require a specific preposition to be correct.

Wrong: Yellow fever is still a threat of travelers in some countries.


Right: Yellow fever is still a threat to travelers in some countries.

TIPS FOR IDIOMS


Most idiom errors test your knowledge of preposition usage.
If you’re not sure which preposition goes with a particular
verb, try substituting the verb for a synonym. You may find
the answer is now obvious.
I am concerned about Steve. I’m worried about Steve.
I am skeptical of Steve. I’m suspicious of Steve.
I am grateful for Steve. I’m thankful for Steve.

As you can see, there are no set rules for idioms.


Unfortunately, you’ve just got to learn them one by one.

WRONG & RIGHT

Example:
Wrong: This week’s results are inconsistent to the results from last week.
Right: This week’s results are inconsistent with the results from last week.
Your Turn:
Wrong: Listening at the audio book, I hated the narrator’s voice.
Right: Listening to the audio book, I hated the narrator's voice.
__________________________________________________________

Wrong: I could tell that Vinny was preoccupied on the television as I spoke.
Right: I could tell that Vinny was preoccupied with the television as I spoke.
__________________________________________________________

274
SKILLS PRACTICE
Supply the correct preposition for each
of the following verbs.

1 to
able _______________ 26 to
indifferent _______________

2 from
absent _______________ 27 that
insist _______________

3 to
acceptable _______________ 28 to
manage _______________

4 to
accustomed _______________ 29 for
mistake _______________

5 with
acquainted _______________ 30 after
modeled _______________

6 at
adept _______________ 31 to
native _______________

7 with
associated_______________ 32 to
oblivious _______________

8 of
capable _______________ 33 in
originate _______________

9 with
collaborate _______________ 34 as
perceive _______________

10 that
conclude _______________ 35 to
predisposed _______________

11 of
conscious _______________ 36 to
preferable _______________

12 with
consistent _______________ 37 against
prejudiced _______________

13 to
contrary _______________ 38 as / by
prized _______________

14 with
credit _______________ 39 from
prohibit _______________

15 as
depict _______________ 40 against
protect _______________

16 from
different _______________ 41 as
regard _______________

17 from
distinguish _______________ 42 to
relavent _______________

18 as
emerge _______________ 43 for / to
responsible _______________

19 to
encourage _______________ 44 as
seen _______________

20 to
equivalent _______________ 45 to
sensitive _______________

21 to / for
essential _______________ 46 as / in
succeed _______________

22 with / to
familiar _______________ 47 of
suspicious _______________

23 by
fascinated _______________ 48 of
tolerant _______________

24 to / from
forbid _______________ 49 to
try _______________

25 to
inclined _______________ 50 as
view _______________

SAT v17.10 Unauthorized copying or reuse of any part of this page is illegal. 275
(SAT Teacher Edition)
Diction and Idioms

276
SAT v17.10 Unauthorized copying or reuse of any part of this page is illegal. 277
(SAT Teacher Edition)
278
Parallelism and Word Pairs

SAT v17.10 Unauthorized copying or reuse of any part of this page is illegal. 279
(SAT Teacher Edition)
PARALLELISM
The two most common parallelism errors on the SAT are:

1. things being compared


2. items on a list

PARALLELISM
Also, watch for words that travel in pairs, which sound weird if you start the
pair but don't finish it. For example:
as... as not only... but also
more... than less... than (never use THEN)
either... or neither... nor
from... to both... and
WRONG & RIGHT

Examples:
Wrong: Pete likes to run, jump, and swimming.
Right: Pete likes to run, jump and swim.

Your Turn:
Wrong: Jogging, lifting weights and the saxophone are my hobbies.
Right: Jogging, lifting weights and playing the saxophone are my hobbies.
_________________________________________________

BASIC SKILLS PRACTICE


In each sentence, underline the items requiring parallelism. Make sure they are all parallel.
When you find an item that breaks parallelism, cross it out, and write in your correction below the
sentence.

1 The exhaust pipe on my station wagon is rusted, broken, and is ready to fall out.
The exhaust pipe on my station wagon is rusted, broken, and ready to fall out.

2 The repairs called for new floorboards, new carpet, and also for wallpaper that had to be imported.
The repairs called for new floorboards, new carpet, and imported wallpaper.

3 Faulty parallelism and to make a faulty comparison are two commonly tested errors on the SAT.
Faulty parallelism and faulty comparisons are two commonly tested errors on the SAT.

4 My neighbor likes her coffee iced, blended, and served in a paper cup.
(correct)

5 Scrapbooking, writing poetry, and to pursue a career in film production are Jeff ’s main goals.
Scrapbooking, writing poetry, and pursuing a career in film production are Jeff ’s main goals.

280
Parallelism and Word Pairs

SAT v17.10 Unauthorized copying or reuse of any part of this page is illegal. 281
(SAT Teacher Edition)
Misplaced Modifier

282
MODIFIERS: LOCATION IS EVERYTHING
Modifiers modify what they are located closest to, even if it makes a silly or illogical sentence.
Your job is to edit the sentence by putting the misplaced modifier as close as possible to the things
they are intended to modify.

THINK LITERALLY
Pay special attention to exactly what the sentence
says, not what you think it’s trying to say.

FIXING A MISPLACED MODIFIER


Remember modifiers modify what they are next to even if it makes the sentence
nonsense. To fix a misplaced modifier, you may need to rearrange the sentence.

Example:
1 Wrong: Howling at the moon, Timmy saw the werewolf.

,
, Y ou m ig ht a ss u me it s
g?
So, who s hoolwf,linbu lly
w er ew t th is sentencethaectmuooa n!
the is howling at
says that Timmy
Right: Timmy saw the werewolf howling at the moon.
Your Turn:
2 Wrong: Trapped high in the tree, Lionel saw the cat.

Right: Lionell saw the cat trapped high in the tree.


____________________________________________

BASIC SKILLS PRACTICE


Underline the modifier and circle the word it modifies according to the sentence. Remember,
modifiers modify what they’re closest to, whether it’s logical or not.

1 Crossing the finish line, the crowd cheered for the athletes.
The crowd cheered for the athletes as they crossed the finish line.
2 Running for his life, Bigfoot chased Marvin out of the convenience store.
Running for his life, Marvin was chased out of the convenience store by Bigfoot.
3 Though he had lost a quite a lot of blood, the doctor announced that the patient would be okay.
Though the patient had lost quite a lot of blood, the doctor announced that the patient would be okay.
4 Though he was afraid of his own shadow, the Mayor introduced the groundhog to the crowd.
Though he was afraid of his own shadow, the groundhog was introduced to the crowd by the mayor.
5 Hot and fresh from the oven, I savored my Aunt Sal’s cookies.
I savored my Aunt Sal’s cookies, hot and fresh from the oven.

SAT v17.10 Unauthorized copying or reuse of any part of this page is illegal. 283
(SAT Teacher Edition)
Misplaced Modifier

284
Redundancy & Wordiness

SAT v17.10 Unauthorized copying or reuse of any part of this page is illegal. 285
(SAT Teacher Edition)
286
TIPS FOR AWKWARD / WORDY SENTENCES
The correct answer is often the shortest or the second
shortest answer choice. If you must guess, guess short.
Avoid the words “being” and “having.” These two often
show up in wordy sentences.
Gerunds can make a sentence sound funny. Whenever
they are not necessary, avoid words that end with -ing.
Avoid redundancy.
Watch for sentences that are in the passive voice.
Be careful that your answer does not change the meaning
of the original sentence.
Watch out for tricky NO CHANGE problems. If you’re not
sure the original sentence is wrong and you can’t find an
answer choice that you’re sure is right, don’t be afraid to
choose NO CHANGE.
WRONG & RIGHT

Examples:
1.1 Wrong: Last week I spent the week in Hawaii.
Right: I spent last week in Hawaii.
1.2 Wrong: At this point in time we are joined by Ronald, who is with us to
ensure that we are able to conduct the election.
Right: Ronald has joined us to facilitate the election.

Your Turn:
1.3 Wrong: Any individual candidate who submits his or her paperwork after
the deadline will be ineligible to be elected in the election.
Right: _________________________________________________________
Candidates who do not submit paperwork by the deadline will be ineligible.
_________________________________________________________

BASIC SKILLS PRACTICE


Eliminate the redundancy and wordiness in the following sentences.

2.1 Watercolor paints are hardly ever used for the painting of graffiti.
Watercolors are rarely used for graffiti.

2.2 Shantelle had just arrived and that was when she was told about the
problem with the faulty plumbing.
Shantelle had just arrived when she told me of the faulty plumbing.

2.3 Smitty was the only person who was able to defeat the video game.
Smitty was the only one to defeat the video game.

2.4 My doctor cautioned me with a warning not to eat vegetables under any
circumstances at all.
My doctor cautioned me to avoid eating vegetables.

SAT v17.10 Unauthorized copying or reuse of any part of this page is illegal. 287
(SAT Teacher Edition)
Redundancy & Wordiness

288
SAT v17.10 Unauthorized copying or reuse of any part of this page is illegal. 289
(SAT Teacher Edition)
Writing Unit #5
Homework Drill #1

290
SAT v17.10 Unauthorized copying or reuse of any part of this page is illegal. 291
(SAT Teacher Edition)
292
Writing Unit #5
Homework Drill #2

SAT v17.10 Unauthorized copying or reuse of any part of this page is illegal. 293
(SAT Teacher Edition)
294
SAT v17.10 Unauthorized copying or reuse of any part of this page is illegal. 295
(SAT Teacher Edition)
Writing Unit #5
Homework Drill #3

296
SAT v17.10 Unauthorized copying or reuse of any part of this page is illegal. 297
(SAT Teacher Edition)
298
Math

SAT v17.10 Unauthorized copying or reuse of any part of this page is illegal. 299
(SAT Teacher Edition)
300
1 Basic Math Strategy 302

Solving Backwards 311


Math Strategy Guide

Picking Your Own Numbers 315


Math Unit #1

Basic Math - Part 1 319

Basic Math - Part 2 323

Homework Drills:

1 2 3
327 328 329

SAT v17.10 Unauthorized copying or reuse of any part of this page is illegal. 301
(SAT Teacher Edition)
General Math Strategy

Format and Content of the SAT Math Sections


• There are two Math sections on the SAT with a total of 58 Math questions.

• The first Math section has 20 questions (15 multiple-choice and 5 grid-in).
You'll be allowed 25 minutes to complete this section.
The use of a calculator is not permitted on this section.

• The second Math section has 38 questions (30 multiple-choice and 8 grid-in).
You'll be allowed 55 minutes to complete this section.
The use of a calculator is permitted on this section.

Heart of Algebra: 19 Questions (33%)


• Analyzing and fluently solving linear equations and systems of linear equations

• Creating linear equations and inequalities to represent relationships between


quantities and to solve problems

• Understanding and using the relationship between linear equations and inequalities
and their graphs to solve problems

Problem Solving and Data Analysis: 17 Questions (29%)


• Creating and analyzing relationships using ratios, proportional relationships,
percentages, and units

• Representing and analyzing quantitative data

• Finding and applying probabilities in context

Passport to Advanced Math: 16 Questions (28%)


• Identifying and creating equivalent algebraic expressions

• Creating, analyzing, and fluently solving quadratic and other nonlinear equations

• Creating, using, and graphing exponential, quadratic, and other nonlinear functions

Additional Topics in Math: 6 Questions (10%)


• Solving problems related to area and volume

• Applying definitions and theorems related to lines, angles, triangles, and circles

• Working with right triangles, the unit circle, and trigonometric functions

302
Get to Know these SAT Math Formulas...
Area of a circle: A = πr2

Circumference of a circle: C = 2πr

Area of a rectangle: A = lw

Area of a triangle: A = 12 bh

Volume of a box: V = lwh

Volume of a cylinder: V = πr2h

Pythagorean Theorem: a2 + b2 = c2

Calculating percent: whole x percent = part

Percent increase (or decrease): percent change = new value - original value
original value

Average (arithmetic mean): sum of values


mean = number of values

SAT v17.10 Unauthorized copying or reuse of any part of this page is illegal. 303
(SAT Teacher Edition)
... and these SAT Math Formulas.
Third side of a triangle: |a − b| < c < a + b

Distance formula: (x2 − x1)2 + (y2 − y1)2

Midpoint formula: 1 2 1 2
x +x y +y
2
( , )
2
y −y
Two-point slope formula: slope = 2 1
x2 − x1

Slope-intercept form: y = mx + b

Quadratic function (standard form): y = ax2 + bx + c

Probability: probability = number of desired outcomes


number of possible outcomes

Geometric probability: geometric probability = target area


total area

"DIRT" formula: distance = rate x time

opposite a
Trigonometry formulas: sin A =
hypotenuse
=
c B

c se
opposite
adjacent b enu a
cos A = = ot
hypotenuse c hyp
adjacent
A b C
opposite b
tan A = =
adjacent c

Special triangles:
60°
60° 13
45° 2x 12
l 2 n n 5
l x 4

45° 30° 60° 60°


l x 3 n 3 5

304
Time Management is Key.
1. Knock out the easy, "must answer" questions first. Skip difficult
problems and come back to them later. On the Calculator Section,
you've got 1.5 minutes per question. On the No-Calculator Section,
you've only got 1.25 minutes per question.

2. On easy questions - bank time to spend on harder problems.

3. On difficult questions - spend the time you banked earlier.

4. When using process of elimination do not start with the top answer.

5. Apply the 10-Second rule. If you arrive at an answer for a difficult


question in only 10 seconds, then you're probably choosing a trap
answer.

Short on Time?
1. NEVER leave a question blank!

2. Use a Ballpark Estimate to Identify and Eliminate Wrong Answers.

3. Use Process of Elimination to Make an Educated guess.

4. Guessing Blindly to finish a section? Pick ONE letter and stick with it.

SAT v17.10 Unauthorized copying or reuse of any part of this page is illegal. 305
(SAT Teacher Edition)
Figure It Out
1. Label the figures that you're given.

What is the perimeter of the figure below if A and D are


right angles?

A 6 B
A) 28

B) 34 10
C) 36

D) 41 D C
14

2. No figure? Then Draw your own.

The dimensions of a steel shipping container measure


12ft x 12ft x 20ft. The two sides with the least surface
area are to be painted blue. If one gallon of paint covers
80 square feet, then how many gallons of paint will be
required to complete the job?

A) 3
B) 3.2
C) 3.6
D) 4.2

Memorize these Common Values


1 1
= 0.10 = 10% = 0.50 = 50%
10 2

1 = 0.20 = 20% 2 2
= 0.66 = 66 %
5 3 3

1 = 0.25 = 25% 3
= 0.75 = 75%
4 4

1 1 1 = 1.00 = 100%
= 0.33 = 33 %
3 3 1
306
12 SAT Math Skills to Master
1. Translating - turning a word problem into an algebraic expression.

2. Using known quantities (area, lengths, angles, etc) to find


unknown quantities.

3. Recognizing the Classic Expressions when they're disguised in


SAT problems.

4. Paying Close Attention to Units and Variables.

5. Working with Inequalities.

6. Using the Distance/Work Formula.

7. Recognizing the different types of triangles in SAT problems.

8. Using specific numerical examples to prove/disprove your guess.

9. Keep it simple. Doing lots of calculations? You may have


overlooked simple fact in the problem that would have saved you
time.

10. Avoiding Trap Answers.

11. Working Systematically and Using Process of Elimination.

12. Entering Answers Carefully and Correctly.

SAT v17.10 Unauthorized copying or reuse of any part of this page is illegal. 307
(SAT Teacher Edition)
SAT Math Mistakes to Avoid
1. Misreading part of the problem

2. Miscalculating while trying to "do it in your head"

3. Failing to write out the problem

4. Punching the wrong buttons on your calculator

5. Attempting to perform too many steps at once

6. Stopping after one or two steps on a medium or hard


problem when the answer requires three or four steps

7. Solving for the wrong variable or expression

8. Trusting a misleading diagram or chart

9. Messing up the PEMDAS order of operations

10. Spending time on a difficult problem before you've


completed all the easier problems.

There's Only One "Silly Mistake"


Do not dismiss a mistake as "silly" and then quickly move on without taking
a moment to appreciate the lesson of that error. Instead, take a moment to
analyze the mistake, ask yourself how it occurred, and consider what may be
done to prevent it from happening again.

If you're serious about increasing your SAT score, then take a moment to
commit yourself to the process. You're going to miss some problems. That's
life. How you react to learning that you've missed a problem will define the
limits of your potential improvement.

Don't rush the process. Analyzing errors is a BIG part of making sure they don't
occur again on test day!

308
Do You Speak Math?
Before we get started, let's make sure you speak the language. Match the words on
the left with their descriptions on the right.

R 1. Positive A. the answer in an addition problem


N 2. Negative B. the top number in a fraction
A 3. Sum C. integers that can be multiplied into a number
I 4. Diameter D. average
T 5. Product E. the space taken up by a shape
F 6. Difference F. the answer in a subtraction problem
L 7. Remainder G. can be divided evenly with no remainder
Y 8. Even H. not divisible by two
H 9. Odd I. the distance across a circle, passing through the center
J 10. Integer J. any number that's not a fraction or decimal
M 11. Prime K. in order
D 12. Mean L. what's left over in a division problem
O 13. Median M. a number that's only divisible by itself and 1
Q 14. Mode N. less than zero
C 15. Factor O. the middle number
Z 16. Multiple P. the space taken up by a three-dimensional shape
G 17. Divisible Q. the most frequently occurring value in a set of numbers
B 18. Numerator R. greater than zero
V 19. Denominator S. the distance around the perimeter of a circle
E 20. Area T. the answer in a multiplication problem
U 21. Perimeter U. the distance around a shape
K 22. Consecutive V. the bottom number in a fraction
W 23. Radius W. the distance from the edge of a circle to its center point
S 24. Circumference X. the digit in the one's place
X 25. Units Digit Y. divisible by two
P 26. Volume Z. the results when a number is multiplied by 2, 3, 4, 5, etc.

SAT v17.10 Unauthorized copying or reuse of any part of this page is illegal. 309
(SAT Teacher Edition)
Calculator Tips
1. Don't overuse your calculator - especially while studying. Remember
that the SAT is still in many ways a reasoning test. Calculators are
great at performing your calculations. They're pretty lousy at
thinking for you.

2. Don't bother installing any special software on your calculator.


Many of these programs promise to help you solve all sorts of
questions with just 24 simple keystrokes. No thanks.

3. Fresh batteries are a must for test day. It's a proven fact that
calculators do not work as well with dead batteries. Keep that in
mind.

4. Use your calculator to quickly find a remainder - without using


long division.
Suppose that 196 people are participating in a parade.
There are 12 floats in the parade, some carrying 12 and
the rest carrying 13. How many floats carry 13?

Basically, we're asked to find the remainder when 196 is divided by 12. But if you
punch that into your calculator, you'll get 16.33333333. That means we've got
16.33333333 groups of 12. We don't care about the 16 groups of 12, so subtract 16 by
pressing ANS - 16. That leaves us with the number of groups of 12 we have left over.
But we want the number of people, not groups. So multiply ANS x 12 and you'll get 4.
This means there are 4 remaining people who have to be added to other floats in the
parade. As a result, we've got 4 floats that carry 13 people.

5. MATH FRAC

A bag contains a total of 63 red and blue marbles?


If the bag contains 27 red marbles, what is the
probability of randomly pulling a blue marble from
the bag?

It's easy enough to see that the probability of pulling the blue marble is 36/63, but
what do you do when that's not one of the fractions in the answer choice? You could
punch it into your calculator, but you'll get .5714285714. What do you do with that?
Just punch MATH FRAC to turn that decimal into fully reduced, equivalent fraction.
In this example (36/63), it's pretty clear that you could just divide top and bottom by
9 to get 4/7, but what do you do with something like 117/169?

6. Make sure the calculator you plan to use is allowed on the SAT
https://sat.collegeboard.org/register/calculator-policy

310
Solving Backwards

1 (No Calculator)

SAT v17.10 Unauthorized copying or reuse of any part of this page is illegal. 311
(SAT Teacher Edition)
• This strategy works well when the problem has complicated algebra but the answer
choices are numbers.

• Use the information in the problem to test each answer choice:

Start with answer choice (C) and work the steps of the problem. There will always
be information in the question that tells you when you get to the right answer.

Since the answer choices are generally ordered from least to greatest, if (C) is
incorrect, you can usually determine whether you're looking for a smaller or larger
answer choice. Now you can eliminate two more answers.

Once an answer choice works, you're done. Choose that answer and move on.

key to this
Time travel isnethede to travel back in
problem. We w many tickets the 1 (No Calculator)
time to see hot the very beginning,
player had a gan giving them away.
before he be

st a rt th er e. W e know that pthtoe


let's dd u
he 's go t 6 le ft over at the ened,tisockets he's got left over must a
We know ckets he gave away + th
sum of the ti tickets he started with.
the number of art
G iv en A w a y = Tickets at the st
+ Tickets
Leftover tickets
6 + 4 + ( )
x-4 + 8 =
2
x

ly by tw o to ge t rid of the fraction.


Multip
12 + 8 + (x - 4) + 16 = 2x
x-4 x
6+4+ 2 +8=
and solve for x.
Combine like terms 32 + x = 2x
36 + (x - 4) = 2x
32 = x
And solve for x.

Solving backwards is a variable assignment with a twist. Instead of picking our own numbers,
we are now using the numbers from the answer choices. If you are tempted to write an
algebraic equation, then you should think about solving backwards.

312
Solving Backwards

1 (No Calculator) 3

2 (No Calculator) 4

SAT v17.10 Unauthorized copying or reuse of any part of this page is illegal. 313
(SAT Teacher Edition)
314
Picking Your Own Numbers

1 (No Calculator)

SAT v17.10 Unauthorized copying or reuse of any part of this page is illegal. 315
(SAT Teacher Edition)
• Use this strategy on problems that have variables in the answer choices or when a
"in terms of x"
problem says ______________________.

small numbers that are easy to work with like _____________.


• Choose _______ 2, 3, 4, 5, or 10

100 is a great number to choose on percent problems. Don't be afraid to choose


• ______
negative
___________ numbers if doing so simplifies the problem.
___________

• Avoid ____
0 and ____
1 because they can give weird results.

• If you pick numbers for a geometry question, make sure all the angles in straight lines
180 degrees
and triangles add up to ___________.

1 (No Calculator)

ld solve this
Obviously, youracoicually, but
problem algebown numbers
picking your time!
saves a lot of 6 does equal 3, so we're
b = 2 . r
at the Let's pick a =N6owa,ndall we've got to do is plug in aound
2
Just make supireckthare "true" e. es si on
numbers you ntext of the good ther a and b on the other expr
within the co ou pick a = 5 and numbers for
problem. If ya won't equal we're all set.
b = 2, then b t the wrong 6(2) = 12 = 2
3 and you'll ge6b . 6 6
answer for a

• If you need to choose numbers for more than one variable, choose different numbers to
avoid coincidences that can disguise the correct answer.

• Work the steps of the problem until you get to the desired answer. Then go to the answer
choices to find the one that gives you the desired answer. If multiple answers work, then
pick different numbers and try again.

• If a number you chose leads to ugly results - fractions, decimals, or anything complicated
- don't be afraid to start over and plug in a better number.

316
Pick Your Own Numbers

1 (No Calculator) 3 (No Calculator)

2 (No Calculator) 4 (No Calculator)

SAT v17.10 Unauthorized copying or reuse of any part of this page is illegal. 317
(SAT Teacher Edition)
318
Basic Math - Part 1

1 (No Calculator)

3/32
/ /
. . . .
0 0 0

SAT v17.10 Unauthorized copying or reuse of any part of this page is illegal. 319
(SAT Teacher Edition)
FRACTION (N): a part of a whole, represented as
the part divided by the whole

1
OUR APPROACH: ob le m w it h tw
ual
o fractions set equation
4
a pr eq NUMERATOR (N): the top number in a fraction,
Whenever you seyeou should consider simplifying theIt works
,
to one another ient art of "cross-multiplication ." representing the number of parts taken from the
whole
by using the anc
like this:
2 = 2b. 1
If ab = 3 then 3a 4
helps
Cross-multiplicatipronoblems DENOMINATOR (N): the bottom number of
clean up harder below! a fraction, representing the number of parts into
too, like the one which the whole is divided

1
4
If you cross-multiply the
equation above, you'll get:
(8x + 3y)(4) = (3y)(5) 1 (No Calculator)

Simplify by distributing
and you'll get:
32x + 12y = 15y
Subtract 12y from each side
3/32
/ /
and you'll get: . . . .
0 0 0
32x = 3y

Keep working towards yx


by dividing both sides by 32. SOMETIMES IT'S OKAY TO BE IMPROPER
You'll get: And taking the SAT is one of those times. Now, that doesn't mean
you should curse loudly during the exam or challenge the proctor to
x = 323 y a spitting contest. No, that would be inappropriate, which in this case
() is very different from being improper.
And finally, isolate yx by dividing
both sides by y and you'll get: We're talking about improper fractions, which are A-okay as far as the
SAT is concerned. So don't waste time trying to convert an improper
yx = 323 fraction like 65 into its more respectable decimal equivalent. Just
4
bubble in 65 and move on!
4

NOT A GOOD TIME TO "MIX THINGS UP!"


Call it close-minded. Call it old-fashioned. Call it whatever you like,
5
but numbers like 6 4 have no place on SAT grid-in questions. That's
because the grading machine can't tell whether you meant to grid-in
5 65
6 4 or 4 . So avoid a mix-up and stay away from mixed numbers!

320
KNOWING FRACTIONS IS 1/2 THE BATTLE
Simplify the following expressions.

1.
7
+
1
= 38 4.
2
+
9
= 11
3 5 15 7 7
7

5
2. 2 =
25 5.
3

x
= 15 - 4x
3 6 4 5 20
5

63 6n +9
3. = 3 6. = 2n + 3
21 12n
4n

CONVERT EACH OF THE FOLLOWING INTO A FRACTION

7. 75% = 3 10. .333... = 1


4 3

8. .6 = 3 11. 125% = 5
5 4

9 3 23
9. 45% = 12. 54 =
20 4

AT THIS TIME, PLEASE REMAIN IN YOUR SEAT AND SOLVE EACH OF THE FOLLOWING.
30 = 5
13. If a bus carries 24 boys and 30 girls, what fraction of the passengers is female?
54 9

3
14. If 4 of the seats in a movie theater are occupied, and there are 12 empty seats, how many total
seats are there in the theater? 48

2
15. If it rained 5 of the days in a 120-day period, how many days got no rain at all? 72

SAT v17.10 Unauthorized copying or reuse of any part of this page is illegal. 321
(SAT Teacher Edition)
Basic Math - Part 1

1 (No Calculator) 3

2 (No Calculator) 4

322
Basic Math - Part 2

1 (No Calculator)

SAT v17.10 Unauthorized copying or reuse of any part of this page is illegal. 323
(SAT Teacher Edition)
POSITIVE (ADJ): describes any number that is
greater than zero; all numbers to the right of zero on
a number line
2
1, 2, 3, 304.75,
3

NEGATIVE (ADJ): describes any number that


is less than zero; all numbers to the left of zero on a
number line
6
-1, -2, -3, -51.5, -
31
slation
APPROA
CH:
h tf o r w ard tran ZERO (N): the neutral number on a number
tra ig
pretty s turn this
This is a o n a s we can pretty
line between the positive and negative numbers,
problem.
As s o
a ti o n , it should be representing no quantity or number; nothing; the
t into an
eq u eals with
statemen o n e , h o wever, d absence of positive and negative quantity
his an
solve. T , which c
easy to
e g a ti v e numbers e ly expect. 0, 0.00
nd n t intuitiv
positive a u m a y n o
ways yo
behave in

1 (No Calculator)

to" "2"
"is equal
number"
ix te e n times" "a
"more th
an" "s
= 2
"Twelve" x
16
+
12

uation is:
e our eq
Looks lik 2
12 + 16x =

e for X:
Let's solv 2
12 + 16x = -12
-12
16x = -10
-10 -5
x = 16 = 8

324
PRACTICE WORKING WITH POSITIVES & NEGATIVES:

ZERO
LESSER GREATER
0
2 -2 8
1. (-5) + (-12) = - 17 6. − = 21
3 -7

2. (-3) − (-2) = - 1 7. 6x2 + -4x2 = 2x2

3. 6 − 14 = - 8 8. -11x2 − (-4x2) = -7x2

4. -4 − 1 = (-4)3(-1)20 = -16
2 - 4 12 or - 92 9.
(22)(53) 125

11 -3 -4
5. 2 −
3
= - 53 10. ( )( ) =
2 3
2

THE "WEIRDNESS" OF NEGATIVE NUMBERS RAISED TO EXPONENTS


Solve these expressions and plot the points on the number line below.

1. (-2)1 = - 2 4. (-2)4 = 16

2. (-2)2 = 4 5. (-2)5 = - 32

3. (-2)3 = - 8 6. (-2)6 = 64
ZERO 4
(-2)5 (-2)3 (-2)1 0 (-2)2 (-2)4 (-2)6

IT GETS WEIRDER: NUMBERS BETWEEN ZERO AND -1


Solve these expressions and plot the points on the number line below.

1. (-.5)1 = - .5 3. (-.5)3 = -.125

After plottiengprtheveious
points in thses, what 2. (-.5)2 = .25 4. (-.5)4 = .0625
two exerci you
patterns do
notice? -.5 ZERO .5

(-.5)1 (-.5)3 0 (-.5)4 (-.5)2

SAT v17.10 Unauthorized copying or reuse of any part of this page is illegal. 325
(SAT Teacher Edition)
Basic Math - Part 2

1 (No Calculator) 3 (No Calculator)

7/3
/ /
. . . .
0 0 0

2 (No Calculator) 4

20
/ /
. . . .
0 0 0

326
Math Unit #1
Homework Drill #1

1 3 (No Calculator)

2 (No Calculator) 4 (No Calculator)

SAT v17.10 Unauthorized copying or reuse of any part of this page is illegal. 327
(SAT Teacher Edition)
Math Unit #1
Homework Drill #2

1 3

2
/ /
. . . .
0 0 0

/ /
6
. . . .
0 0 0

2 (No Calculator) 4

328
Math Unit #1
Homework Drill #3

1 3

/
32
/
. . . .
0 0 0

2 (No Calculator) 4

SAT v17.10 Unauthorized copying or reuse of any part of this page is illegal. 329
(SAT Teacher Edition)
330
2 Translating Word Problems - Part 1 333

Translating Word Problems - Part 2 339

Solving Linear Equations - Part 1 343

Solving Linear Equations - Part 2 349


Heart of Algebra
Math Unit #2

Interpreting Linear Equations 353

Linear Inequalities & Absolute Value 361

Solving Linear Systems 367

Graphs of Lines 373

Homework Drills:

1 2 3
377 378 379

SAT v17.10 Unauthorized copying or reuse of any part of this page is illegal. 331
(SAT Teacher Edition)
332
Translating Word Problems - Part 1

1 (No Calculator)

SAT v17.10 Unauthorized copying or reuse of any part of this page is illegal. 333
(SAT Teacher Edition)
TRANSLATION (N): the super-useful SAT
math skill of converting long bodies of text into
simple, easy to solve, mathematical expressions and
equations

CH: a way to
APPROA u n c ti o n is just Tom receives a monthly paycheck equal to $2,000.00
er that a
f le, given
Rememb e o f o ne variab een
plus 15% of the dollar amount of his monthly sales.
e the va lu ship betw
e relation
If his paycheck last month was $9,800.00, what was
determin a n d th
variable the dollar amount of his sales?
another it is.
That's all
the two. e
tion in th
e g iv e n the func e s the
se, we'r m that d
escrib
In this ca p r o b le e -w eek 9,800 = 2,000 + (.15)s
a word f a on
form of e e n th e price o s it
hip betw f mile
relations umber o
r e n ta l and the n a ti o n, we're
bulldozer th a t inform All that's left is to solve for the variable s.
en. Giv e n renting a
gets driv in e th e cost of
determ g miles.
asked to d r iving it 10
a n d
bulldozer

1 (No Calculator)

e week.
w e $ 9 00 for th
e we o s it's not
Looks lik x e d , w h ich mean
t is fi we drive
the
That par
te d b y how far o r every
affec w e $21.50 f
e'll als o o on looks
thing. W iv e , s o o ur equati
e dr
10 miles w
like th is :

900 +
21.50 (10g)
10 out,
s cancel
e n o u g h , the ten
ntly
Convenie :
'r e left with
so we
0g
900 + 21.5
A!
r choice
s lik e it 's answe
Look

334
Math Translation Guide
Often times the SAT overwhelms students with long, wordy problems. Do not get
overwhelmed, instead translate the words into a mathematical equation.

This translates to This


increased by / sum / more than +
decreased / difference / less than -
is / was / has / is equal to =
product / of / times / multiplied by x
for / per ÷
square of / squared / times itself ( )2
square root √
percent ÷ 100 (divide by 100)
what / some number /a certain number a variable (a, b, c, d, e, n, m, x, y, z, etc.)
the same number the same variable you've already used

Word-for-Word Rule: For every translation problem, read left to right and
translate word for word. Don't skip any words, and don't try to translate the
whole thing all at once. Break it into chunks, and carefully translate them, one
chunk at a time.

Less is More Difficult Than You Think: "Less than" does mean subtract. That part
is simple. But what do you subtract from? When you see "less than," remember
that the number that comes first in the sentence comes second in the equation.
For example: "7 less than 6" translates to the expression 6 - 7. Crazy, right? But,
you do it every day without thinking about it. Suppose someone asks you,
"What's 3 inches less than your height?" First you take your height, and second
you subtract 3 from it.

Remember to follow the Order of Operations: Use the mnemonic device


"Please Excuse My Dear Aunt Sally" to help you remember the order in which
you should solve an algebra problem: Parentheses, Exponents (and Roots!),
Multiplication, Division, Addition, and Subtraction.

SAT v17.10 Unauthorized copying or reuse of any part of this page is illegal. 335
(SAT Teacher Edition)
TRANSLATE EACH OF THE FOLLOWING
INTO AN ALGEBRAIC EXPRESSION

1. The sum of x and 7 is equal to 31.


x + 7 = 31

2. The difference of p and 22 is equal to 34.


p − 22 = 34

3. The product of a number and 5 is 22.


5x = 22

4. A number cubed is 55.


x3 = 55

5. The product of a number and 6 is greater than


or equal to 20.
6x ≥ 20

6. 9 more than a number is less than or equal to 21.


x + 9 ≤ 21

7. The 5th power of x is 100,000.


x5 = 100,000

8. 4 to the p power is equal to 256 .


4p = 256

9. 3 less than x is 12.


x − 3 = 12

10. A number increased by 7 is 22.


x + 7 = 22

336
Translating Word Problems - Part 1

1 (No Calculator) 3 (No Calculator)

2 (No Calculator) 4 (No Calculator)

SAT v17.10 Unauthorized copying or reuse of any part of this page is illegal. 337
(SAT Teacher Edition)
338
Translating Word Problems - Part 2

1 (No Calculator)

SAT v17.10 Unauthorized copying or reuse of any part of this page is illegal. 339
(SAT Teacher Edition)
YOUR SUPER-SECRET DECODER RING: THE WORD "IS"
Some sentences are confusing. They just are. Despite that, you can
uncover the true meaning of a confusing sentence by rewriting it as
an "is" statement.

"There are twice as many hotdogs as cheeseburgers."

So, which is it? 2h = c or 2c = h ?

Let's rewrite it as an "is" statement.:


"The number of hotdogs is twice the number of cheeseburgers."
=
Boom! Now it's obvious! h = 2c

:
CAUTION
'R E A SKED TO
WE
VALUE
FIND THE 1 (No Calculator)
OF 2X,

RE YOU
MAKE SU HE
HOOSE T
DON'T C
F X!
VALUE O

at
as a gre
P R O A C H: e d th is problem e answer
AP
if y o u spott y p lu g g ing in th
First of
a ll, ward b st keep
y to s o lve back n o t, n o biggie. Ju
it this
opportun for you!
If
still solve
ic e s , th en good ti m e . We can t to
ch o at nex t g o
ut for th an, we've
an eye o b u t b e f ore we c m a th to form
e a lg e b r aically, e p r o b lem into
on s in th
the word
translate
tion.
our equa 33
11 +
14 + 6x =
d. Just
s tr a ig h tforwar
etty and
ut, it's pr mistakes
r o m h e re on o a n y a r ithmetic r 2 x.
F make lve fo
c a r e f ul not to e e n a s k ed to so
b e e've b
r that w
remembe
11 + 33
14 + 6x = -14
-14
6x = 30
3 3

2x = 10
340
MAKE LIKE JEAN-FRANÇOIS CHAMPOLLION,* AND TRANSLATE THESE STATEMENTS.
Translate these statements, and solve for the variable indicated in terms of another variable.

1. If m is 5 less than n divided by 3, what is n 3. If a times b is 7 less than the value of c


in terms of m? divided by d, what is the value of d in terms
of a, b, and c ?
n = 3m + 15
c
d = ab + 7

2. If x is 21 more than three times y, what is y 4. -4 times the product of a and b is eight more
in terms of x? than the product of c and d. What is b in terms
of a, c, and d?
y = x - 21
3 cd + 8
b = -4a

INVENTORY PROBLEMS
SAT tests often include at least one problem that challenges you to determine how many items of a set remain
after a certain number of those items have been used or given away. The trick is that you're not given any real
numbers to work with. Remember not to freak out when you see "in terms of..." in the question. Just create an
expression that accurately expresses the relationship between all the variables, and then isolate the variable
that the question asks you to find in terms of the others.

5.. 6.

* In 1822, Jean-François Champollion, who despite being largely self-educated was fluent in 12 languages by the age of 16, succeeded in translating the hieroglyphs

on the Rosetta Stone. Despite his linguistic genius, he did poorly on the Math portion of the SAT and was denied admission to UC Berkeley.**

**Parts of the above story have been modified for didactic purposes.

SAT v17.10 Unauthorized copying or reuse of any part of this page is illegal. 341
(SAT Teacher Edition)
Translating Word Problems - Part 2

1 (No Calculator) 3

2 (No Calculator) 4

342
Solving Linear Equations - Part 1

1 (No Calculator)

9/25
/ /
. . . .
0 0 0

SAT v17.10 Unauthorized copying or reuse of any part of this page is illegal. 343
(SAT Teacher Edition)
so
problem,
P R O A C H: f o r w a r d algebra e 'r e
AP traight at w
pretty s te of wh
This is a F ir s t, m ake a no to find the
d iv e right in. , w e 'r e asked mbining
le t' s
fi n d . In this ca s e
t a m a tt er of co
asked to is jus uation.
x . S o , r eally, this e s id e o f the eq
value of n on
lating x o
e te r m s and iso ns in the
lik
a t th e fractio
tion is th ks like
a d d e d complica d e n o m in ator. Loo
The only on
e a comm
r o b le m don't hav s ta r t.
p e to
a t' s a g reat plac
th

en: 1 + 6
We're giv 4 x = 15
1 x+ 5
3 3
,
right side
5
x b ut on the d a
got 3 fin
ft, we've need to
On the le c o m p lic ated. We
ore
m
it's a little r.
enominato
commo n d 1 (3) + 6
5 x= 15
5 (3)
3

1 (No Calculator)

3 +6
5 x=
15 15
3
9
ORDER OF OPERATIONS (N):
5 x= a collection of rules that define which procedures
15
3 to perform first in order to evaluate a given
mathematical expression
!
lve for x
ly and so
as 5 .
s m u lt ip x
cros the same
Now let's t 5 x is 3
When reading an english book, you begin reading
er tha 3
Rememb each line on the left and move to the right. You
don't skip around, so there's never any confusion.
x Sometimes, you need to skip around with math.
27 = 75
6+3x2 + 4-2
27 = x 4
4x6 8+2
75
As you can see, the result of that expression is
9 =x 9/25 affected by the order you perform the various
25 / /
operations. That's why 9 out of 10 NASA scientists
. . . .
recommend you use:
0 0 0

P.E.M.D.A.S.
("PG - ER - MD - AS" - if you're not into the whole
brevity... thing.) Let's take a look on the next page.

344
6+3x2 + 4-2 Order of Operations:
PG - ER - MD - AS
2
4x6 4+2

1. Parentheses and Grouping Symbols:


(6 + 3 x 2) + (4 - 2)
4
(4 x 6) (8 + 2 ) Work from innermost to outermost and left to right.
Fraction bars and radicals count as grouping symbols,
so we'll just draw those in to help you visualize this
(6 + 3 x 2) + (2)
4 solution. At this point, if a set of parenthesis contains
(24) (8 + 2 ) only one operation, go ahead and knock that out.

2. Exponents and Roots:


(6 + 3 x 2) + (2)
4
(24) (8 + 2 ) So far, so good. Now let's take a look at Exponents
and Roots. Again, work from innermost to outermost
(6 + 3 x 2) + (2) 2
and left to right. Looks like we've only got the 2 to
(24) (8 + 16 ) worry about. Let's take care of that.

3. Multiplication and Division:


(6 + 3 x 2) + (2)
(24) (8 + 16 ) Next we do the multiplication inside the parentheses.
That means it's time to multiply the 3 x 2.
(6 + 6) + (2)
(24) (8 + 16 )

(6 + 6) + (2) 4. Addition and Subtraction:


(24) (8 + 16 )
Almost done. Time to Add and Subtract. Once more,
work from left to right. Add the 6 + 6 and the 8 + 16.
(12) + (2)
(24) (24 )

12 + 2 = 14 = 7 At this point, we're all done with the parentheses, so


24 24 24 12 we can divide numerators by the denominators. But
hold on a second! The two fractions share a common
denominator, so we can add them together now, and
then reduce the resulting fraction. This will save us the
extra work of having to find a common denominator
again in order to add the two fractions together later.
SAT v17.10 Unauthorized copying or reuse of any part of this page is illegal. 345
(SAT Teacher Edition)
DISTRIBUTIVE LAWS (N):
When multiplying by a grouped sum, you don't have to do
the grouped sum first; you can multiply first, so long as you
distribute the multiplication over the entire sum.

a(b + c) = ab + ac
3 × (2 + 5) = (3 × 2) + (3 × 5)

But be careful that you don't "over-distribute." For example:

3(2 × 5) �= (3 × 2) + (3 × 5) �= (3 × 2)(3 × 5)

The same law holds true for distributing division.


15a + 3ab 15a 3ab 5a
= + = +a
3b 3b 3b b

LET'S TAKE A SECOND TO REVIEW A FEW OF THE SKILLS WE'VE COVERED

3x
1. (1 − (1 + 1) − (2 + 3) + (−1 + 2) − 2) = 4. If + 3 = 7, then 9x + 4 =
2
-7 28

2. When 12 is subtracted from 4 times a 5. On a typical day, a deli sells n pounds


a number, the result is 56. What is the of sliced turkey for d dollars each.
number? One day the price is reduced by 20%
and the deli sells 50% more sliced
x = 17
turkey. Which of the following
represents the revenue for the day's
sliced turkey sales, in dollars?
3. If 7c + 15 = 29, then c − 3 =
-1 A) 1.5nd - 0.2
B) 1.05nd
C) 1.50nd
D) 1.2nd

346
THE LAW OF SUBSTITUTION (N): TRY A COUPLE:
If two expressions are equal, then you may
substitute one expression in place of the other.
This really helps in cases when you've got more
than one equation to work with.
5y − 2x = 13 3x − 5y = −7

If we can isolate one of the variables and then


plug it back into the other equation.

5y = 2x + 13

3x − 5y = −7
3x − (2x + 13) = −7
x − 13 = −7

At this point, we can substitute 6 in


to solve for y. = 5
x=6

5y − 2(6) = 13
5y − 12 = 13
5y = 25
y=5

DISTRIBUTE: SUBSTITUTE AND SOLVE:

1. 7(x − 8) + 7(−4x + 2) 1. x=0 y = −1


3x − 4y = 4
3x + y = −1
−21x − 42

2. −3n(2n + 7) − 6n(n − 4) 2. −4x + 5y = 8 x=3 y=4


−x + y = 1
−12n2 + 3n

3. 3p(4p − 2) + 2p(1 + 4p) 3. −7x + 2y = −24 x=2 y = −5


−x − 4y = 18
20p2 − 4p

SAT v17.10 Unauthorized copying or reuse of any part of this page is illegal. 347
(SAT Teacher Edition)
Solving Linear Equations - Part 1

1 (No Calculator) 3 (No Calculator)

2 4

348
Solving Linear Equations - Part 2

1 (No Calculator)

SAT v17.10 Unauthorized copying or reuse of any part of this page is illegal. 349
(SAT Teacher Edition)
RECIPROCAL (N):

The reciprocal of a fraction is just the same


fraction inverted. For example:

2 1
is the reciprocal of
estion.
g this qu
1 2
O A C H: u t s o lv in
APPR bo
s to go a eciproca
l
r e a r e two way s id e s by the r
Th e th
ultiply bo
Reciprocals are important because whenever
u could m raction:
First, yo th e f you multiply a fraction by its reciprocal, you
3 to get rid of

( )
get 1. As a result, this is a great technique for
of 7

( )
7 5 eliminating fractions in SAT problems, so you
7 3 m = 3
9 can more easily isolate variables.
3 7
:
isolated m 3 1
lik e th a t, we've is the reciprocal of 2
And just 7 3
35
m = 27

1 (No Calculator)

USE THESE TECHNIQUES TO SIMPLIFY ALGEBRAIC MANIPULATION

• Find a common denominator or multiply by the reciprocal.


• Combine like terms.
• Cancel to eliminate fractions.
• ALWAYS do the same thing to both sides of the equation.
• Work toward isolating the variable or expression you're asked to find.
• If you're asked to solve for an expression, keep an eye out for shortcuts to solve
directly for that expression.

350
ALGEBRAIC EXPRESSION (N):
a mathematical phrase that can contain ordinary
numbers, variables (like x and y), and operations
(like +, −, x, and ÷)

Distribute: x (n - m) = 51 (3z − 2y)2 2y + 13


xn - xm = 51
9n
Plug in 3 for b b 7
xm to solve 4+ (n2)( )
isolate xn: xn - 3 = 51 2 2

Solve for xn: xn = 54 WORK TOWARD YOUR ANSWER!


Sometimes you want to take the scenic route. SAT
OUR APPROACH: y ou 'r e ca re fu
tricky
l but super-dupestr-ribute
test day is not one of those times. For the above
type of question, it's often very easy to solve for
sy if to di
This problem is eale ss . O n th is problem, you'll needat you solve for one of the variables in the question, but doing
re
if you get ca ad carefully to make sure th so leaves you with no bridge to get back to the
correctly and re ession. variable or expression that problem actually asked
the correct expr e you to find. Don't go down that road! Make sure
x a nd th en fo r n and then xn, youK'reep it you're always working toward your answer.
e for be.
If you try to solvore difficult than it actually should
m
making it way for xn in the first place!
simple and solve
ru le of th u m b w hen solving for aonn look like
ple ti
Remember this sigomal is to make one side of the eqnuaadd, subtract,
expression: the you're asked to solve for. You ca ver it takes to
the expression square, and square root - whate
divide, multiply, ession you're asked to find.
isolate the expr

SOLVE FOR THE FOLLOWING


EXPRESSIONS:

1. If − 8(−2k + 3) = −2 + (5 × k), then what is th


If − 8(−2k + 3) = −2 + (5 × k), then what is the value of 6k + 1?
k=2
6k + 1 = 13

2. If − 7(m + 3) = −3m − 1, then what is the valu


If − 7(m + 3) = −3m − 1, then what is the value of − 3m + 1?
m = −5
− 3m + 1 = 16

3. If 6(−4n − 3) = 33 − 7n, then what is the value o


If 6(−4n − 3) = 33 − 7n, then what is the value of 5 + 2n?
n = −3
5 + 2n = −1

SAT v17.10 Unauthorized copying or reuse of any part of this page is illegal. 351
(SAT Teacher Edition)
Solving Linear Equations - Part 2

1 (No Calculator) 3 (No Calculator)

2 (No Calculator) 4 (No Calculator)

352
Interpreting Linear Equations

1 (No Calculator)

/
25
/
. . . .
0 0 0

SAT v17.10 Unauthorized copying or reuse of any part of this page is illegal. 353
(SAT Teacher Edition)
LINEAR EQUATION (N):
a first degree polynomial that can take
ms
CH: f proble the form, y = mx + b, where the constant
APPROA a l o f a lot o m
blem is ty
p ic of pr le o b m is the slope and c is the y-intercept.
This pro S A T . T his type e a r
on the parts of
a lin
you'll find tify the a
to id e n b e written in
asks you ns can
o ays
In the linear equation below, we can see that the
e q u a ti r e a lw
u a ti o n . Linear u t th e p arts a rate of change (m) is , which sets up a relationship
eq rms, b
erent fo
few diff
between x and y.
.
the same e
d that th
a b ly notice
In this ca
se, you
in th e p
p r o
roblem lo
b
oks a lot
e a r e
like
q uation. Do
n't y = 3x + 5
q u a ti o n f a lin
e rm o the equa
tion
rcept fo
slope-inte a t th e parts of d e finitely So if x = 4, then all we have to do is plug that
e f o o le d by th n a m e s . This is value in for x and solve for y.
b riable !
erent va e-interce
pt form
have diff n in s lo p
quatio
a linear e

a ti o n m e ans the s
ame thing
:
y = 3(4) + 5
This equ
25
y = 12m +
Looks like when x is 4, y is 17. By the way, that
means that the point (4, 17) is on that line.

0
een $25.0
e n t m u st have b o f 25
stm r
initial inve he numbe
Alfred's th e v a lu e of d (t p r o b ably / /
that's ou can
because time = 0 . Y
but the
e has) at equation,
. . . .
dollars h okin g a t th e
u plug in
just by lo when yo
0 0 0
see that at d = 2 5
ind it is th
math beh
t.
zero for (1, 25+18)

+ 25 (t, d)
d = 12(0) +18
(0, 25)
(t, d)
d = 25
's
at Alfred
n tl y n o interest o m e s
ar e is be c
re is app e then th
Note: the the r e w e r
onents . A n d t (time) 1 2
cause if with exp
bank, be p r o b le m
ou're n o t
teresting onents, y
a more in e a lin g with exp
u're d
when yo equation.
g w it h a LINEAR d (money)
dealin

Not to scale... obviously!


354
e
The m (slope) and ththe
Slope-Intercept Form: y = mx + b b (y-intercept) are the
parts that affect
shape of the line.
m is the slope. It tells you the angle and direction of the line. Looking at the graph from left to
right, positive slopes go up and negative slopes go down. Slopes with bigger numbers (either + or -)
are steeper. Slopes with smaller numbers are flatter.

m is positive m is negative
SLOPE (N): A measure of the direction (graph goes up) (graph goes down)
of a line on the coordinate plane. Slope is
represented by m in the slope intercept form. m is smaller 1
) 1 m=- 2
(fla tt e r gr a ph m = 2

(y1 - y2)
slope = m =
(x1 - x2)
m is bigger
(steeper graph) m = 3 m = -3

Horizontal lines are expressed as y = k.


Vertical lines are expressed as x = k.
It's really easy to mix up vertical and horizontal lines. It seems like a vertical line would be
y = k and a horizontal line would be x = k because we usually think of x as dealing with the
horizontal movement and y as dealing with vertical movement. Don't get tricked!

b is the y-intercept. It tells you the point where the line crosses the y-axis. Positive y-intercept
means the line crosses the y-axis above the origin. Negative y-intercept means that the line crosses
the y-axis below the origin.

Crosses the b=2


1
Crosses the y-axis in.
y-axis 2 units unit below the orig
above the origin. b = -1

SAT v17.10 Unauthorized copying or reuse of any part of this page is illegal. 355
(SAT Teacher Edition)
Finding the x-intercept
The x-intercept of a line is the point where the line crosses the x-axis.
The x-intercept is located at point (something, 0).
To find the x-intercept, plug zero into the equation for y, then solve for x.

Counting units on a graph


Use the "counting units" to find the slope of a graph using "rise over run".
Pick a point on the line.
Count the number of units up (the rise), then the number of units over (the run) to another
point on the line.
Divide rise by run to get the slope.
Remember, lines that go up to the right have a positive slope. Down = negative slope.
1) = 1 unit
"Rise" (from 0 up to to +1) = 4 units
(-3,0)
(1 ,1 ) "Run" (from -3 over
"Rise" = 1
=
Slope "Run" 4
To find either the x or y-intercept, you can simply count the units from the origin to the point
where the line crosses the axis.

Parallel & perpendicular lines:


Parallel lines have the same slope.
Perpendicular lines have opposite reciprocal slopes.
that is
To find the opposite reciprocal, flip If the slope ...the slope of a line is - 1
the fraction and change the sign. of a line is 3... perpendicular to it 3
m=3 m = - 31
Matching equations and graphs
Problems that ask you to match a linear equation with the graph of a line are very common. You
may be able to find the answer quickly without even looking at the actual slopes and intercepts.
Always start by checking the sign (+ or -).
Use process of elimination:
Is the slope positive or negative? Eliminate all the answers that don't match. Next consider the
y-intercept. Positive or negative? Eliminate again. You may have your answer already. If not,
take a look at the actual numbers and choose accordingly.

356
Standard Form: Ax + By = C C is a nt.
A and Bienatrse. consta
coeffic
3
3x + 2y = 16 is the same as y = - 2 x + 8

Point-Slope Form: y - y1 = m(x - x1)


(X 1, Y 1) is anynpdoinmtis
on the line a the line.
the slope of
The point-slope form expresses the fact that the difference in the y coordinate between
two points on a line (that is, y - y1) is proportional to the difference in the x coordinate (that
is, x - x1). The proportionality constant is m (the slope of the line).

If we're given m = 4 and the point (2, 3), we can use the Point-Slope Form to find the
y = mx + b form of the line:
y - 3 = 4(x - 2)
y = 4x - 5

Tables & lines


It's common for an SAT problem to give you a table of values that represents a line.
Some of these problems ask you to determine if the coordinates in the table are actually linear (form a line).
Other problems might require that you find missing values in the table.
The trick to these problems is to find the relationship between the variables in the table. Ask yourself,
"When x goes up by 1 (or some other convenient amount), what does y do?" Once you can answer that, you
can find any missing value in the table.

goes
Notice how when x by 3.
ll us that this x y up by 2, y goes up
A problem might te linear function 2 10
table represents a rm a line), and by 2 to
(the coordinates fothe value of n. 4 13 So, when x goes upcrease
then ask you for 6 n 6, y must again in n = 16.
by 3, from 13 to 16.

SAT v17.10 Unauthorized copying or reuse of any part of this page is illegal. 357
(SAT Teacher Edition)
PRACTICE WITH LINEAR PROBLEMS

1. 4.

−5

2.
2
3

3.

2
5

USE THE GIVEN INFORMATION TO FIND THE SLOPE INTERCEPT FORM OF EACH LINE
1
1. (−4, 2) m = −2 3. (5, 2), m =
5
y = −2x − 6 1
y = 5 x+1

23 −1
2. (0, 2), (7, 0) 4. (4, ), m =
3 3
y = − 2 x+2 1
y =− 3 x+9
7

358
Interpreting Linear Equations

1 (No Calculator) 3

2 (No Calculator) 4

SAT v17.10 Unauthorized copying or reuse of any part of this page is illegal. 359
(SAT Teacher Edition)
360
Linear Inequalities & Absolute Values

1 (No Calculator)

SAT v17.10 Unauthorized copying or reuse of any part of this page is illegal. 361
(SAT Teacher Edition)
INEQUALITY (N): the relationship between two
expressions that are not equal, employing a sign
such as:
≠ "does not equal"
OUR APPROACH: is a gr ea t problem for solveningwell > "greater than"
th is
If you're thinkiplngugging in the answer choices, th < "less than"
backwards by ≥ "greater than or equal to"
done!
ge t th e inequality into ma e like ≤ "less than or equal to"
ill ha ve to so
That said, we stsier to w or k with. Let's combine
form that' s ea 4≠ 5
terms. 17 > 5

5 - 2x ≥ 4 - +3 3x 600 < 601


+3x ≥ x
7≥7
5+x ≥ 4
-5 ≥ -51 6.5 ≤ 7
x≥-
Think of the inequality sign as the mouth of a
x is eq u a l or gr ea ter than -1. Thaluttion to the hungry alligator. It's always open toward the
Looks like
a ns w er ch oi ce A cannot be a so greater meal.
means that our answer!
inequality. A is

1 (No Calculator)

DON'T FLIP THE ALLIGATOR UNLESS YOU HAVE TO

Keep the inequality sign facing the same direction unless you divide or multiply both sides by a negative
number. Then, and only then, should you flip the alligator... carefully!

362
BRINGING IT ALL TOGETHER: ABSOLUTE VALUE AND INEQUALITIES
Solving an inequality that includes an absolute value can seem tricky at first. However, if you slow
down and work deliberately, you'll do fine. Check out the example below.

Ex. If |x − 4| > 2, which of the following could be the value of x?

(A) 1
(B) 2
(C) 3
(D) 4
(E) 5

1. Write out the


original equation. lx - 4l > 2
2. Remove the absolute value bars
by splitting the inequality into two
separate inequalities:
x-4>2 x - 4 < -2
The first is the same as The second has the same parts as the
the original, but with the original, but the absolute value bars are
absolute value bars removed. gone, the side opposite the absolute
value has been multiplied by -1, and the
inequality sign is flipped.

x-4>2 3. Solve 'em both:


x - 4 < -2
+4 +4 +4 +4
x>6 4. Interpret your results: x<2

x < 2 or x > 6
5. Finally, use this new information to eliminate incorrect answer choices, and then choose the
best remaining answer. At this point we can easily eliminate any answer choice equal or greater
than 2 and equal or less than 6. So, (B), (C), (D), and (E) are all toast. That just leaves (A), which is
our answer!

SAT v17.10 Unauthorized copying or reuse of any part of this page is illegal. 363
(SAT Teacher Edition)
ABSOLUTE VALUES AND INEQUALITIES GO HAND IN HAND.

Many inequality questions also require an understanding of absolute value.


With that in mind, let's quickly review absolute value and see how it relates to
inequalities.

ABSOLUTE VALUE (N): The distance between a


number and zero represented mathematically as two
vertical bars - keep in mind that because a distance
cannot be negative, the absolute value of a number is
always positive.

|4|=4 |-4 | = 4

FINDING THE DISTANCE: The distance between any two numbers can be found
by subtracting either number from the other number. Then, take whatever the answer is,
and make it positive. Simple, right? Let's check out a few simple examples that may help
you see this more clearly.
Positive!
10 − 4 = 6, so the distance between 10 and 4 is 6.

Still
Positive!
10 − - 4 = 14, so the distance between 10 and -4 is 14.

Still
Positive!
-5 − - 8 = 3, so the distance between -5 and -8 is 3.

YOUR TURN:
Find the range of possible values for the variable x in each inequality.

1. |x − 6| > 4 x > 10 or x < 2 5. -2x > 12 x < -6

2. |x + 4| < 3 -7 < x < -1 6. x + 6 > x − 2 all real numbers

3. |2x − 1| > 8 x > 4.5 x < -3.5 7. 8 − x2 > 8 all real numbers

4. x2 < -4 ≤ x ≤ 4 x<2
− 16 8. 4x > 2x2

364
SKETCH A QUICK GRAPH OF EACH LINEAR INEQUALITY
SKETCH5A QUICK GRAPH OF EACH LINEAR INEQUALITY 2
1. y ≤ 5 − 2 6. y < −
23x − 2
1.
4
1. y ≤ − 2 6.
6. y < − x − 2
4 3

5 y
2. y < −5 x − 4 7. 10x −y ≤ 15
2. y < − 4x − 4
2. 7. 10x − 3≤ 15
7.
4 3

9
3. x + 2y > −8 8. 2y ≥ 2 +9 x
3.
3. x + 2y > −8 8.
8. 2y ≥ 2 + x2
2

4.
4.
3x − 2y < 0
4. 3x − 2y < 0 9.9. y y>>|x|
9. |x|−−1 1

−2y
5.
5. 4x +
≥ 4x
≥ + 22 10.y y>>|x|x++3|3|++
10.
10. 22
5

SAT v17.10 Unauthorized copying or reuseEdition)


(Teacher
(Teacher of any part of this page is illegal.
Edition) 365
365
365
(SAT Teacher Edition)
Linear Inequalities & Absolute Values

1 (No Calculator) 3

2 (No Calculator) 4

366
Solving Linear Systems

1 (No Calculator)

/ /
0
. . . .
0 0 0

SAT v17.10 Unauthorized copying or reuse of any part of this page is illegal. 367
(SAT Teacher Edition)
HOW MANY INTERSECTIONS?

If a system of equations has no solutions, then the


lines do not intersect. That means they're parallel.
OUR APPROACH: on st ra te th e L in ear Combination
lem to dem ion Method
This is a greatuprslyob, we've demonstrated the Addvait riables. The If a system of equations has one solution, then
method. Previo be used to get rid of one of the ks with systems they do intersect.
and how it can that method is that it only worthat will cancel
limitation with at just happen to have terms If a system of equations has an infinite number of
of equations th solutions, then you're dealing with two equations
out. for the same line.
se th a t m et ho d even when the
e can u n make
Well, good news! tWbeing cooperative. In fact, we ca
equations aren' !
them cooperate
g to m u lt ip ly on e or both of the
just goin unity to
Essentially, we'renumber that will create the opepoofrtthe
equations by a ons together and cancel out on
add the equati ty simple, right?
variables. Pret

1 (No Calculator)

2x - 6y = 10 0
(3) (3x + 2y = 15) / /
9x + 6y = 45 . . . .

m u lt ip lie d on e of our equations by 0 0 0


y see, we the new
As you can probaenbld up with the opportunity to eaddy term.
3, so that we'de top equation and cancel out th
equation to th
2x - 6y = 10
+9x + 6y = 45
11x = 55
x = 5
a t w e ca n su bs titute back into
lue for x th
Now, we've got auva a tions to solve for y
.
a ny of th es e eq
2(5) - 6y = 10
10 - 6y = 10
6y = 0
y=0
368
SOLVE WITH LINEAR COMBINATION
LINEAR COMBINATION (N): x − y = 18
1. x + y = 2 x = 10 y = − 8
Step 1: Rearrange the equations so terms line up as:
x − y = 18
x+y =2
5x + 3y = 26
4x + 6y = 28
Step 2: Multiply none, or one, or both equations by
constant(s) so that the coefficients of one of the variables
are opposites.

Step 3: Add the two equations together to eliminate one of


the variables.
2. −3a + 2b = −12 a=6 b=3
(-2)(5x + 3y = 26) . 6a − 3b = 27
+ 4x + 6y = 28
-6x = -24
Step 4: Solve for the remaining variable.

-6x = -24
x= 4
Step 5: Use one of the equations and this value to solve for
the other variable.

5(4) + 3y = 26 LINEAR SUBSTITUTION (N):


20 + 3y = 26 Step 1: Solve one of the equations (your choice!) for one of
3y = 6 the variables (your choice!).
y = 2
x + 2y = 12
2x + 3y = 23

x = 12 - 2y
SOLVE WITH SUBSTITUTION Step 2: Substitute the equivalent expression into the other
equation in place of the variable you solved for in step 1.

1. −5x − y = 12 x = −2 y = −2 2(12 - 2y) + 3y = 23


. 4x + y = −10 24 - 4y +3y = 23
24 - 23 = y
1=y
Step 3: Use one of the equations and this value to solve for
the other variable.

x + 2(1) = 12
x = 10

SAT v17.10 Unauthorized copying or reuse of any part of this page is illegal. 369
(SAT Teacher Edition)
SOLVE EACH SYSTEM OF LINEAR EQUATIONS

1. x= x y=−y 1− 1 5.
. .− 3−+3 3x
+ 3x= 3y
= 3y

no solution

2. 15 = −4x − 3y
. 24 + 3y = −x

x = 3 y = −9

3. −10y + 30x = −70 6.


. 6y − 42 = 7x

x=0 y=7

$7.50

3 1
4. 0=x− y− 7.
4 2
. − 10y + 6x = 14

x = −1 y = −2

m = $2.00 36m = $72.00

370
Solving Linear Systems

1 (No Calculator) 3

/
30
/
. . . .
0 0 0

2 (No Calculator) 4

3
/ /
. . . .
0 0 0

4
/ /
. . . .
0 0 0

SAT v17.10 Unauthorized copying or reuse of any part of this page is illegal. 371
(SAT Teacher Edition)
372
Graphs of Lines

1 (No Calculator)

SAT v17.10 Unauthorized copying or reuse of any part of this page is illegal. 373
(SAT Teacher Edition)
lue
A PP R O A C H: th a t y ou 'r e su pp osed to find the va
OUR ct so.
ke su re y ou pa y attention to theyfa et , th en no w 's a good time to do
First, ma at
a - b. I f y ou ha ven't underlined th gu re out where theyem
of e're a sk ed to fi th
ve n de sc ri pt io ns of two lines, andstwmakers had just thought to oupught .
So, we've been sgiwould be pretty simple if the teke us work for this one. Fair en
intersect. Thi t form, but they're going to ma
in slope intercep (1, 5) a nd (- 1, 3) . Point-Slope form
points
ow ? The fi rs t lin e passes throughhat's next?
What do we kn hat the slope of that line is. W
should tell us w
y - y 1 = m(x - x 1)
5 - 3 = m(1 - -1 )
2 = m(2)
1=m a nd so lv e fo r th e y-intercept.
form
in m a nd pu t it in to slope-intercept
Now plug
y = mx + b
5=1(1)+b
4=b
1 (No Calculator)
ation for the
Looks like the equ+ 4.
first line is: y = x
Let's find the eq uation for the
can set the
second line, so wnde the point of
two equal to fi , b).
intersection: (a
about the
What do we know it has a slope
second line? Well, th ough (1, 3).
of 2 and passes here is the
All we're missinget's plug in x, y, t th em equal to see where
y-intercept. L for b. Easy! do
All that remainstetorsect:
is to se
and m to solve the two lines in
y = mx + b x + 4 = 2x + 1
3 = (2)(1) + b 3=x
3=2+b
w ha t th e y co or dinate is where
1=b in to see
And plug 3 in fortexrsect.
y-intercept
Note: this b is ththe e b we're the two lines in
of line #2, NOTract from a in y=x+4
looking to subt areful! So the y=3+4
the problem. C e second line in y=7
equation for th form is: e point of intersection, (a, b),
slope-intercept co or
Looks like the means:
di na te a t th
y = 2x + 1 is (3, 7), which
a - b = 3 - 7 = -4
-4 is our answer!

374
Standard Form: Ax + By = C C is a nt.
A and Bienatrse. consta
coeffic
3
3x + 2y = 16 is the same as y = - 2 x + 8

Point-Slope Form: y - y1 = m(x - x1)


(X 1, Y 1) is anynpdoinmtis
on the line a the line.
the slope of

The m (slope) racnedpt)


Slope-Intercept Form: y = mx + b the b (y-intets that
are the par hape of
affect the s
the line.

The Distance Formula: Given the two points (x1, y1) and (x2, y2), the distance
between these points is given by the formula:
d = (x2 - x1)2 + (y2 - y1)2

FIND THE DISTANCE BETWEEN EACH PAIR OF POINTS

1. (0, 8) (0, 5) 3 3. (-4, 4) (6, 7) √109 5. (1, 5√5) (3,√5) √84

2. (5, 1) (7, 2) √5 4. (2, -1) (-6, -6) √89 6. (3√3, -4) (√3, -2) 4

SAT v17.10 Unauthorized copying or reuse of any part of this page is illegal. 375
(SAT Teacher Edition)
Graphs of Lines

1 (No Calculator) 3 (No Calculator)

2 (No Calculator) 4 (No Calculator)

376
Math Unit #2
Homework Drill #1

3 (No Calculator)

2
130
/ /
. . . .
0 0 0

SAT v17.10 Unauthorized copying or reuse of any part of this page is illegal. 377
(SAT Teacher Edition)
Math Unit #2
Homework Drill #2

1 (No Calculator) 3 (No Calculator)

2 (No Calculator) 4

21
/ /
. . . .
0 0 0

378
Math Unit #2
Homework Drill #3

1 (No Calculator) 3

15
/ /
. . . .
0 0 0

2 (No Calculator) 4 (No Calculator)

SAT v17.10 Unauthorized copying or reuse of any part of this page is illegal. 379
(SAT Teacher Edition)
380
3 Rates, Ratios, & Proportions - Part 1 383

Rates, Ratios, & Proportions - Part 2 389

Percentages 393
Problem Solving & Data Analysis

Probability 399
Math Unit #3

Data Tables 403

Charts and Graphs 407

Mean, Median, Mode, and Range 413

Homework Drills:

1 2 3
418 419 420

SAT v17.10 Unauthorized copying or reuse of any part of this page is illegal. 381
(SAT Teacher Edition)
382
Rates, Ratios, and Proportions - Part 1

/
50
/
. . . .
0 0 0

SAT v17.10 Unauthorized copying or reuse of any part of this page is illegal. 383
(SAT Teacher Edition)
RATIO (N): a mathematical comparison of two
quantities, based on the operation of division
f ratios.
APPROA
CH:
b o u t a couple o
s us a ause it's
blem ask vious bec
This pro o s is o b r
ese rati 4 . That's ou
One of th
The ratio "four to seven" can be expressed as:
in th e problem: 9 r e d to the
right the
re
f m e n compa
or the r
atio o In order 4
target f in th e study.
total num
ber of s
ubjec
, th
ts
e n we mu
st achiev
e a 4:7 4 to 7 7
u e got:
to be tr . We've
for that to 5 women
4 m e n
ratio of 1
2 0 0 men = 1
men = 14
0 + 6 0 = 200 women
2 0 0
women
f men to
e t th e ratio o
going to
g to add
If we're , th e n we'll need
own to 4
:5 y do we
women d tu d y . How man
men to th
e s below.
some wo t u p a n ew ratio
s se
add? Let'
need to
1

iply when
c a n c ross-mult
Rememb
er, yo u t equal to
th a t are se
two ra ti o s st of this
4 you see a k e the re
140 + 60 = one anoth
er. That
w ill m
men = 200 + x
5
a breeze
!
women problem
ed
ing we ne
w e r is 50, mean
s to
800 + 4
x e our an in order
1000 = Looks lik
o m e n to the study we want.
to add 5
0 w
(m e n : women)
tio
e 4:5 ra
200 = 4x achieve th in
gging 50
u r a n s w er by plu e r atio
= 50 check o ally delive
rs th
50 x We can if it a c tu
d seeing
/ / for x an
e.
. . . . we desir 4
0 0 0 200 =
140 + 60 = 250
5
men = 200 + 5
0
women
THE LAW OF CROSS-MULTIPLICATION:
If two ratios are equal, then their "cross-products"
must also be equal. THE LAW OF CROSS-SWAPPING:
If two ratios are equal, then their "cross-swapped"
ratios must also be equal.
if
a = c then ad = bc.
b d a c a b
if
b =d then
c = d
384
GET TO KNOW THE TWO TYPES OF RATIOS.

There are 2 types of ratios: "Part to Part" and "Part to Whole."


They're both ratios, but there is a big difference between the two. Make sure
you know how to tell them apart, so you don't get tricked on test day!

In a certain classroom, there are 7 right-handed students


for every 2 left-handed students. We could represent this
info as several different ratios. Notice the difference.

"Part to Part" Right-handed students to left-handed students "7 to 2" or "7:2"


Right-handed students to all students "7 to 9" or "7:9"
"Part to Whole" Left-handed students to all students "2 to 9" or "2:9"

For many problems, you will need to convert from one type to the other.

Set up a table to stay organized.


Make a table with one column for each part and one column for the whole or total.
Use one row for the ratio and another for actual values.
For each row, the parts must add up to the whole. This is useful for finding missing info.
Finally, you can make a proportion out of any 4 spots that form a rectangle. Use a variable
for the missing piece, then solve.
The totaofl is
R L T th e su m the
Ratio 7 2 9 parts. 7 + = 9.
2
Actual Values 28 ?
This is whr.at we're
looking fo
R L T
Ratio 7 2 9 These 4 sp, ot s form a make a
Actual Values 28 x rectangle so we can r x.
e fo
proportion and solv

7 = 9
28 x
7x = 9(28)
x = 36

SAT v17.10 Unauthorized copying or reuse of any part of this page is illegal. 385
(SAT Teacher Edition)
These phrases mean you're dealing with a ratio:
"directly proportional" - "x is directly proportional to y"
"in a certain recipe" - "In a certain recipe, the ratio of flour to sugar is..."
"scale" - "In a scale drawing" -or- "The distance on a map" (The same goes for blueprints.)
"similar" - "ABC and DEF are similar triangles"

There are two ways to solve:


Like all ratio problems, you can make a ratio table and set up a proportion. For these problems,
OPTION #1 you don't need to worry about a total column like with the "part-to-part-to-whole" problems.
Usually there will only be 2 columns - one for each variable.
ots that
We have 4ctspang le -
If x is directly proportional to y and x = 3 when
x y form a rea propor
time fo r tion.
y = 6, what is the value of y when x = 5 ? 3 6 3 = 6
5 ? 5 y
3y = 5(6)
y = 10

OPTION #2
You can also use the "directly proportional" formula.
This formula means that for
x = (k) x y any value of x, you can multiply
by k to get the value of y.
k first.
The trick is to find

Use the gifivendn kx


and y to Then, usengk yto. find
the missi
x = (k) x y
3 = (k) x 6 x = (k) x y
k = 21 5 = 21 x y
y = 10

Other easy tips for "directly proportional":


When x is equal to zero, y must also be equal to zero.
If the equation is a function, the line must go through the origin.

386
GET TO KNOW THE UNKNOWN MULTIPLIER.
Solve the following problems by finding the unknown multiplier and then solving for the missing information.

1. The ratio of the glass beads to wooden beads 3. Virginia and Keith have been collecting donations
is 6:11 in a container of 51 beads. How many for their local soup kitchen. The ratio of Virginia's
glass beads are in the container? donations to Keith's donations is 8:5. If Keith
has collected $110.00 in donations, how many dollars
has Virginia collected?

3
U.M. = ______ 18
glass beads = ______ 22
U.M. = ______ 176
Virginia donation = ______

2. During the 2004 - 2005 session, the U.S. 4. The ratio of blondes to brunettes in a sorority is
Supreme Court reversed 16 of 19 decisions 5:3. If there are currently 56 women in the sorority,
from a circuit court. At this rate, how many how many brunettes must be invited to join in order
rulings would be reversed if the court reviewed to bring the ratio of blondes to brunettes to 1:1?
114 cases?

reversed
6
U.M. = ______ cases 96
= ______ 7
U.M. = ______ 14
brunettes = ______

TRICKY TRIPLE RATIOS: Sometimes you'll There is, of course, more than one method to solve
be given two separate ratios and asked to find a this problem. Use whichever method you find more
third one based on the first two. This can be tricky, comfortable.
but it's doable if you're careful. For example:

If the ratio of b to c is 6 to 5, and the ratio of a to b If the ratio of b to c is 6 to 5, and the ratio of a to b
is 7 to 3, what is the ratio of c to a? is 7 to 3, what is the ratio of c to a?
Method #1 Method #2
1. Use translation to carefully set up the ratios: 1. Use translation to carefully set up the ratios:
b = 6 and a = 7 b =
6 and a = 7
c 5 b 3 c 5 b 3
2. Multiply the ratios together: 2. Cross multiply and isolate the variable b that you
b x a = ba = a can later substitute into the other equation:
c b bc c
b= 6 6c + 5b b = 65 c
3. Do the same with the numbers above: c 5
a= 7
6 x 7 = 42 = a b 3 3a + 7b
5 3 15 c
4. Flip the fraction, because we're looking 3. Plug it in and solve:
6
for ( ac ), not ( ac ). 3a = 7( 5 c )
3a = 42
15
c
15 c
42 = a = the ratio of c to a a = 42 c a = 42
15 c 15
5. Finally, we need to reduce the fraction: 4. Finally, reduce the fraction:
15 5 a = 42 = 14
42 = 14 c 15 5

SAT v17.10 Unauthorized copying or reuse of any part of this page is illegal. 387
(SAT Teacher Edition)
Rates, Ratios, and Proportions - Part 1

1 (No Calculator) 3 (No Calculator)

2 4

/
44
/
. . . .
0 0 0

388
Rates, Ratios, and Proportions - Part 2

/
45
/
. . . .
0 0 0

SAT v17.10 Unauthorized copying or reuse of any part of this page is illegal. 389
(SAT Teacher Edition)
RATE (N): A rate is just a ratio of related qualities that
have different units. Use this formula for any problem
that asks you how far someone traveled in a given time
(or how many of something were produced in a given
OUR APPROACH: disguised as a dirm fficult time) at a given rate.
m pl e pr ob le m to
This is a si
he ck it ou t. U se the rate fo ulaxt page.
problem. C e ne work = rate x time
ep th in gs si m pl e. It's explained onndtha distance
ke fi
Remember that ta it can be used toork completed.
l amount of w
traveled or a to
e in fo rm a ti on th at the problem Work = (rate) x (time)
ll th
Once you plug in ahave to do is solve for the missing
ou
provides, all y 're done! 25 miles
variable, and you 100 miles hour 4 hours

10 Pies 2 pies 5 weeks


week

Work Rate Time


45
Roy's
Work ? = 50 words
1 minute
x 30 minutes
.
/
.
/
. .
0 0 0
+

Nelson's
Work ? = 70 words
1 minute
x 30 minutes

ta l nu m be r of words typed. s
=

e to
BEWARE! This isfith
nd th e to ta
aph
l number of paragr, the
The sum of to
We're asked ed to divide this number by 80
Nelson's and typed, so we ne per paragraph.
Roy's Work
number of words
0 words = 45
Numberraopfhs = 360 80 !
Parag 45 is our answer
YOU ALREADY KNEW ALL OF THIS
Seriously, you did. If I ask you how far someone travels in 2 hours at a rate of 20 miles per
hour, you'd know the answer immediately. You'd simply multiply 20 miles per hour times
2 hours. Bingo! 40 miles. It's not any more difficult when I ask you how many miles you
travel in 7 hours at a rate of 43 miles per hour. You'd use the same setup as above!

390
the PAYDIRT AND MORE
a t on e ty pe of rate problem, buattion
looked form The "dirt" formula is a handy mnemonic
So far, we've onulyp and ask you for other kinds ofbeinfore the sun device to help you remember one version
SAT can mix it ust Bill travel in order to arrive with steamroller of the rate formula, but you can set it up in
like, "How fast m ng until cement-mixer A collides C produce in 3 a variety of ways to isolate other variables:
rises?" or "How lo more waffles can waffle-makerhours?" Sounds
B?" or "How many le-maker D can produce in 5.25
hours than wafft? d=rxt
complicated, righ ed using the same distance = rate x time
le m s ca n be so lv
of these prob re out which
Luckily for you, arell d on the previous page. Just findguplug them in. rate = distance
formula we covermula are given in the problem, a ece. Finally, reread time
pieces of the fo ve to do is solve for the missing pi e correct piece of distance
time =rate
Then, all you hamake sure you have solved for th get tricked!
the problem to on. And be careful! It's easy to
missing informati

SET UP THE PROBLEM AND SOLVE FOR THE MISSING INFORMATION

Sean-Patrick and Rob decide to have a race


E
STEPToONsolve
between two lifeguard towers on the beach. this version, we need to begin by solving
Sean-Patrick runs at 10 meters per second and
for t, the amount of time it took Sean-Patrick to
Rob runs at 8 meters per second. If the towers
run the race. Then we'll use that information to
find out how far Rob ran in the same amount of
are 400 meters apart, by how many meters does time. Finally, to find the distance Rob lost by, we'll
Sean-Patrick win the race? subtract the distance Rob covered from the total
length of the race, 400 meters.
Distance Rate Time
Sean-Patrick finishes the race
400
meters = 10 meters
per second
x t in t seconds, so let's solve for
the missing variable. Looks like
10(t) = 400, so t = 40. Now we
plug that number in to see how
far Rob ran in the same amount
of time.
400
meters = 8 meters
per second
x
t
TRY THIS ONE:
STEP TWO
Sean-Patrick finishes the race in t seconds, so
let's solve for the missing variable.
Looks like 10 x 40 = 400, t = 40. Now we plug 14 min.
that number in to see how far Rob ran in the
same amount of time. If t = 40, then 8(t) = 320.
Is this our answer?
Careful! 320 meters is the distance Rob ran, not
the distance by which he lost! So, we still need
to subtract 320 from 400, which gives us our
answer, 80 meters.

SAT v17.10 Unauthorized copying or reuse of any part of this page is illegal. 391
(SAT Teacher Edition)
Rates, Ratios, and Proportions - Part 2

1 (No Calculator) 3

2 (No Calculator) 4

392
Percentages

SAT v17.10 Unauthorized copying or reuse of any part of this page is illegal. 393
(SAT Teacher Edition)
OUR APPROACH: m do es n't give you real choose
PERCENTAGE (N): an expression of a
number as a fraction of 100
ge pr ob le
If a percenta k with, choose your own! But mple
numbers to wor so you can keep the problem as sit
them carefully osing 100 is almost always a grea 10
as possible. Cho ge problems. 10% = 100 = .10
idea on percenta
50
50% = 100 = .50
Original price: $100.00
(we chose this)
25
Opening Night Price: $100 + (.10)(100) = $110.00 25% = 100 = .25
(original price plus
10% of the original price)

Discounted price: $110 - (.2)(110) = $88.00


(opening night price minus
20% of the opening night price)
Divide the discounted price by Discounted Price = 88 = 88%
the original price to find what Original Price 100
percentage of the original price
the discounted price represents. 1
Reread that last part a few
times to make sure you've got
the idea.

add or
NEVER EVER EVERrcentages.
subtract two peand subtract
If you add 10% tricked and
20%, you'll get 90. This is a
end up with (A), wer.
classic Trap Ans

I'D HATE TO SEE WHAT THEY CHARGE FOR POPCORN!!


Keep it simple, and just pick 100 bucks for all prices on percentage
problems. Do movie tickets really cost $100.00? Of course not, but
that doesn't matter. Keep it simple and charge $100.00 for everything.
You're not trying to develop a business strategy here. You're trying to
ace the SAT.

HOW MUCH IS THAT HAMSTER IN THE WINDOW?


The original price of a hamster is lowered by 50% due to a
surplus of hamsters, and then raised 300% due to a shortage of
hamsters. What percentage of the original price is the final price?

394
SHORTCUT CITY!
In the problem on the previous page, we kept it simple by choosing 100 as the price. On this page,
we'll examine another way to keep it even simpler! We can skip a step if we just remember that
100 percent means "per 100."

If, for example, a price is raised 10% then we've got two ways to find the new price:

OPTION #1 New Price = (100% of the old price) + (10% of the old price) = 110% of the old price.

That's the first way, but you can keep it even simpler by combining like terms, like so:
OPTION #2 New Price = (110% of the old price)

And using translation, you can immediately see the equation:

New price = (1.10)(old price) lo ok s lik e, w hi ch makes perfect


This is what 110% think about it.
sense, when you
ld pr ic e) = ol d pr ic e. Easy, right?
1 (o
x
e) + .10 x (old price)] , which
x (o ld pr ic
1.10 x (old price) = [1e old price. Still easy, right?
equals 110% of th

It works the other way too. If the price of an item goes down by 20%, then you can make it easier by
focusing on the part of that original price that you're still paying. Don't bother taking the original price
and subtracting 20% of the original price from it. Just multiply the original price by the percentage of
it that you're still paying, 80%.

If, for example, a $100.00 lamp goes on sale for 20% off, you'll pay 20% less than the original price.
Instead of finding 20% of the original price and then subtracting that from the original price, just skip a
step and find 80% of the original price. I know what you're thinking, "does it work with foul-tempered
farm animals?" The answer is yes! It most certainly does! Read on, my friend!

A cow was originally on sale for $510.00, but


due to her aggressive temperament, the price has is multiply
been lowered by 30%. What is the new price of All you need toicdo .70, the
the cow? the original pre eorbyin al price
amount of thctuaiglly be paying).
that you'll a
$510.00 x (.70) = $357.00

THE PERCENT CHANGE FORMULA


Use this formula to answer questions that ask you to find what percent of an original price is
represented by the new price.

Change in Value = (New Value) − (Original Value) x 100 = Percentage Changed


Original Value Original Value from the Original Value

SAT v17.10 Unauthorized copying or reuse of any part of this page is illegal. 395
(SAT Teacher Edition)
TRANSLATING PERCENT PROBLEMS
Good translation is the key to success on percent problems. Remember that percent simply means
"divided by 100." Got that? Well then check out this chart.

Phrase What to Do Math Example


n
"Percent" or "%" divide by 100 100 "n percent"

"of" multiply (.10) x 50 10% of 50

"increase" multiply the initial value by:


"8% more than n"
or n x (1.08) or
"more than" (1 + the
100
%
) "n increases by 8%"
multiply the initial value by: n x (1 - .60) "60% less than n"
"decrease"
or or
"less than" ( 1 - the
100
%
) n x (.4) "n decreases by 60%"

"some .75n
number" choose a 75 "75% of
variable (n) x 100 some number"

THE PERCENT FORMULA

Whole X Percent = Part

A bag contains 200 marbles, of which 23 percent are blue. How many blue marbles are in the bag?
200 x (.23) = b
46 = b

BE 100% CAREFUL, NOT 10,000% CAREFUL.


Careful when you convert percentages into decimals. It's
very easy to miss problems by forgetting to divide the
percentage by 100 when you convert it into a decimal.

100% = 1.00
10% = 0.10
1% = 0.01
0.1% = 0.001

396
PRACTICE USING YOUR COMMON (PER)CENTS
Translate each of the following into an equation and then solve.

1. What is 20% of 15.2?


3.04

2. If Dan answered 18 out of 25 X% OF Y = Y% OF X


questions correctly, what percentage How does this help?
of the questions did he answer
incorrectly? Well, funny you should ask,
because the world can be a
28%
pretty unpredictable place, and
sometimes it's much easier to
3. 10.4 is what percent of 80? find x% of y than it is to find
y% of x. For example:
13%
1 If I asked you to find 4% of 25,
4. What is 6 % as a decimal?
4 you'd probably reach for your
.0625 calculator. But suppose your
calculator had been eaten by a
5. .001 is what percent of 10? moose.

.01 Then what?

Well you can forget about your


9
6. What is as a percentage? calculator, but that doesn't mean
200
you've got to give up on the SAT
4.5% problem. Using the formula
above, you don't need even a
7. A share of stock is now worth 20% calculator!
of its value from one year ago. If it is
now worth $44,250, what was it worth Just switch the x (4) and the y (25)
a year ago? and ask yourself, what's 25% of 4.
$221,250 That's easy - 25% of 4 is 1!

8. Marge dropped from 160 pounds


down to 128 pounds on her fad diet.
What percent of her old weight is her
new weight?
80%

SAT v17.10 Unauthorized copying or reuse of any part of this page is illegal. 397
(SAT Teacher Edition)
Percentages

1 (No Calculator) 3 (No Calculator)

2 (No Calculator) 4

398
Probability

1 (No Calculator)

SAT v17.10 Unauthorized copying or reuse of any part of this page is illegal. 399
(SAT Teacher Edition)
PROBABILITY (N):
OUR APPROACH: rw ard concept. In ob order to the likelihood that a desired outcome will
Probability is a praet
ty st ra ig ht
A
fo
T of te n co m bi ne s pr ability witesh occur, determined by the formula:
nging, the S oblem requir
make it more ch itlleional concepts. For example, thisleprcting a girl over
one or more add t the probability of randomly ses class.
you to figure ou among all the girls in Mrs. Smith' # of desired outcomes
4 feet tall from
m be r of desired results # of possible outcomes
y th e nu
'll recall, is simpl se, we're
Probability, as youmber of possible results. In thislecacting among Matt flips a dime. What is the probability
divided by the nugirl over 4 feet tall and we're se to miss this that it will land "heads" up?
trying to find a e class. Careful though, it's easy g among all the
all the girls in th isread it and think we're selectin
problem if you m e class. heads
STUDENTS in th
n th e to ta l nu m ber of girls in theare
# of desired outcomes
# of possible outcomes = = 21
rectly give girls who
Now, we weren'tadit, we've got to add the number ofgirls who are heads tails
class. To find than 4 feet tall (9) to the number of4 girls in the class!
equal or less th tall (15). Looks like there are 2
more than 4 feet
1 (No Calculator)

a ny of th em a re over 4 feet tall?tion


wm rma
ve go t 2 4 to ta l gi rls in the class. Ho4ft tall. Now we've got all the info
So, we' >
art, 14 of them are
According to our ch
we need. is
15 , which reduces
tal po ss ib le re su lt s
ti o of ou r de si re d result to the to 24
The ra
to 5 .
8 : 5
That's our answer 8
THINK OF PROBABILITY AS
A PART-TO-WHOLE RATIO
400
INDEPENDENT EVENTS
Two events, A and B, are independent if the fact that event A occurs does NOT affect the probability that B will
occur. For example, a coin-toss in one room does not affect the probability that a six-sided die will roll an odd
number in an adjacent room.

Each day for five days, Luis wears a hat from his
collection of five hats. Assuming Luis wears exactly
one hat per day so as to avoid being mistaken for a
lunatic, how many different combinations of hats
could be worn over the five-day period?

5 x 5 x 5 x 5 x 5 = 3125
t
of hi s 5 hats on any of Let's just say thtaa
ea r a ny
Luis could w re are no restrictions, and the dude has go
the 5 days. Theer which hat he wore the few hat options!
it doesn't matt hese events are independent.
previous day. T

DEPENDENT EVENTS
Two events, A and B, are dependent if the fact that A occurs affects the probability that B will occur.

For example, a card is pulled at random from a deck of five cards. Without replacing the first card, a second card
is chosen. These are dependent events because the probability of pulling a particular card on the second pull
has increased from one-in-five (20%) to one-in-four (25%).

Each day for five days, Tyler wears a different scarf


from his collection of five scarves. Assuming Tyler
wears only one scarf per day so as to avoid being
mistaken for the frontman of a popular 90's rock
5 x 4 x 3 x 2 x 1 = 120

group, how many different combinations of scarves


could be worn over the five-day period? Not nearly
enough scarf
options!

SAT v17.10 Unauthorized copying or reuse of any part of this page is illegal. 401
(SAT Teacher Edition)
Probability

1 (No Calculator) 3

2 (No Calculator) 4 (No Calculator)

30
/ /
. . . .
0 0 0

402
Data Tables

California Texas Total


Support 659 194 853
Oppose 186 499 685
Total 845 693 1538
Respondents for and against Measure 994

SAT v17.10 Unauthorized copying or reuse of any part of this page is illegal. 403
(SAT Teacher Edition)
OUR APPROACH: is pr et ty si m pl e. T he test makers
rt
tables on the SAT me-consuming to so DATA TABLE (N):
Working with dagitave you anything very difficult orpetiof problem because A method of displaying relational data in
aren't going to said, many students miss this ty s. cells, which are arranged in columns and
through. That quickly and make careless mistake rows. These tables are used to provide a basic
they work too re y ou know EXACTLY picture of the interrelation between two
is to m a ke su that
The key to avoidon ing those mistakends -- and EXACTLY where to find of those variables and can help highlight interactions
asks you to fi you know both between them.
what the questi k slowly until you are certain that
information. Woration. Two-way charts typically include descriptions
pieces of inform ps of respondents inmber
of each column and row at the top and left
ch of th e gr ou side. Totals for each column and row are
e're asked whi of the total nu
In this question,cews represents approximately 45%lem, we first need to generally found along the bottom and right
the answer choi dents. In order to solve this probs. side of the table.
of survey respon l number of survey respondent
identify the tota er choice representsto. tal
ce ll ea ch a ns w
n, w e ne ed to fi gure out which bers in each of those cells by the %
The to divide the numhich one gives us approximately 45
Finally, we'll need de nts and see w
number of responthe decimal .45).
1
(represented by
California Texas Total
Simple, right? Support B 659 194 853
Oppose 186 A 499 685
Total D 845 C 693
Respondents for and against Measure 994
1538
TOTAL

m be r of re sp on de nts from Californiwaer


l nu e ans
r of re sp on de nt s will include the toerta: 1538. Now, let's test some of tth45% of
The total numbe's got to be the bottom right corn ore we just dive in, consider thaber that's a
and Texas, so it hich one gives us 45% of 1538. Befso we should be looking for a num
choices to see wbe a little less than 50% of 1538,
1538 is going to 1538/2 = 769 Keep that in mind. %
little less than su re . 49 9/ 15 38 = approximately 32 e th an
so m e ke
math to ma oice D is way too big. It's m get or
all, bu t le t' s do wer ch C, w e
A is clearly too smchoice A. On the other end, ansha t ju st le a ve s B and C. Trying
w er
Eliminate ans we can eliminate that one too. r answer! T
50% of 1538, so ximately 45%. Looks like C is ou
404 693/1538 = appro
PRACTICE WITH DATA TABLES
Sometimes you'll be given a two-way table that lacks important information. In these cases, you'll
need to use the information you've got to identify the information you need to know in order to solve
the problem. Take a look at the example below.

LET'S EXAMINE THE DATA


1. How many total attendees did the theme park employee observe?
2050
2. What was the average (arithmetic mean) cost, in tickets, for an attendee to visit these attractions?
8
3. If an attendee were to visit each attractions twice, how much would it cost the attendee, in tickets?
64
4. If tickets can be purchased in packages of 10 for $0.50 per ticket, then what is the cost, in dollars,
to visit the haunted house?
$3
5. If the theme park makes $0.25 profit on each ticket sold and has the same operating costs for each
attraction, then how much more revenue was generated by the Roller coaster than the haunted
house?
$722.50
6. If each attraction costs $200 per day to operate, and the data above was collected during a single
day, then how much money, in dollars, did the theme park earn from the sale of tickets to these
attractions? (Assume all tickets were purchaed for $0.50 each and all tickets sold were used.)
$5152.50
7. Approximately what percentage of the tickets collected were collected at the Roller Coaster?
40.66%
8. If a ticket is chosen at random from all those collected, what is the probability that this ticket was
collected at the log ride?
14.11%
SAT v17.10 Unauthorized copying or reuse of any part of this page is illegal. 405
(SAT Teacher Edition)
Data Tables

1 3

Completed Did Not Complete


Obstacle Course Obstacle Course
Trained Doing 67 133
Pull-ups
Did Not Train 18 182
Doing Pull-ups

2 4

Gender
Grade Level Male Female Total
Elementary 27 30 57
Junior High 14 26 40
High School 3 20 23
Total 44 76 120

1/2
/ /
. . . .
0 0 0

406
Charts and Graphs

1 (No Calculator)

SAT v17.10 Unauthorized copying or reuse of any part of this page is illegal. 407
(SAT Teacher Edition)
OUR APPROACH: sh ow th e relationship betw
een
LINE CHART (N):
SAT ty pi ca lly proba on't
bl y w
Line Charts on there time. As with data tables, youT, so the key to A line chart or line graph is a type of chart
two variables ovrticularly complicated on the SA our time and make which displays information as a series of
data connected points. This type of chart is
see anything pae questions correctly is to take y common on the SAT. Line Charts are most
answering thes exactly what's being asked. commonly used to display the change in
sure you know many
qu es ti on th a t' s pretty easy, buthout values over time.
ample of a ickly wit
This is a great exs because they try to answer it quasking them to
students will mis ing what the question is actually
fully understand
find. and
e ov er a ll di ff er en ce between priva,tebut that's
th 0 15
It's obvious that is greatest between 2005 anda2t trap, read the
de bt
government asked to find. If you fell fo r th
not what we're e time and try again.
question one mor

1 (No Calculator)

25% to 25% 80% to


is a ratio In 1995, we've gotto 1 ratio.
of 1 to 1. 20%. That's a 4CREASES
The RATIO IN to a 13 to 1
over the decadeprivate debt
ratio when the vernment
hits 130% and go . That's the
debt falls to 10% , so C is our
biggest increase
answer!
The word
RATIO is
the key to t de bt a re bo th about 25% of GDtiP.o
this problem! bt and governmen t ra
of nearly 1 to 1. Tinha1985 looks
In 1965, private de the same, th en it 's a ra ti o
he ratio
If they're abou1tto 1 in 1965 to about 2 to 1 in 1975. T. But from 1995 to
increased from 1 and it increases to 4 to 1 in 1995 fell from around 20%
to be about 3 too EXPLODES as government debt ed from 80% of GDP to
2005, that rati GDP and private debt skyrocket t's our answer.
of GDP to 10% of t gives us a ratio of 13 to 1. Tha
130% of GDP. Tha l the
oi ce D y ou a sk ? Good question! Wivelate debt to
er ch of pr
What about ansbowut the INCREASE IN THE RATIaOctually goes DOWN
question asks a . If you look closely, the RATIOround 20% of GDP while
government debtment debt goes up from 10% to a rted in 2005. As a
because govern 2015 ends up about where it staund 7 to 1.
408 private debt in actually goes back down to aro
result the ratio
PRACTICE WITH PIE CHARTS
Just as the SAT will sometimes give you two-way tables that lack important information, it will
also give you charts and graphs that require you to work out some important information before
attempting to solve the question directly. Let's take a look at this one for a bit more practice.

LET'S EXAMINE THE DATA


1. Which of the categories was responsible for the most work place injuries in 2010?
automobile accidents

2. If 40,000 people were injured in automobile accidents, how many were injured in fires and
explosions?
18,000

3. If the sector of the pie chart representing giraffe attacks were to be extended to the center of
the circle, what would the measure of its central angle be?
54°

4. If the number of workplace injuries attributable to environmental factors were to be reduced by


half, how many lives would be saved?
2000

5. If the data were to be reorganized to omit the injuries resulting from equipment failure, what
percentage of the new total number of workplace injuries would be attributable to falls?
10.5%

SAT v17.10 Unauthorized copying or reuse of any part of this page is illegal. 409
(SAT Teacher Edition)
MORE PRACTICE WITH CHARTS.
A scatterplot is a great way to get a general sense of the data without getting bogged down in a bunch of
math. You can usually see the relationship between the variables just by glancing at the scatterplot. In that
sense, scatterplots are like the opposite of a data tables, which provide very detailed information but require
you to crunch numbers before you can really understand it. With that in mind, most SAT questions dealing
with scatterplots focus on the general relationship between the variables as opposed to the math behind that
relationship. On the occasions you are asked for a specific number as an answer, you'll usually have a line of
best fit to use as your guide. It's important that you know how to use a line of best fit.

#1
Exam Score (% correct)

#2 #4

Time Studying (hours)

The scatterplot above shows information about students in an organic chemistry class. The x-axis represents
the number of hours the student spent studying. The y-axis represents the percentage of questions answered
correctly on the final exam.

LET'S EXAMINE THE DATA.


1. Find the mark that represents the student who studied the most.

2. Find the mark that represents the student who studied the least.

3. How many hours were spent studying by the student who scored the nearest to 70%?
15 hours
4. Identify the outlier.

5. If the outlier data point were to be removed from the dataset, what would happen to the slope of the line
of best fit?
It would become slightly steeper.
6. What is the approximate slope of the line of best fit?
Approximately (95 − 55) / (30 − 0) = 40/30 = 4/3

410
Charts and Graphs

3 (No Calculator)

1 (No Calculator)

2 (No Calculator)

SAT v17.10 Unauthorized copying or reuse of any part of this page is illegal. 411
(SAT Teacher Edition)
4 (No Calculator) 6 (No Calculator)

5 (No Calculator) 7 (No Calculator)

2100
/ /
. . . .
412 0 0 0
Mean, Median, Mode, and Range

1 (No Calculator)

SAT v17.10 Unauthorized copying or reuse of any part of this page is illegal. 413
(SAT Teacher Edition)
ARITHMETIC MEAN (N):
OUR APPROACH: ve y ou to o
keep
much trouble if yrouof broken
a measure of average calculated by dividing the sum
of the values by the number of values in the set
ld n' t gi mbe
This problem shou e m ea n is si m ply the averagesunurvey.
in mind tha t th the
s su st a in ed by the respondents of mean = sum of values
number of values
bone oken
, si m pl y fi nd th e total number of brIn
s.
To find the averadige vi de by
ondent
the number of rengsp, so don't forget
th en {9, 12, 8, 11}
bones and st og
adi
ram could be mislenes by the number of
this ca se , th e hi
mber of broken bo oken
to multiply each nueach sustained that number of abrt they'd mean = 9 + 12 4+ 8 + 11 = 10
respondents who ple, five respondents reported th at's 5 x 2
bones. For exam bones during their careers, so th
had two broken t five or two.
broken bones, no 1 (No Calculator)

ve got:
th ro u gh th e m a th, it appears we'5) + (1 x 6)
Going x 3) + (2 x 4) + (3 x
(3 x 1 ) + (5 x 2) + (1
So that's
+ 8 + 15 + 6 = 45 total broken bones.
3 + 10 + 3
vide
w e' ve go t 15 to ta l respondents, so di
And s our answer!
MEDIAN (N): a measure of average that basically 45 by 15 and that'
.
translates to "the one in the middle." To find the median,
nes per respondent
just put all the numbers in order from least to greatest and 45/15 = 3 broken bo
then cross off one on each end until you have only one value
remaining in the middle.

{3, 19, 21, 6, 11} the numbers


{3, 6, 11, 19, 21}
Remember to puatst to greatest
in order from le rt crossing them off,
before you sta n't actually finding
or else you are
the median.

414
BE AN AVERAGE STUDENT: FIND THE MEAN, MEDIAN, AND MODE OF EACH SET.

1. {4, 14, 6, 6, 5} 3. {3, 4, 1900, 3, 1900}

Mean: 7
_____ 6 Mode(s): _____
Median: _____ 6 Mean: 762
_____ 4 Mode(s): 3,
Median: _____ 1900
_____

2. {200, 33, 19, 66, 57} 4. {7, 7, 7, 7, 7, 7, 14}

Mean: 75
_____ 57 Mode(s): _____
Median: _____ none Mean: 8
_____ 7 Mode(s): _____
Median: _____ 7

NOT YOUR AVERAGE AVERAGE PROBLEMS:

{x, x, 4x} 3. The sum of 5 consecutive integers is 405, what is the


1. Find the mean, median, and mode of the above set. median of these integers? What is the mean of the
integers? What is the greatest of the integers?

Mean: 2x
_____ x Mode(s): _____
Median: _____ x
Mean: 81
_____ 81 greatest: _____
Median: _____ 83
2. Ernest scored 79, 81, 81, 88, 91, 94, 90, 77, and 84
on a series of exams. Find the mean, median, and
mode of Ernest's scores.
Watch out for problems that ask you about the sum
of consecutive integers. On this type of problem, like
the one above, you can find the average (arithmetic
mean) by simply dividing the sum of the consecutive
numbers by the number of consecutive numbers.
Mean: 85
_____ 84 Mode(s): _____
Median: _____ 81 Know what else? The mean will always be the same
as the median on this type of problem.

MODE (N): The mode of a set of numbers is the one that


occurs most often. So, in the set {1,5,7,5,9}, the mode is 5
because there are 2 fives and only one of each of the others.

RANGE (N): The difference between the lowest and


highest values. In {4, 6, 9, 3, 7} the lowest value is 3, and the
highest is 9, so the range is 9 − 3 = 6. Range can also mean
all the output values of a function.

SAT v17.10 Unauthorized copying or reuse of any part of this page is illegal. 415
(SAT Teacher Edition)
GET TO KNOW THE AVERAGE FORMULA AND THE ANT TABLE
The average formula that you learn in school (add up the values and divide by the number
of values) may be occasionally useful, but it can get you into trouble on the SAT.
Instead, use the ANT formula for most average problems.

Average x Number of Items = Total of the Items


Especially for more complicated ANT problems involving multiple groups with separate
averages, it can be helpful to keep track of your work in an ANT table.
Multiply theesero#w's.
Ax N = T to get th
If 6 items in group #1 have an
average of 9 and 3 items in group Group #1 9 6 54
#2 have an average of 12, when Group #2 12 3 36
both groups are combined, what is Combined 10 9 90
the average? Add the colu#m'sns. to
get these
Divide thaensrowwer.to
NEVER Average Averages! get the
I repeat, never average averages! When you are given averages for two separate
groups, you CANNOT take a simple average of the two averages. This is a very common
SAT trap.
If one group has an average of 10 and another group has an
average of 20, when you combine the two groups, the combined
average does NOT NECESSARILY equal 15. Don't makettahkise!
common mis
*The average depends on the size of each group.

Group #1 Group #2 Groups 1 & 2 20 + 10 = 15


Avg = 2
Avg = 20 Avg = 10 Combined

416
Mean, Median, Mode, and Range

2 (No Calculator)

3 (No Calculator)

Gold and Silver Medals for Men's Hockey

1 (No Calculator)

4 (No Calculator)

SAT v17.10 Unauthorized copying or reuse of any part of this page is illegal. 417
(SAT Teacher Edition)
Math Unit #3
Homework Drill #1
1 (No Calculator) 3

2 (No Calculator) 4

418
Math Unit #3
Homework Drill #2

3 (No Calculator)

1 (No Calculator)

2 (No Calculator) 4 (No Calculator)

3000
/ /
SAT v17.10 Unauthorized copying or reuse of any part of this page is illegal.
. . . . 419
(SAT Teacher Edition)
0 0 0
Math Unit #3
Homework Drill #3
1 3 (No Calculator)

2 4

420
4 Roots & Exponents 423

Operations with Polynomials - Part 1 429

Operations with Polynomials - Part 2 435


Passport to Advanced Math

Solving Quadratics 441


Math Unit #4

Graphing Quadratics (Parabolas) 447

Function Notation 457

Real-Life Nonlinear Models 465

Homework Drills:

1 2 3
468 469 470

SAT v17.10 Unauthorized copying or reuse of any part of this page is illegal. 421
(SAT Teacher Edition)
422
Roots & Exponents

1 (No Calculator)

SAT v17.10 Unauthorized copying or reuse of any part of this page is illegal. 423
(SAT Teacher Edition)
EXPONENT (N): the smaller number that's above and
to the right of the base; "the power" to which the base is
raised; the number of times that the base is multiplied by
itself

BASE (N): the larger number below the exponent. This


is the number "being raised to a power."
OUR APPROACH: need to know a fetest day.
w
w s, fr ie nd : y ou 'll l on
Bad ne ou want to do wel COEFFICIENT (N): a multiplier placed before the
exponent rules ifis ythat it's not like these rules variable in an algebraic expression
The good news ulas, encrypted and written e
are esoteric formvisible ink, locked in a vault on th ou
backwards in in moon - not anymore anyway. Y EXPONENT
dark side of thend retrieved them last summer.
see, we went a
ty thoroughly on thfee w
5y3
ex pl a in ed pr et
You'll find them 're welcome. So, memorize these
next page. You re test day. No excuses! BASE
simple rules befo COEFFICIENT
la r pr ob le m , y ou must know thatme
u
For this partic ponent terms that have the sa
when dividing ex subtract the exponents.
base, you must
1 (No Calculator)

de d sh a re a co m mon base, so all


ing divi e
(A) is a TRAP ANpSW
ER! You'll The two terms abeccording to the rules, is subtract th
nswers based we need to do,
always find tra exa ponent rules. exponents.
on breaking thetrap any student
This one would exponents instead 96m 6- m=m
who divides the them. 9
of subtracting +m + m
6 = 2m
3=m ck
ch ec k y ou r a ns wer, just plug it bances.
And if you want toto make sure the whole thing bala
into the equation
963 = 531441
729 = 729
9
Looks like this
one checks out!

424
EXPONENT RULES

When...
multiplying exponential
. . ..
x2 x3 = (x x)(x x x) = x2+3 = x5
terms with the same base... Add the exponents.

n = (n.n.n.n.n.n) = (n/./n.n/.n/.n.n) = n
n (n.n.n.n) (n/./n.n/.n/)
6
2
When... 4
dividing exponential
terms with the same base... Subtract the exponents.

When...
one exponential term is the base
of another exponential term
()
y = (y .y.
32
y)(y .y.y) = y ( )=y 3.2 6

(a power is raised to a power)... Multiply the exponents.

2
3
When... 27 = (27)2 = 729 = 9
3 3
an exponential term is raised
to a fraction... The top of the fraction acts like an exponent,
and the bottom of the fraction acts like a radical.

-1 -2
x
x= =-1 1 3
3 = 1 = 31 2 = 91
-2
When... 1 x
an exponential term is raised to a Just flip the fraction, and get rid of the negative sign. "What to a
fraction," you ask? Remember that x-2 is the same as x , so flip it,
2
negative number...
and get rid of the negative sign. 1

When...
n1 = n 121 = 12
an exponential term is raised
to the power of 1... The base is unchanged.

When... n0 = 1, 80 = 1 and (-21)0 = 1


an exponential term is raised
to the power of 0... The result is always 1.

SAT v17.10 Unauthorized copying or reuse of any part of this page is illegal. 425
(SAT Teacher Edition)
A POWER-FULL EXPONENT REVIEW
Dissect each term into it's basic parts.
NEGATIVE
EXPONENTS
Coefficient Base Exponent Simplify each expression.
1
3y5 3 y 5 1. x -1
x
= ______
1
mn-2 m n 16
-2 2. 4-2 = ______
1
( 19 )
-2
1 125
3. 5-3 = ______
1 -2
9
9
2 -2
4. ( ) = ______
4
3
x4
2
5. ( )
-4
16
= ______
x
SIMPLIFY YOUR LIFE
Simplify each of the following expressions.

6. a2b − 6a2b = -5a2b


____________ BE A PROBLEM SOLVER.
n
12. If (300)(2000) = 6 x 10 , then n =
A) 6
B) 5
C) 4
7. 6x3 + 2x4 + 2x3 = 2x + 8x
4
____________
3
D) 3

-6x2
13. If x = 2.4, then (-6x)2 =
8(21)15 A) -1
8. 2(21)11 = 777924
____________ 1
B) -
6
1
C) -
36
1
D)
6
9. ((2)17 + (3)17) + ((2)17 − (3)17) = 262144
____________ 1

14. If t is a positive integer, which of the


t t t t
following is equivalent to 4 + 4 +4 +4 ?

10. (2)2f(4)2f = 2 6f
____________ t+1
A) 4
4t
B) 4
4t +1
C) 4
t
D) 16
4t
16
3(6 y )
24(6x)y
11. = 2y
____________
8(2x)y
-n3
15. n()
-4 -3
64
= ______

426
COVERING YOUR BASES: EXPONENTS WITH MISMATCHED BASES

MATCH-MAKING 101:
When the bases don't match, the answer often lies in using substitution to make
the bases match. Observe:

If 32x = 27(x − 1), what is the value of x?

A) 1 STEP 1: Note that the bases don't match. Ask yourself how
B) 2 you could use substitution to make them match.
C) 3
D) 4 STEP 2: Substitute (33) in for 27.
32x = 27(x - 1) 32x = (33)(x - 1)
STEP 3: Now that the bases match, we can forget about
STRATEGY CHECK them entirely and create a new equation by setting the
We've solved this problem on exponential expressions equal to one another and solving for x.
the right using algebra, but Remember to follow the exponent rules and distribute the 3
you could probably solve it to the x and the 1 when you multiply 3 times (x - 1).
much more quickly using one
of the three basic strategies for
32x = (33)(x-1) 2x = 3x-3
SAT Math. Which one? STEP 4: Solve for x. 2x =3x-3
+3 +3
2x +3 = 3x
-2x -2x
3=x
USE SUBSTITUTION TO MAKE THE BASES MATCH, THEN SOLVE.

3
6
1. If 36 = 6x, then x = ______
3
12
6. If 16 = 2x, then x = ______

5
15
2. If 8 = 2x, then x = ______
3
12
7. If 81 = 3x, then x = ______

1 x−2 1 n
8. If ( ) = ( ) , then in terms
x−1 n
3. If 125 = 5 , then in terms
3 9
3x - 3
of x, n = ______ of x, n = ______ x-2
2

4. If 64
2x − 4 n
= 4 , then in terms 4
9. If 256 = 2x, then x = ______
6x - 12
of x, n = ______

x+2 n
5. If 49 = 7 , then in terms
-2 x
-4
10. If 16 = 4 , then x = ______
2x + 4
of x, n = ______

SAT v17.10 Unauthorized copying or reuse of any part of this page is illegal. 427
(SAT Teacher Edition)
Roots & Exponents

1 (No Calculator) 3 (No Calculator)

2 (No Calculator) 4 (No Calculator)

81
/ /
. . . .
0 0 0

428
Operations with Polynomials - Part 1

1 (No Calculator)

SAT v17.10 Unauthorized copying or reuse of any part of this page is illegal. 429
(SAT Teacher Edition)
POLYNOMIAL (N):
An expression made up of one or more algebraic
nomial
APPROA
CH: d e gree poly y
terms, usually the sum of several terms that
're given
a 4 th which ma
Here, we p o w e r s of 4) s
contain different powers of variables.
ve give
terms ha the SAT
(x and y ti n g , but when a ted,
tim id a p lic
appear in th a t lo oks com TERM (N):
n
xpressio lify it.
you an e a tr ic k to simp A "building block" of a polynomial. A number,
sua lly
there's u polynomia
l, or the product of a number and one or more
a h ig h e r degree variables with whole number exponents.
ugh this
is the
Even tho form of
t lik e a common ls ) th at you
lo lynomia
it looks a g r e e p o DEGREE (N):
s (2nd de
quadratic e past.
seen in th The degree of a polynomial is the greatest
may have t
a perfec
s v e r y similar to exponent in the polynomial.
w it look
Notice ho
uadratic.
square q
Polynomials Can't Contain:
1) Negative or Fractional Exponents/Roots
terms
and last
The first
2) Variable Exponents
squares : 3) Variables in a denominator
are both 2
x ) rm is a
2
4
= (5
2 5 x middle te This 2nd
degree
And the o
other tw
- and -
2
"m ix" of the
lo w er deg r e e (2nd). x + 4x + 4
2
polynomia
l has
) wit h a 3 Terms.
9y = (3y
4 2

1 (No Calculator)

re
ect squa
w th a t for perf
We kn o
s:
quadratic 2

2 + 2ab + b
2
= (a + b)
a

:
pression
in g w it h our ex
a = 5x
2

tr y th e same th )
2
= a
2
,
x
2
Let's 25x = (5
4

square: b = 3y
2

te r m is a 2
= b ,
2
fi r s t 4
(3 y
2
)
Our =
quare: 9y
la s t te rm is a s
And our
the
we take
s q u a re, when
perfe c t ur middle
p o ly n o mial is a b y 2 , w e'd get o
r ly
So, if ou and multip
x and 3y
2
2
c t o f 5
produ
4
term. 0x y
2 2
0x y + 9
2 2 y
x y ) = 3 25x + 3
4
Nailed It!
2 2
) = 2 (1 5
x )(3y
2 2 2
(5 ) + (3y )
2
2 a b = 2 3 y
2
(5 x
2
)(
(5x ) + 2
2
2

+ b
2

lify our a + 2ab


2

a n s w e can simp 2
That me ing it into (a + b)
e s s io n b y factor
expr we
2
, just like (5x + 3y
)
2
t square
2
f e c s
430 a p e r
adratic. And that'
for a qu s w er!
would do our a n
SOME SPECIAL POLYNOMIALS TO KNOW

uare
rfect sq
a2 + 2ab + b2 = (a + b)2 Called pe
because
they can
trinomials square.
x p r e s sed as a
a2 − 2ab + b2 = (a − b)2 be e

the
is called
This one uares."
a − b = (a + b)(a − b)
2 2
"differen
ce of sq
spot
y easy to
It's prett on the
s h o w s up a lot
and
SAT!

And, as we saw on the previous problem, knowing these can


sometimes be useful for higher degree expressions too.

PRACTICE FACTORING SPECIAL POLYNOMIALS

1. 4a2 + 12a + 9 5. x2 − 16
2
(2a + 3) (x + 4)(x − 4)

2. a2 − 10a + 25 6. 81 − a2
2
(a − 5) (9 + a)(9 − a)

3. y2 + 24y + 144 7. x6 + 6x3 + 9


2 3 2
(y + 12) (x + 3)

4. x2 − 14x + 49 8. 100 − x10


2 5 5
(x − 7) (10 + x )(10 − x )

SAT v17.10 Unauthorized copying or reuse of any part of this page is illegal. 431
(SAT Teacher Edition)
PERFECT SQUARES: THE QUICK(ISH) METHOD

If you have a polynomial in this form: ax2 ± 2 ∙ (bx) + c2


You can check to see if it's a perfect
square using this equation: a∙c=b
And if it is, you can write it in this form: ( ax ± c)2

9x2 − 24x + 16
First,
the test:
a = 9
b = 12 a . c = 3 . 4 = 12 = b
a 2b c
2
c = 4 That works!

So, we can write this expression 2 2


( a x - c) = (3x - 4)
as a perfect square:

OPERATIONS WITH POLYNOMIALS: REVIEWING THE BASICS


When we're dealing with polynomials on the SAT, it's very common that the solution will involve
factoring or recognizing one of the special polynomials above, but you still need to know the basics.
Let's review the rules for working with polynomials.

ADDING POLYNOMIALS 3 2
(x + 2x − 4x + 1) + (x + 3x − 5)
2

To add, just combine like terms. Watch the 3 2 2


x + (2x + x ) + (−4x + 3x) + (1 − 5)
variables and exponents carefully!
3 2
x + 3x − x − 4

SUBTRACTING POLYNOMIALS 3 2 2
Remember, subtracting is just "adding a negative." (x + 2x − 4x + 1) − (x + 3x − 5)
3 2 2
Change the signs for each of the terms of the (x + 2x − 4x + 1) + (− x − 3x + 5)
second polynomial and add them by combining 3 2
like terms. x + x − 7x + 6

MULTIPLYING POLYNOMIALS
To multiply polynomials, you need to distribute. (x + 2)(x − 5)
For binomials (2 terms) this means FOILing. With 2
more terms it gets a little more complicated. Be x + 2x − 5x − 10
sure to fully distribute each term. 2
x − 3x − 10

DIVIDING POLYNOMIALS
x − 4 = (x + 2)(x − 2)
2
Dividing polynomials is trickier. The long way,
which we review in a later section, is called x+2 x+2
"Polynomial Long Division." It's takes a lot of time
and paper. On the SAT, you should first write the x−2
polynomials as a fraction, then try to factor the
numerator and/or denominator to see if anything
cancels. If not, rock the PLD.

432
USE YOUR KNOWLEDGE OF POLYNOMIALS TO SIMPLIFY THESE EXPRESSIONS

1. (3x2 + 14x − 5) + (x2 + 6x + 30) 5. (x − 3)(x + 2)


2 2
4x + 20x + 25 x −x−6

2. (2x3 + 4x2 + 3xy + 5y2) + (x2 + 2xy + 6y2) 6. (4 − x)(x + 2)


3 2 2 2
2x + 5x + 5xy + 11y −x + 2x + 8

3. (y2 − 3y + 6) − (2y2 + 5y − 7) 7. 3x2 − 19x − 40


2 x−8
−y − 8y + 13
3x + 5

4. (3y3 + y2 − 4y) − (y3 + 5y − 10) 8. 9x2 − 4


3 2 3x + 2
2y + y − 9y + 10
3x − 2

SAT v17.10 Unauthorized copying or reuse of any part of this page is illegal. 433
(SAT Teacher Edition)
Operations with Polynomials - Part 1

1 (No Calculator) 3 (No Calculator)

2 (No Calculator) 4 (No Calculator)

2
/ /
. . . .
0 0 0

3
/ /
. . . .
0 0 0

434
Operations with Polynomials - Part 2

1 (No Calculator)

SAT v17.10 Unauthorized copying or reuse of any part of this page is illegal. 435
(SAT Teacher Edition)
POLYNOMIAL LONG DIVISION (N):
CH: s like it
APPROA p r o blem look
glance, th is the SAT, a way of dividing one polynomial by another
At first e f a c to ring. On
olve som in both th
e polynomial of the same or lower degree.
might inv p olynomials in g to
u h a v e r , tr y
when yo th e d e n ominato
r and od instinc
t. Polynomial long division has the same steps as the long
numerato n c el is a go
n d c a division that you learned in elementary school. You just
factor a use them on polynomials instead of numbers.
t road,
u g o down tha r
o numerato
If y With PLD, we can go from this:
u 'll fi n d that the ).
yo 4)(x - 2 Numerator
to (3x +
factors there,
r e 's n o (x - 3) in Denominator
Th e able to
ouldn't be
so we w h ich make
s To this:
n c e l a n y thing, w
ca
end. Remainder
it a dead Quotient +
Denominator
nom ial Technically, when you're dividing, the correct
lled "poly
e r a p p roach ca is io n terms are "dividend" and "divisor." But, you may
anoth g div
Let's try t like lon
is io n ." P LD is jus it h polynomia
ls find it less confusing to think of them as the
long div u m b e r s , only w "numerator" and "denominator."
ular n
with reg
instead .
2 (No Calculator)
tion in
in g o u t the frac
wr it
Start by ion form
:
c h o o l" long divis
"old-s

- 8
3x - 2x
2
ith PLD!
x - 3 div id e th is by this w
We can
r
to r Numerato
Denomina

,
g division
th e s te ps of lon
w s.
Next, follo ding term
w it h the lea
begin n in g der over
e Remain
ACT: 5) Put th and add
3) SUBTR minator
e terms
and the Deno end the
Line u lik
p lt to the
1) DIVIDE: e leading Be care
ful not the resu
all but th subtract. mixed up
!
Ignoring 2 y x = 3x: ur signs quotient.
m s , 3 x divided b to get yo
ter 13
3x 3x
3x + 7 + x - 3
- 8
3x - 2x
2
- 8
3x - 2x x - 3
2

x - 3
3x - 9x
2

e's
And ther
wer!
7x - 8 your an s

IPLY:
2) MULT isor
y the div & REPEA
T:
Multiply b d put the 4) RINSE
n o m in ator) an
(d e Line up
re s u lt u n d erneath. 3x + 7
like term
s: - 8
3x - 2x
2

x - 3
3x - 9x
2
3x

3x - 2x
2 - 8 7x - 8
x - 3
7x - 21
3x - 9x
2

436 r
13
Remainde
WATCH YOUR SIGNS!
With PLD, you'll be doing a lot of subtracting negatives, so it's
really easy to get your signs messed up. You may find that for the
"subtraction" step, it's easier to change the signs and add. Practice,
write out the steps, and be consistent.

YOUR TURN: BRUSH UP YOUR "PLD"

1. x2 + 3x − 5 4. 4x3 + 5x − 6
x−4 x−1
23 3
x+7+ x−4 4x2 + 4x + 9 +
x−1

2. − 2x2 + 4x − 5 5. 5x3 + 3x2 − x + 15


x+3 x2 + 3
35 16x + 6
−2x + 10 − x + 3 5x + 3 − x2 + 3

3. x3 + 4x2 + x + 5 6. 8x2 − 4x − 3
x+2 2x + 1
11 1
x2 + 2x − 3 + x + 2 4x − 4 + 2x + 1

SAT v17.10 Unauthorized copying or reuse of any part of this page is illegal. 437
(SAT Teacher Edition)
POLYNOMIAL REMAINDER THEOREM (N):
The real definition looks way more complex than it actually is, so let's focus
instead on what you can do with it. If you want the (not so) juicy details, you
can Google them.

Basically, PRT says that if you have a polynomial, and divide it by any first
degree polynomial (x plus or minus something) the theorem gives you
the remainder. You don't need to do any long division. Just plug in a few
numbers, and you'll skip straight to the remainder.

Some Polynomial, p(x)


If:
(x − a)

Then: The Remainder = p(a)

e the
if we tak
3x2 − 2x + 8 a = 3, so
, a nd plug 3
in for
r a to r
nume ainder.
x−3 x, we'll ge
t the rem

) + 8
3(3) - 2(3
2

8 = 29 ainde r
27 - 7 + The Rem

Recognize those numbers? What if we'd used the


Remainder Theorem to eliminate answer choices in
our challenge problem? Hate PLD? Me too.

YOUR TURN: LET'S FIND SOME REMAINDERS

1. 2x2 + x − 8 3. −x2 + 5x + 6
x−4 x+3

Remainder = 28 Remainder = −18

2. x2 − x + 3 4. x3 + 5x + 3
x+2 x+3

Remainder = 9 Remainder = −39

438
MORE STUFF TO KNOW ABOUT POLYNOMIALS
Some SAT problems won't even give you numbers to work with. They'll just ask you something about some
polynomial p(x), without even telling you what the polynomial is. For these problems, you have to know some
facts about polynomial functions.

Using remainder theorem, if the remainder = 0, (x − a) is a factor


m
r Theore
Remainde 0
2
(5) + 5 = ainde r
2(5) - 9 The Rem
:
2x2 − 9x + 5 now that
So, we k 2
9x + 5
x− 5 f a c t o r of 2x -
x - 5 is a 2
9x + 5
o f f (x ) = 2x -
ro
5 is a ze

Positive Leading Coefficient vs. Negative Leading Coefficient


If the coefficient of the leading term is positive, the graph goes up at the end. Negative leading
coefficient, down at the end.

Positive Leading Coefficient: Negative Leading Coefficient:


3
Ex: f(x) = 3x + . . . Ex:
3
f(x) = -3x + . . .
Odd Degree vs. Even Degree
If the degree of the polynomial is even, then the ends of the graph point the same way (both up or
both down). If the degree is odd, they point opposite ways (one up, one down).

Even degree (2) = same direction Odd degree (3) = one up, one down
3
Ex: f(x) = x + . . .
2 Ex: f(x) = x + . . .

Maximum Number of Zeros = The Degree

2
The graph of y = x + . . . has a degree Or, it could have
of 2 so it has (at most) 2 zeros. fewer than two.

Maximum Number of Extreme Values = Degree − 1


The maximum number of times a graph can "change direction" is one less than the degree.

5
Ex: y = x + ...

Higher degree functions can change


2nd degree (parabola) direction more times.
changes direction once.
*Note this can also mean more zeros.
SAT v17.10 Unauthorized copying or reuse of any part of this page is illegal. 439
(SAT Teacher Edition)
Operations with Polynomials - Part 2

1 (No Calculator) 3 (No Calculator)

2 (No Calculator) 4 (No Calculator)

440
Solving Quadratics

1 (No Calculator)

SAT v17.10 Unauthorized copying or reuse of any part of this page is illegal. 441
(SAT Teacher Edition)
CH: uadratic
APPROA a y s to solve a q QUADRATIC EQUATION (N):
e a r e s everal w a ti o n , s o me
Ther g on the
situ an equation of degree 2, which we learned earlier
q u a ti o n . Dependin a n o th ers. means that the highest exponent is 2.
e tter th
o d s w ill work be
meth od
eck meth
g u e s s and ch The standard form of a quadratic is:
c to r in g using th e
s b e s t when the
Fa k
ut it wor st
easiest, b The highe
is usually
solution(s
) are inte
g e r s .
r answer c
hoice s y = ax + bx + c 2
expo n e n t is
a ll o u y s 2 .
tice that u might a
lso alwa
Here, no lo oking. Yo
of c r a z y f a
are kind e form o
th e y 'r e all in th ar? a, b, and c are constants (numbers) called
notice th
a t ok fa ili
m
o m e r a dical. Lo
+/- s coefficients. We can use these coefficients
number ?
f o r m u la anyone
c to help us solve the equation.
Quadrati
tor in g
here fac
" q u a d ratics, w e
For thes
e "ug ly , we h v a
a r d o r imposible
super h
is either
ons: any
two opti ork for
e m ethods w
squar e T h e s fa tor,
c
lete the you can't
1) Comp ula qua d r a ti c . If
tic form our go-to
.
) U s e th e quadra they're y
2

1 (No Calculator)
BY
O SOLVE E:
STEPS T T H E SQUAR
TING
COMPLE

nt ("a")
coefficie
G e t r id of the .
1)
e x term
2
ould
before th rs, we c
e w e re intege
If th e s ring. But,
by facto
+ 6 = 0 s o lv e
2x + 16x aight to
2
probab ly
2 n 't , we go str
2 2 since the
y a r e
mpleting
the
2 r m ula or co
c f o
3 = 0 quadrati
x + 8x +
2
ethod.
square m

the
term to
v e th e constant f th e way.
2) M o it out o
to g e t
right side
3 = 0
x + 8x +
2
3
- 3 -
= -3
x + 8x
2

of both
constant e s q u are root nt.
g for a 4) Take
th e expone
e'r e lo o k in
ft to ma
ke e t rid of th
3) Now w to the le sides to g rget the
c a n a d d to Don't fo
that we add2 it root!
re, then
2

a p e r f ect squa (b /2) . (x +4) = 13 negative


it ays
th s id e s . Hint: Alw
= - 13
2
bo +
= -3 (x + 4)
x + 8x
2
+16 - 13
+
+16 x + 4 =
16 = 13
x + 8x +
2

x:
tor olve for
5) And, s
2
Then fac = 13
(x + 4)
+ 13
x = -4 -
442
THE QUADRATIC FORMULA
If you haven't memorized it yet, you probably want to do that now. The equation is derived from the
"complete the square" method, but if you use it instead, you get to plug in a few numbers and skip all the
other algebra. Like the "complete the square" method, it works on any quadratic.

When you put a quadratic equation in standard form...

ax2 + bx + c = 0
... and a, b, and c are constants, then you can find the two solutions using this formula:

-b ± b2
− 4ac
x=
2a

3x2 + 12x + 6 = 0 -b +
2
- b - 4ac
x =
3 3 2a

-4 +
2
a = 1 - 4 - 4(1)(2)
x =
(1)x + 4x + 2 = 0
2
b = 4 2(1)
c = 2
-4 +
- 8 -4 + 2 2
x = = -
2 2 2

= -2 +
- 2

YOUR TURN: SOLVE USING YOUR METHOD OF CHOICE

1. x2 + 6x + 7 = 0 4. x2 − 8 = 0
x = −3 ± 2 x = ±2 2

2. 3x2 − 5x + 1 = 0 5. 2x2 + 4x − 8 = 0
5 ± 13 x = -1 ± 5
x=
6

2
3. −2x + x + 8 = 0 6. x2 − 6x − 10 = 0
1 ± 65 x = 3 ± 19
x=
4

SAT v17.10 Unauthorized copying or reuse of any part of this page is illegal. 443
(SAT Teacher Edition)
SOLVE BY FACTORING
Not every quadratic can be easily factored, but for those that can be, factoring is the fastest and least error prone
way to solve. You've probably done quite a bit of factoring by now, but here's a quick brush up on how it works:

Steps to Basic Factoring


Basic factoring is easiest when there is no leading coefficient (coefficient of 1).
Here are the steps:

x2 + 2x = 8 1) Move all terms to the left side of the equation.


-8 -8 Combine like terms if necessary. You want only a
zero left on the right side.
x + 2x - 8
2
= 0

r that:
2 numbe this 2) Next, you need to find two numbers that, when
ad up to
d is
ly to get th multiplied give you the third term, and, when added,
and multip
give you the second coefficient.
4 x -2 = -8
4 + (-2) = 2

3) Now you can rewrite the equation in factored form:


(x + 4)(x - 2) = 0 (x - a)(x - b) = 0

4) And here's the trick. If the product of two numbers


equals zero. One or both must be zero. Therefore,
x - 2 = 0 we can conclude that (x - 4) = 0 or (x + 2) = 0, or
x + 4 = 0
both. Write out those equations.

x = -4 x = 2 5) Solve for x.

Factor by Grouping
When the leading coefficient is greater than one, factoring can be more difficult. You can
attack these problems using a method called "grouping."

3x2 − 13x − 10 = 0
1) Find a and b, such that ab = leading coefficient
(-15) x 2 = -30
times the last coefficient (3 x (-10)) = -30, and
(-15) + 2 = -13 add up to the second coefficient (-13).

2
3x - 15x + 2x - 10 = 0 2) Split the second term into two terms using our
numbers as the new coefficients.

(3x - 15x) + (2x - 10) = 0


2
3) Group the first terms and the last terms.

3x(x - 5) + 2(x - 5) = 0 4) Factor both binomials.

(x - 5)(3x + 2) = 0 5) Factor out the (x - 5).

x - 5 = 0 3x + 2 = 0 6) Just like we did above, use the zero product


property and solve for x.
x = 5 x = -2
3
444
YOUR TURN: SOLVE USING YOUR METHOD OF CHOICE
(HINT: CHOOSE FACTORING)

1. x2 + 2x − 8 = 0 4. −x2 − 6x + 7 = 0
−4, 2 1, −7

2. x2 − x = 12 5. 9x2 + 12x + 4 = 0
4, −3
− 2
3

3. 4x2 + 5x − 6 = 0 6. 5x2 − 20x + 15 = 0


3 , −2 3, 1
4

QUADRATIC AND LINEAR SYSTEMS


There will likely be at least one question on your test involving the solution(s) of a system of a quadratic and a line.

y y y
Number of Solutions
A quadratic function is a parabola, and a linear function
is a line. When you plot the functions on an x-y plane,
you can see that there can be 2, 1, or 0 solutions. x x x

Zero One Two


Finding the Solutions
When you're given a system of a quadratic equation and a linear equation, the trick is to isolate y, then use
substitution to eliminate y and solve for x. This will give you the x-value(s) of the solution(s). If you need the
corresponding y-value(s), plug the x-value(s) back into either equation.
1) Use substitution to set the
3x + 2 = x - 1
2
two equal.
-3x -2 -2 -3x
y = 3x + 2 0 = x -3
2
-3x 2) Now, solve for x. Since it
y=x −12 2
0 = x - 3x - 3
doesn't look like factoring is
going to work, we have to go
to the quadratic formula.
3 + 21
x = -
2 2 Yuck! Looks like we
chose a ugly example.

SAT v17.10 Unauthorized copying or reuse of any part of this page is illegal. 445
(SAT Teacher Edition)
Solving Quadratics

1 (No Calculator) 3 (No Calculator)

3
/ /
. . . .
0 0 0

2 (No Calculator) 4

446
Graphing Quadratics (Parabolas)

1 (No Calculator)

SAT v17.10 Unauthorized copying or reuse of any part of this page is illegal. 447
(SAT Teacher Edition)
PARABOLA (N):
The graph of a second degree (quadratic) function.
Quadratics are expressed in various forms, each of
which can reveal various properties of the parabola.

"Standard" Form:
y = ax2 + bx + c
Each term consists of a coefficient (number)
CH: and a power of x. Any of the coefficients
APPROA a parabo
la,
us this is (except "a") can be zero.
They tell dy kno w that.
w e s h o uld alrea
but rm? "Factored" Form:
e the fo
Recogniz

, th e y ask for
a differ
ent
t
y = (ax + b)(cx + d)
Th e n ation tha
f th e same equ Good for finding the x-intercept(s).
form o pts as
u s th e x-interce
e
would giv ficients.
n ts or coef "Vertex" Form:
consta

y = a(x − h)2 + k
Good for finding the vertex of a parabola

? 1 (No Calculator)
ficie nts? Huh
and Coef
As Constants

number.
is ju s t a plain old
nt tion
A consta the equa
ig in a l f orm of
In the o r onstant.
f o r m ), 15 is a c
d"
("standar
r at the
t is th e numbe
A coeffi
cie n iplies"
h te r m that "mult
of eac quation,
beginning original e
th e
the varia
ble . In cients.
r e coeffi of the
ple, 1 a n d -8 a
th e s e is a form
for exam Each of are usef
ul.
a l e q u a ti on. Some
origin ch.
ot so mu
Others, n
st found
they've ju
So, really omplicate
d
u n n e c e ssarily c
an
sk us:
way to a
us
tion gives
r m o f the equa e rs?"
o b
"What f in plain o
ld num re d" form:
c e p t( s ) In "facto )
b)(cx + d
r
the x-inte
y = (ax +
and -d.
n we're rcep ts are -b
n o w that whe The x-inte
And w e k lso called
r x -i n te rcepts (a
o
looking f ctored"
form rcepts
o r z e ros), "fa u a ti o n s , the x-inte
roots r eq r
So, in ou in answe
y to go. nstants)
is the wa are 3 an
d 5 (c o
.
choice D
)(x - 5)
y = (x - 3
448
ANATOMY OF A PARABOLA
A parabola has several values that we may need to find and/or understand. SAT problems test your knowledge of
quadratic functions and how they relate to their graphs.

Get comfortable moving back and forth between quadratic equations and their graphs, and be able to manipulate
the equations between common forms to identify important characteristics.

p
ns u
Ope
pt(s)
x-interce
(roo )
t s

ex
erce pt The Vert
The y-int

Number of x-intercepts Upward or Downward


A parabola can have 2, 1, or 0 x-intercepts.

The parabola above has 2 x-intercepts. Also, parabolas can open either upward or
Others have one (both roots are the same) downward.
or zero (no real roots).
y y
y y

x
x x

rce pt rce pts ward wnwa rd


One x-inte No x-inte Opens Up Opens Do

SAT v17.10 Unauthorized copying or reuse of any part of this page is illegal. 449
(SAT Teacher Edition)
STANDARD FORM
Quadratic functions in standard form are common on the SAT. It's easy to recognize, but can be a little more
cumbersome to work with than some of the other forms. You can use algebra to convert the equation into a more useful
form, or you can memorize some formulas to calculate properties of the graph directly.

y = ax2 + bx + c

y
x-intercept(s): upward vs. downward:
x = -b ± b − 4ac
2
Positive a, parabola goes upward.
2a Negative a, parabola goes downward. x

This is the quadratic formula. If you How to remember:


don't have it memorized, do it now. Think of the simplest parabola: y = x
2

Here, a = 1 (positive) and it opens upward.

y-intercept: y = c vertex:
How to remember:
How to remember:
At the y-intercept, x=0. Plug in zero -b The x-value is the quadratic
x= equation without the +/- part.
for x, and all you'll be left with is c. 2a Then, plug back in to get y.

How many x-intercepts are there?


2
The part of the quadratic formula under the radical (b - 4ac) is called the determinant. It "determines" how many
real roots (x-intercepts) that the function has.

b2 − 4ac > 0 two x-intercepts


x = -b ± b − 4ac
2

2a b2 − 4ac = 0 one x-intercept

The Dete
rmin ant. b2 − 4ac < 0 zero x-intercepts

uld our
What wo look
formula
quadratic2 radical.
- 4ac = 0
?
"li v e s " under a
like if b rminan t
we'd hav
e
The dete is negative,
m in a n t
ter at do
If the de dical. Wh
u n d e r the ra ts?
a negati v e uare roo
b o u t n e gative sq
a
we know

450
Getting the vertex:
y = −2x + 5x + 32
-b 5
x = = -5 =
a = -2, b = 5, c = 3 2a 2(-2) 4

5 y
How many x-intercepts: So, the vertex is at:
4 ,
2 5
b - 4ac = (5) - 4(-2)(3)
2
Plugging back into the equation,
ant is 4
= 25 - (-24) t e rmin
D e ave we get:
s o we h
pos ,
= 49 ts.
t e rcep 2
2 x - in y = -2x + 5x + 3
5 2
5
Use the quadratic formula to y = -2 + 5 + 3
4 4
find them:
49
-b +
-
2
b - 4ac y = 5 49
x = 8 Vertex:
2a 4 8
,

-5 +
2
- 5 - 4(-2)(3)
x =
2(-2)
's
Here aph: 5 49
r
x =
-5 +
- 7 = -1
3 or 2 the g 4 , 8

-4

"a" is negative, so it opens downward. -1


,0 (3, 0
)
2

YOUR TURN
Find the x-intercept(s) & vertex. Then, sketch the parabola.

1. y = x2 + 2x − 15
y = (x + 5)(x − 3) y

(−5, 0) (3, 0)
x

−5, 3
x-intercept(s): ______

(−1, −16)
vertex: ______ (−1, −16)

SAT v17.10 Unauthorized copying or reuse of any part of this page is illegal. 451
(SAT Teacher Edition)
FACTORED FORM
Factored form makes it much easier to find the x-intercepts.

y = (ax + b)(cx + d)

x-intercept(s): y = (−x + 3)(2x − 1)


ple
these sim
u c a n m emorize
Yo ant, but
if you w
-b formulas d it easie
r 0 = (-x + 3)(2x - 1)
x= a y o u 'll p r o bably fin
e "zero p
roduct
u s e th
just to te two
r o p e r ty " to crea 0 = 2x - 1
p for x. 0 = -x + 3
en solve
-d equa ti o n s , th
x= c x = 3 1 = 2x
1
x =
2

Getting to Factored Form


To get a quadratic function into factored form, start with standard form, then factor it. If the leading coefficient is
greater than one, try to factor by grouping. If you can't factor, you'll need to use either the quadratic equation or
the next form, called "vertex" form.

Factor by Grouping
We covered this earlier, but here's a review. Grouping is a technique for factoring that is especially useful when
the leading coefficient is not equal to one.

y = 3x2 − 13x − 10
1) Find a and b, such that ab = leading coefficient
(-15) x 2 = -30
times the last coefficient (3 x (-10)) = -30, and
(-15) + 2 = -13 add up to the second coefficient (-13).

2
y = 3x - 15x + 2x - 10 2) Split the second term into two terms using our
numbers as the new coefficients.

y = (3x - 15x) + (2x - 10)


2
3) Group the first terms and the last terms.

y = 3x(x - 5) + 2(x - 5) 4) Factor both binomials.

y = (x - 5)(3x + 2) 5) Factor out the (x - 5).

452
VERTEX FORM
This form makes it super easy to find the vertex, and it's also probably the easiest to graph from.

y = a(x − h)2 + k

ad,
roblem h
Vertex: (h, k) If th e c h allenge p
sked us:
instead, a
s
Upward vs. Downward g display
h o f th e followin
"whic rtex
f the ve
Positive a: opens upward. c o o r dinates o "
th e ficients?
Negative a: opens downward. n s ta n ts or coef
as c o
n
have bee
definitely
−k Then, w e w o u ld
vertex f orm.
Roots: x=± a +h looking f
o r a n e q uation in

Getting to vertex form


To get from standard form to vertex form, you use the "complete the square" method that we covered earlier.

y = 2x2 − 4x + 5
2 2 2 2 1) Get rid of the coefficient before
2
the x term by dividing by "a."
y 5
= x - 2x +
2

2 2
y 5
= x - 2x
2
- 2) Move the constant term to the "y-side,"
2 2 away from the x2 and x terms.

y 5
+ 1 = x - 2x + 1
2
- 3) Complete the square on the right side of the
2 2 equation by adding a number to both sides.
2
Hint: The number will always be (b/2) .

y 5 2
- + 1 = (x - 1) 4) Now, factor the perfect square you made.
2 2
y 3 2
5) Combine like terms.
- = (x - 1)
2 2
y 2
3
= (x - 1) + 6) Move the constant term back to the right.
2 2
2
y = 2(x - 1) + 3 7) Isolate y.

SAT v17.10 Unauthorized copying or reuse of any part of this page is illegal. 453
(SAT Teacher Edition)
YOUR TURN: "FACTORED FORM"
Convert the following equations to "Factored Form," and find the x-intercepts.

1. y = x2 + 4x + 3 3. y = x2 + 5x
y = (x + 3)(x + 1) y = x(x + 5)
− 3, −1 0, −5

2. y = 6x2 − 7x − 3 4. y = −2x2 + 9x − 10
y = (3x + 1)(2x − 3) y = (− x + 2)(2x − 5)
− 1, 3 2, 5
3 2 2

YOUR TURN: "VERTEX FORM"


Convert the following equations to "Vertex Form," and find the vertex.

1. y = x2 − 4x + 7 3. y = x2 − 10x + 16
y = (x − 2)2 + 3 y = (x − 5)2 − 9
(2, 3) (5, −9)

1 2 7
2. y = 2x2 + 4x − 3 4. y= x −x−
2 2
2
y = 2(x + 1) − 5 y= 1 2
(x − 1) − 4
(−1, −5) 2
(1, −4)

454
Graphing Quadratics (Parabolas)

1 (No Calculator) 3

2 (No Calculator) 4 (No Calculator)

SAT v17.10 Unauthorized copying or reuse of any part of this page is illegal. 455
(SAT Teacher Edition)
456
Function Notation

1 (No Calculator)

SAT v17.10 Unauthorized copying or reuse of any part of this page is illegal. 457
(SAT Teacher Edition)
FUNCTION (N): an equation with y isolated
on the left side, but with y replaced with
f(x). Because functions are essentially just
k more
o b le m s often loo
p r e. equations, they can easily be graphed.
Function ctually ar
a te d th an they a
complic

Rememb
er, a f u n c ti o n is just
a recipe
h the "inp
.
ut" in
f(x) = 4x + 2
u w h a t to do wit
It tells yo utput."
get the "o
order to

ff(x)
f(
(x) = 44x + 2 means the same thing as
(x
- 10
(x) = -3x
y = 4x + 2. That means you can even graph
n c ti o n : f
us a fu it the same way as you ordinarily would. This
They give helps a ton when you need to graph a function.
to
ipe says
So remember that function notation is just like
Our rec any other equation, but y has been changed to
e input
multiply th n f(x), g(x), or something in that format.
ve 3, the
by negati
10.
subtract

1 (No Calculator)

g"), just
("somethin
To get f g" into
"somethin
plug that ywhere
tion ever
the func
n "x".
there's a

all we
e th in g " is −5x, so
m
Here, "so into our
d o is plug -5x x:
have to have an
e v e r y w here we
function
- 10
f(x) = -3x
10
-3(-5x) -
f(-5x) =
15x - 10
f(-5x) =
e's
And, ther
s w er.
your an

458
FUNCTION, FUNCTION, WHAT'S YOUR... FUNCTION?
A function is just a series of steps for you to perform. That's all. It may help you to think of a function as a
button on your calculator. If you type a number into your calculator and then hit the "squared" button, it
performs a function. Your calculator multiplies whatever number you gave it by that same number. You could
think of the "squared" button as performing the function f(x) = x2.

What if your calculator had a "g(x)" button? That button would also be a function, but what would it do? Well,
that all depends on what g(x) is defined as. If g(x) is defined as g(x) = x2 − 4, then that's what the g(x) button
would do. It would take whatever number you punch into the calculator, square it, and then subtract 4 from
the product. Obviously, the end result will depend on what number you punch into the calculator before
hitting the g(x) button. That's what makes it a function!

That's really all you're doing on an SAT function problem. The problem will usually give you the definition of
the function, meaning a list of steps to perform. Then the problem will ask something like "What is the value
of f(x) when x = 5?" All you need to do is plug in 5 for x and solve for f(x) as if it were any other variable. f(x) is
basically just a stand-in for y, after all.

If f(x) = x3 + 3x, then f(4) =

This statement basically tells you to take the number in the parentheses after the f and plug it in wherever
you see an x. Put it into the equation wherever you see an x and solve for f(4) as it were the variable y.

f(4) = (4)3 + 3(4)

f(4) = 64 + 12

f(4) = 76

WHATEVER YOU FIND IN THE PARENTHESES IS WHAT YOU PLUG IN FOR X

Plug in whatever is in the parentheses for x, no matter what it is!

If you're asked to find f(5)? Use (5) in place of x. Simple, right?

If you're asked to find f(x + 1)? Plug in (x + 1). Still pretty simple.

If you're asked to find f (hamburger)? Plug in the word (hamburger) in place of x.

Remember to keep parentheses around whatever you plug in for x because it represents one number.

SAT v17.10 Unauthorized copying or reuse of any part of this page is illegal. 459
(SAT Teacher Edition)
A BASIC FUNCTION IS LIKE A MICROWAVE

f(x)
x machine
f(x)
You put in a frozen te ... and out comesd aburrito.
burrito.... f(x) = heat x + rota heated, rotate

A FUNCTION WITHIN A FUNCTION IS LIKE MICROWAVING SOMETHING TWICE

f(x)
x machine
f(x)
You put in a frozen te ... and out comes ated
burrito.... f(x) = heat x + rota heated and rotastill kind
burrito that is middle.
of frozen in the
THEN, YOU RUN IT THROUGH AGAIN!

f(x)
f(x) machine
f(f(x))
You put in a heate
d and burrito that's
at's te ... and out comes arotated and
rotated burritoenthin the f(x) = heat x + rota been heated and ted again.
still kind of froz heated and rota
middle...

460
A FUNCTION WITHIN A DIFFERENT FUNCTION IS A LOT LIKE DOING LAUNDRY

f(x) washing machine

x f(x)
hes in bunch of clea n
You put dirty clot ine... ... and out comes a
the washing mach h(x) = wash x + spin wet clothes.

g(x) dryer

f(x) g(f(x))
You put clean wetdryer... bunch of
ble ... and out comesclaean clothes.
clothes into the g(x) = heat x + tum dried, tumbled,

FUN WITH FUNCTIONS


1. If f(x) = 3x + 6 and g(x) = −2x − 2 for all values 3. If f(x) = 2x + 10 and g(x) = x2 − 4 for all values of
of x, what is the value of g(f(2))? x, what is the value of g(f(x)) − f(g(x))?

- 26 2x2 + 40x + 94

2. If f(x) = x2 − 4x and g(x) = 2x + x2 for all values of 4. If f(x) = 2x − x2 and g(x) = x2 − 2x for all values
x, what is the f(g(3))? of x, what is the value of f(g(f(3)))?

165 - 195

SAT v17.10 Unauthorized copying or reuse of any part of this page is illegal. 461
(SAT Teacher Edition)
GRAPHING A FUNCTION
It's helpful to think of the graph of a function as the value of the "output" (the y-value) for every
"input" (the x-value). To graph a function, just replace f(x) with y and graph the function like you
would any other equation.
y
f(x) = 2x + 1

's
Here aph: (x)
r y = f
the g

READING THE GRAPH OF A FUNCTION


Sometimes they'll give you the graph of a function, but not the actual function. Here, you just
have to work backwards. For any "x" (the x-coordinate), f(x) = the y-coordinate.

y
y = f(x) f(−2) = 3
(−2,3)
the
idea what
We have no
but we
or f(x) is,
x equation f
raph
rom the g
can read f
) = 3.
-2, then f(x
that if x =

YOUR TURN
Here's the graph of a function g(x). Use the graph to answer the following questions.

1. What is g(4)?
y

(4,7) 2. What's a value of x for


y = g(x) which g(x) = −3?
(−4,0)
x
(0,−3) 3. For how many values of x,
where −4 < x < 4, does g(x) = −3?

462
Function Notation

1 (No Calculator) 3

2 4

SAT v17.10 Unauthorized copying or reuse of any part of this page is illegal. 463
(SAT Teacher Edition)
464
Real-Life Nonlinear Models

SAT v17.10 Unauthorized copying or reuse of any part of this page is illegal. 465
(SAT Teacher Edition)
EXPONENTIAL GROWTH/DECAY:
Use this formula to calculate the value of something
that increases (grows) or decreases (decays) by the
same percentage for each period of time.

t
Growth: A = P(1 + r)
t
Decay: A = P(1 − r)
A = The amount at the end

P = The amount at the beginning

mon r = The percent change for each period


uper com
o f p r o blem is s (amount of time) expressed as a decimal
This type n focus
on
T ! T hey ofte a
on the S A parts of
ta n d in g of the t = The number of periods (time) during
your und
e r s to p g
lu
e th a n y our ability which the amount increases or
mor
formula swer.
s a n d g et the an decreases. Be sure your units match!
r
in numbe

percent
s b y a constant
e
Decreas this case
), so
io d (year in /
g with per p e r al growth
r e , w e 'r e workin u s e th e exponenti
He so we we can
al decay, rmula.
exponenti ion of decay fo
- r) vers
use the (1
ula: instead, it
the form What if, g
d by 8 m
decrease
?
per year
t
r)
A = P(1 -
r
750 mg sure you
= In it ia l amount = Be triple h.
P time matc
al) = .08 units of
(as decim y give us
r = rate Here, the s,
in years g in year
t = time everythin
good.
t so we're
(1 - .08)
A = 750
t
2)
= 750(.9
our
There's y
answer!

466
Real-Life Nonlinear Models

1 3

2 4

SAT v17.10 Unauthorized copying or reuse of any part of this page is illegal. 467
(SAT Teacher Edition)
Math Unit #4
Homework Drill #1

1 (No Calculator) 3 (No Calculator)

11
/ /
. . . .
0 0 0

2 (No Calculator) 4

468
Math Unit #4
Homework Drill #2

1 (No Calculator) 3

2 (No Calculator) 4

SAT v17.10 Unauthorized copying or reuse of any part of this page is illegal. 469
(SAT Teacher Edition)
Math Unit #4
Homework Drill #3

1 (No Calculator) 3 (No Calculator)

3
/ /
. . . .
0 0 0

2 (No Calculator) 4 (No Calculator)

470
5 Angles 473

Triangles 479

Circles 485
Additional Topics

Area & Volume 491


Math Unit #5

Complex Numbers 495

Basic Trigonometry 499

Homework Drills:

1 2 3
503 504 505

SAT v17.10 Unauthorized copying or reuse of any part of this page is illegal. 471
(SAT Teacher Edition)
GENERAL APPROACH TO GEOMETRY PROBLEMS

1. Write the information in the problem directly onto the figure.


If you know an angle or side length, write it down on the figure.

2. Know your geometry rules and formulas, and how to apply them both to
solving the problem.

3. Let your pencil do the work in the event no figure is given to you.
Get in the habit of writing or drawing figures or axes on a coordinate plane if neither is
drawn for you in the problem.

4. According to the instructions for the SAT, figures provided in geometry


problems are "not necessarily drawn to scale." This means you should not
attempt to estimate an answer based on a figure.

472
Angles

1 (No Calculator)

SAT v17.10 Unauthorized copying or reuse of any part of this page is illegal. 473
(SAT Teacher Edition)
OUR APPROACH: ti on co rr ec tl y , y ou need a basic VERTICAL ANGLES (N):
this ques ry:
In order to answeralgebra and a few rules of geomet the angles diagonally across from one
understanding of s. another when two lines cross
su re of a st ra ig ht line is 180 degreeve equal
1. The angle meashare a vertex (vertical angles) ha
2. Angles that
measures. ta
e' re go in g to a ss ume that you'vehgotwo b a=a
e book, w miliar w it
At this point in thing of algebra Also, if you're unfation on the next page. a ba b=b
basic understandabove, you can find more informa
geometric rules ith our solution.
Let's proceed w
raight line)
3x + 2x = 180 (a st
5x = 180
x = 36
wer.
This is NOT our anslooking for the oking for
Remember, we're o make sure Remember, we're lo using our
value of y, not x. T the wrong the value of y. So,ical angles,
you don't solve for mend that you knowledge of vert = y. Then
variable, we recom tion in the text we can see that 3x set 3x = y,
underline the ques all we need to do is the value of
of the problem. then plug in 36 for for y.
x, and finally, solve
3x = y
3(36) = y
108 = y

Remember that you can pick your own numbers on geometry problems with
variables in them. Just be careful that the numbers you choose don't violate the
rules of geometry.

In the figure below, if x = 20, what is the value of a + b?

a° b°

A) 200
B) 210
C) 220
D) 240

474
COVERING ALL YOUR ANGLES

The sum of the angles that form a straight line is always 180.

a b c a + b + c = 180

The sum of the angles that form a complete circle is 360.

a b c f = 360
f e d a+b+c+d+e+

The sum of the angles of any triangle is 180.


b
e a + b + c = 180
d + e + f = 180
a c d f
Vertical angles are always congruent. These are the angles that are diagonal from each
other when two lines cross.
b a=a
a b a b=b

When parallel lines are crossed by another line, all the smaller angles are congruent and all
the larger angles are congruent.
inters ection
a b Notice that eachrtical angles.
b a has 2 pairs of ve
a b
b a
a b
b a ach of the
Also notice that eat form a
pairs of angles t0h.
line add up to 18

SAT v17.10 Unauthorized copying or reuse of any part of this page is illegal. 475
(SAT Teacher Edition)
If several angles form a larger angle, they must add up to the larger angle.

e a+b+c+d = e
a bcd

Bisectors cut an angle in half. The halves are congruent and add up to the larger angle.

2a Bisector
aa

b p
a a m n

b = 2a p=m+n

The sum of the angles of a polygon with n sides is: (n - 2) x 180


0
(6 - 2) x 180 = 72
(3 - 2) x 180 = 180

4 sides 5 sides 6 sides


3 sides

0
(4 - 2) x 180 = 36
0 (5 - 2) x 180 = 54

476
TRY THESE

1. 4.

72 72

2. 5.

144
95

3. 6.

230 58

SAT v17.10 Unauthorized copying or reuse of any part of this page is illegal. 477
(SAT Teacher Edition)
Angles

1 (No Calculator) 3 (No Calculator)

2 (No Calculator) 4 (No Calculator)

478
Triangles

1 (No Calculator)

SAT v17.10 Unauthorized copying or reuse of any part of this page is illegal. 479
(SAT Teacher Edition)
OUR APPROACH: ti on co rr ec tl y , y ou need a basic
this ques triangles:
In order to answera few rules about geometry and
understanding of tr ia ngle is always 180l .
a ng le s of a
e interior ve equa
1. The sum of thshare a vertex (vertical angles) ha
2. Angles that
measures.
30 + 40 + x = 180
x = 110
z = 180 - 110 - 45
z = 25

1 (No Calculator)

z = 180 - 110 - 45
110 z = 25

110

480
Triangles
Basic Triangle Stuff
The angles of any triangle add up to 180 degrees.
1
Area of a Triangle = 2 Base x Height

Advanced Triangle Stuff


If two angles are congruent, then the sides opposite those angles are congruent. Same
applies for when all sides are congruent, then all angles are congruent. Larger angles
are across from longer legs. Smaller angles are across from shorter legs. The third side
rule of a triangle states that the third side of a triangle must be greater than the
difference of the other two sides but less than their sum.

Just for Right Triangles


For all right triangles, you can use the Pythagorean Theorem to find the length of a
missing side:
rem: s from
c is always acre osang
Pythago2rean2 Th2eo c le. It's
the 90 degrepotenuse, and
a +b =c a
called the hy e longest leg.
it's always th
b
Pythagorean Triples:
"Pythagorean triples" are integer solutions to the Pythagorean Theorem, a2 + b2 = c2
3:4:5 5 : 12 : 13 7 : 24 : 25
(6 : x8 :210) (10 :x242): 26 14 :(x482:)50 And b e ca u se y ou ca n multiply all thtaent
ngle by a cons
dimensions of artrtriaiangle, these are all
to get a similaTriples too!
Pythagorean
SAT v17.10 Unauthorized copying or reuse of any part of this page is illegal. 481
(SAT Teacher Edition)
Special Right Triangles
Special right triangles are right triangles that follow certain patterns.
Watch for these special right triangles:

" "
The "45-45-90 : The "30-60-90:
Right Triangle Right Triangle

45 L2 60 2L
L L
45 30
L L3

Angles of 30, 45, or 60 degrees are hints that you have a special right triangle.
Strategy for Triangle Problems
Every time you see a right triangle, check to see if it's a special right triangle. Special
right triangles are common on the SAT. If you spot them, you'll save a lot of time.
It's often helpful to break a triangle into two right triangles divided by an imaginary line.

45 60
45 30
45 30
Some word problems require you to draw a right triangle to figure out a length or
distance. These are almost always special right triangles.

Similar Triangles
We call two triangles "similar" if the ratio of the lengths of their sides is the same.

4 5 12 15
6
18
These sides have a ratio of 4:5:6 These sides have a ratio of 12:15:18
482
MAKE SURE YOU KNOW THE BASICS ABOUT TRIANGLES

1. 4.

59
3

2.

5.

11

6.
3.

7 6

SAT v17.10 Unauthorized copying or reuse of any part of this page is illegal. 483
(SAT Teacher Edition)
Triangles

1 (No Calculator) 3 (No Calculator)

10 . 5
/ /
. . . .
0 0 0

2 (No Calculator) 4 (No Calculator)

484
Circles

1 (No Calculator)

SAT v17.10 Unauthorized copying or reuse of any part of this page is illegal. 485
(SAT Teacher Edition)
OUR APPROACH: lls u s th a t ∆
we
ABC is equilaterabel, cause
CIRCUMFERENCE (N): the perimeter
of a circle
m te And
Because the probofle its sides are of equal length. du ce that:
know that ea ch l le ng th , w e ca n de
are of eq u a ARC LENGTH (N): the distance it
we know its sides takes to travel between two points along
AB = BC = CA the circumference of a circle
of
B C + C A = th e en tire circumference
AB +
Keep in mind thawt e're getting somewhere!
the circle. Now
s, A O = 6, to fi nd the circumference.
ven radiu e by 23 because
We can use the gi ti re ci rc u m fe re nc
iply the en is 23 of the circle
's
Then we can mult is e fr om B to A
g clockw
the distance movin
circumference..
1 (No Calculator)

C = 2 (r)
C = 2 (6)
+ CA
C = 12 = AB + BC
CA = 8
2
()
C = 12 3 = BC +

USE THE RADIUS TO GET FROM ONE FORMULA TO ANOTHER

r = radius = r
circle
ra di u s is th e ke y to solving everthy en plug it in
d = diameter = 2r The the radius, and for whatever
problem. First, find formula to solve
to the appropriatiteon the question asks you to find.
c = circumference = 2πr piece of informa
It's that simple!
a = area = πr2
is s a ci rc le pr ob le m on the SAT!
You should never m

486
PARTS OF A CIRCLE
the
The diameter (d)wisay across
distance all the lways passes
he distance from d the circle. It a er of the
The radius (r) ishte circle, to any through the centh is twice
the center of t umference of the r circle. Its lengt e radius.
point on the circ e radius is the the length of th
circle. Finding thverything else!
key to finding e

e
The area (A) is trch le.
space inside a ci (c) is the
The circumferencweay around
distance all the circle.
the outside of a
ce
ice of An arc is like a pireence.
A sector is like asusl ally of the circumfe
the circle. We uarea. We usually care .
care about its about its length

Inscribed
Angle
Central
Angle
1 radian
25°
50° arc 50° arc
50° 1 radius 50°

A RADIAN is simply the


radius of the circle used as
a measuring stick for an
arc on the circle and for its
corresponding central angle.

The Circle Formulas


Area of a Circle = r2 Sector Area = Area of Circle x Sector Angle
360
Circumference = 2 r
Diameter = 2r Arc Length = Circumference x Arc Angle
360

SAT v17.10 Unauthorized copying or reuse of any part of this page is illegal. 487
(SAT Teacher Edition)
MAKE SURE YOU KNOW THE BASICS ABOUT CIRCLES

1. 4.

π 4π

2. 5.

2.5
5

3. 6.

488
Circles

1 (No Calculator) 3 (No Calculator)

2 (No Calculator) 4 (No Calculator)

2/3
/ /
. . . .
0 0 0

SAT v17.10 Unauthorized copying or reuse of any part of this page is illegal. 489
(SAT Teacher Edition)
490
Area & Volume

1 (No Calculator)

SAT v17.10 Unauthorized copying or reuse of any part of this page is illegal. 491
(SAT Teacher Edition)
OUR APPROACH: th e in fo rm a tion we've been
TRAPEZOID (N): a quadrilateral with
exactly one set of parallel sides, or bases.
pl u g
All we need to dormis ula for the area of a trapezoid.
given into the fo

is
( )
56 = 7 13 +2
so
b
have
lve for b and we'lldes by 7.
e ha ve to do
Then all w et's start by dividing both si
our answer! L AREA OF A TRAPEZOID:
( )
8 = 13 +2 b
si de s by 2 to ge t rid of the fractio
n.
( )
A = h x b1 + b2
Then we'll multiply
both 2
16 = 13 + b

fr om bo th si de s and you're done!


Subtract 13
3=b
1 (No Calculator)

a misleading
Don't get fooled abyy this is drawn
diagram. The w you the impression
sure doesn't give That's intentional.
that b is only 3. alled" the diagram
And if you "eyeb 9, then you got
and guessed 8 or
tricked!

15 5 x

13
x

29 2x

Height: ______
12 Area: ______
264 Height: ______
x Area: ______
2x2

492
PARALLELOGRAM (N):
17 a quadrilateral with exactly two sets of
parallel sides, but whose angles are
not necessarily 90°

10 h

AREA OF A PARALLELOGRAM

A=b xh
Height: ______
6 Area: ______
102
To find the height of a parallelogram,
140 draw a perpendicular line from one
base to the opposite base, creating a
right triangle. Then, use Pythagorean
Theorem to solve for the length of the
50 2 perpendicular line you drew.

50

SPECIAL RIGHT TRIANGLES


Height: ______
50 Area: ______
7000 TO THE RESCUE!

Parallelograms aren't very difficult by


themselves, so the test makers mix
things up by disguising information
you need to solve the problem.

One of the ways they do this is by


3 providing you with incomplete special
30° right triangles that you can use to
35 identify the height or side length of a
parallelogram.

So before you begin using


Pythagorean Theorem, take a moment
to see if there's a quicker method to
find the missing information!
Height: ______
3 Area: ______
9√3 + 105

SAT v17.10 Unauthorized copying or reuse of any part of this page is illegal. 493
(SAT Teacher Edition)
Area and Volume

1 (No Calculator) 3

2 (No Calculator) 4

494
Complex Numbers

1 (No Calculator)

SAT v17.10 Unauthorized copying or reuse of any part of this page is illegal. 495
(SAT Teacher Edition)
IMAGINARY NUMBER (N):
OUR APPROACH: h im a gi na ry (c om plex) numbers is An imaginary number is a complex
Addition and subt raction wit raction with polynomials. The number that can be written as a real
ition and subt ly adding and
very similar to addto be very careful that you're on number multiplied by the imaginary unit
main challenge is terms." i, which is defined by its property i2 = -1.
subtracting "like ns that you needif
plex numbers , lik e te rm s m ea
parately. So i=
In the case of coremal terms and imaginary terms se
to work on the COMPLEX CONJUGATE (N):
you've got: Denoted as z* where the sign of the

' ' '


imaginary part is opposite.

a n d z = a + b i
z=a+bi If z = a + bi, then the complex conjugate of
z is z* = a - bi. The complex conjugate is

REAL part of Z
then...
z + z = (a + 'a ) + (b i + b) ' ' sometimes referred to as the C.C.

1 (No Calculator)
IMAGINARY
part of z

b, but we're
This is the value of
looking for a + b.
a, but again,
Here's the valueedofto find a + b.
we've been ask
d
er pr op er ty of co mplex numbers caoflle
se anoth parts
For division, we uju ga te to ge t ri d of the imaginathrye top and bottom
the Complex Con ons. It's sort of like multiplying lp you set up a
complex expressi the same number in order to headvantage of the
i2 = -1
of a fraction byator. In this case, we're taking plex Conjugate
common denomin plex number multiplied by its Com t:
fact that a comlt in a REAL number. Check it ou 15 i = 5 + 3i
will always resu
i+2 2 i - 11 i2
= 5 2 5 +
11 i) (2 - i) = 14 - 7
(7 + 11 i) (2 - i) = (7 + i - i2 5
i
7 + 11 = (2 + i)(2 - i) 4 - 2i + 2
2+i
2
i = -1 a + b i = 5 + 3 i
This is the
same thing a + b = 5+3
as multiplying
by 11 Looks likew8er!
is our ans
496
i IS SUCH A LONELY WEIRD NUMBER
You probably know that a negative number squared results in a positive number.
If you're really smart, then you know that a negative number to the third power is
negative again. Fourth power? Positive. And so on. Negative numbers raised to
even powers are positive. Negative numbers raised to odd powers are odd. You
might think that i that would behave similarly, but it's completely different.

i1 = i i5 = i i13 = i
i2 = −1 i6 = −1 i30 = −1
i3 = −i i7 = −i i35 = −i
i4 = 1 i8 = 1 i52 = 1

As you can see, i raised to various powers just repeats the same four results. You can
use that information to help you convert i to a real number in some problems. Just
replace the exponent with the remainder when the exponent is divided by four.

SOLVE THESE PROBLEMS

1. 3.

2. 4.

SAT v17.10 Unauthorized copying or reuse of any part of this page is illegal. 497
(SAT Teacher Edition)
Complex Numbers

1 (No Calculator) 3 (No Calculator)

Basic Trigo

2 (No Calculator) 4 (No Calculator)

498
Basic Trigonometry

1 (No Calculator)

onometry

SAT v17.10 Unauthorized copying or reuse of any part of this page is illegal. 499
(SAT Teacher Edition)
OUR APPROACH: All we
st raightforward. th
Thi s on e is pr et ty
e a ns w er ch oi ce at sets up
th
have to do is findo using the information that we TRIGONOMETRIC RATIOS: the lengths of the
adjacent side, opposite side, and hypotenuse may be
the correct rati in the problem. determined using these ratios
have been given
info
le t' s ta ke in ve nt ory of the pieces of sin A =
opposite
=
a
First, hypotenuse c B
we've been given:
θ
and the ground = c
use

opposite
ee n th e la dd er adjacent b ten
The angle betw nt to that angle is 12 ft long. cos A =
hypotenuse
=
c hyp
o a

The side adjace is 60 ft long. A


adjacent
The hypotenuse opposite b
b C

ig ra ti o in vo lv in g the adjacent side tan A =


adjacent
=
c
Looks like the trnuse is cosine. Let's check answer
and the hypote
choice (B). adjacent = 60 12
cos A = hypotenuse
1 (No Calculator)

AL's APPLIANCES
60 ft.

θ
12 ft.

USE SOH - CAH - TOA TO REMEMBER


THESE THREE TRIG RATIOS!

S-O-H: Sin = Opposite / Hypotenuse

C-A-H: Cosine = Adjacent / Hypotenuse

T-O-A: Tangent = Opposite / Adjacent

500
TRY YOUR HAND AT A FEW BASIC TRIG PROBLEMS

A C
4

1. What is the sine of A in right triangle ABC above? 3.

√5
3

2. In right triangle ABC, if A and B are acute and if 4.


7
tan B = , what is the value of sin A? 7:5
5

5
√74

SAT v17.10 Unauthorized copying or reuse of any part of this page is illegal. 501
(SAT Teacher Edition)
Basic Trigonometry

1 (No Calculator) 3 (No Calculator)

2 (No Calculator) 4 (No Calculator)

502
Math Unit #5
Homework Drill #1
1 (No Calculator) 3 (No Calculator)

2 (No Calculator) 4 (No Calculator)

SAT v17.10 Unauthorized copying or reuse of any part of this page is illegal. 503
(SAT Teacher Edition)
Math Unit #5
Homework Drill #2
1 (No Calculator) 3 (No Calculator)

2 4

504
Math Unit #5
Homework Drill #3
1 (No Calculator) 3

2 (No Calculator) 4 (No Calculator)

SAT v17.10 Unauthorized copying or reuse of any part of this page is illegal. 505
(SAT Teacher Edition)
506

You might also like